Case Digest Consti

You might also like

Download as docx, pdf, or txt
Download as docx, pdf, or txt
You are on page 1of 62

12.14 ALVAREZ V.

CFI 64 PHIL 33 has been drawn in such a manner that perjury could
be charged thereon and affiant be held liable for
Facts: On 3 June 1936, the chief of the secret service damages caused. The affidavit, which served as the
of the Anti- Usury Board, of the Department of exclusive basis of the search warrant, is insufficient
Justice,presented to Judge Eduardo Gutierrez David and fatally defective by reason of the manner in which
then presiding over the Court of First Instance of the oath was made, and therefore, the search warrant
Tayabas, an affidavit alleging that according to and the subsequent seizure of the books, documents
reliable information, Narciso Alvarez kept in his and other papers are illegal. Neither the Constitution
house in Infanta, Tayabas, books, documents, nor General Orders 58 provides that it is of
receipts, lists, chits and other papers used by him in imperative necessity to take the depositions of the
connection with his activities as a moneylender, witnesses to be presented by the applicant or
charging usurious rates of interest in violation of the complainant in addition to the affidavit of the latter.
law. In his oath at the end of the affidavit, the chief of The purpose of both in requiring the presentation of
the secret service stated that his answers to the depositions is nothing more than to satisfy the
questions were correct to the best of his knowledge committing magistrate of the existence of probable
and belief. He did not swear to the truth of his cause. Therefore, if the affidavit of the applicant or
statements upon his own knowledge of the facts but complainant is sufficient, the judge may dispense with
upon the information received by him from a reliable that of other witnesses. Inasmuch as the affidavit of
person. Upon the affidavit the judge, on said date, the agent was insufficient because his knowledge of
issued the warrant which is the subject matter of the the facts
petition, ordering the search of the Alvarez's house at
any time of the day or night, the seizure of the books 12.15 WEBB V. DE LEON GR 121234 AUG. 23,1995
and documents and the immediate delivery thereof to
him to be disposed of in accordance with the law. With Facts:On June 19, 1994, the National Bureau of
said warrant, several agents of the Anti-Usury Board Investigation filed with the DOJ a letter-complaint
entered Alvarez's store and residence on 4 June 1936, charging petitioners Hubert Webb, Michael
and seized and took possession of his confidential Gatchalian, Antonio J. Lejano and 6 other persons
files. The search for and seizure of said articles were with the crime of Rape and Homicide of Carmela N.
made with the opposition of Alvarez who stated his Vizconde, her mother Estrellita Nicolas-Vizconde, and
protest below the inventories on the ground that the her sister Anne Marie Jennifer in their home at
agents seized even the originals of the documents. As Number 80 W. Vinzons, St., BF Homes, Paranaque,
the articles had not been brought immediately to the Metro Manila on June 30, 1991. Forthwith, the DOJ
judge who issued the search warrant, Alvarez, formed a panel of prosecutors headed by Asst Chief
through his attorney, filed a motion on 8 June 1936, State Prosecutor Jovencio R. Zuno to conduct the
praying that the agent Emilio L. Siongco, or any other preliminary investigation. The DOJ Panel for its
agent, be ordered immediately to deposit all the seized finding of probable cause. The credibility of Jessica
articles in the office of the clerk of court and that said Alfaro was assailed as inherently weak and
agent be declared guilty of contempt for having uncorroborated due to her inconsistencies between
disobeyed the order of the court. her April 28, 1995 and May 22, 1995 sown
statements. They criticize the procedure followed by
Issue:Whether the search warrant issued by the court the DOJ Panel when it did not examine witnesses to
is illegal because it has been based upon the affidavit clarify the alleged inconsistencies. Petitioners charge
ofagent Almeda in whose oath he declared that he had that respondent Judge Raul de Leon and respondent
no personal knowledge of the facts. Judge Amelita Tolentino issued warrants of arrest
against them without conducting the required
Ruling:Section 1, paragraph 3, of Article III of the preliminary examination. Complain about the denial
Constitution and Section 97 of General Orders 58 of their constitutional right to due process and
requirethat there be not only probable cause before violation of their right to an impartial investigation.
the issuance of a search warrant but that the search They also assail the prejudicial publicity that
warrant must be based upon an application supported attended their preliminary investigation.
by oath of the applicant and the witnesses he may
produce. The oath required must refer to the truth of ISSUES: W/N respondent judges de Leon and
the facts within the personal knowledge of the Tolentino gravely abuse their discretion when they
petitioner or his witnesses, because the purpose failed to conduct a preliminary examination before
thereof is to convince the committing magistrate, not issuing warrants of arrest against the accused?
the individual making the affidavit and seeking the
issuance of the warrant, of the existence of probable Ruling: NO. It is a valid arrest -- In arrest cases,
cause. The true test of sufficiency of an affidavit to there must be a probable cause that a crime has been
warrant issuance of a search warrant is whether it committed and that the person arrested committed it.
Consti II (section 2) 1
Section 6 of Rule 112 provides that – “upon filing of 1866 (Illegal Possession of Firearm, etc). In the
an information, the RTC may issue a warrant for the deposition of witness (P/Lt. Florencio C. Angeles), it
accused.” Clearly then, our laws repudiate the was made mentioned of “result of our continuous
submission that respondent judges should have surveillance conducted for several days. We gathered
conducted “searching examination of witnesses” information from verified sources that the holders of
before issuing warrants of arrest against them. said firearms and explosives as well as ammunitions
aren’t licensed to possess said firearms and
For Search: ammunition. Further, the premises is a school and the
holders of these firearms are not student who were
12.16 BURGOS V. CHIEF OF STAFF 133 SCRA 800 not supposed to possess firearms, explosives and
ammunitions. Person to be searched in Nemesio
Facts: Armed with a search warrant issued by the
Prudente at the Polytechnic University of the
Court of First Instance of Rizal, law enforcement
Philippines, Sta. Mesa, Sampaloc, Manila, has in his
officers searched the offices of the “We forum” and
control or possession firearms, explosives hand
“Metropolitan Mail” newspapers. During the course of
grenades and ammunitions which are illegally
the search, the law enforcement officers seized office
possesses at the office of Department of Military
and printing machines, equipment, paraphernalia and
Science and Tactics and at the office of the President.
several other materials used in the distribution of
newspapers. Petitioner avers, among others, that the Petitioner moved to quash the Search Warrant. He
seizure of the properties mentioned above amounts to claimed that:
seizure of real properties, which cannot be validly
conducted under the strength of a search warrant. It 1. Petitioners, had no personal knowledge of the
must be noted that real properties are not susceptible facts
of confiscation under a search warrant. Hence this
appeal which assails the validity of the search and the 2. The examination of the said witness was not
seizure of the properties of the petitioner. in form of searching questions and answers

Issue: Whether there is merit in the petitioner’s 3. Search warrant was a general warrant
assertion that real property were invalidly seized
under the disputed warrants. 4. Violation of Circular No. 19 of the Supreme
Court in that the complainant failed to allege
Ruling: No. The petitioner’s assertion does not hold under oath that the issuance of the search
water. Under Article 415(5) of the civil code, warrant on a Saturday, urgent.
“machinery, receptacles, instruments or implements
intended by the owner of the tenement for an industry ISSUE: Whether or not the search and seizure was
or works which may be carried on in a building or on a valid?
piece of land and which tend directly to meet the
needs of the said industry or works” are considered Ruling: No. Valid search warrant to issue, there
immovable property. In another case decided by the must be probable cause, which is to be determined
Court, in which the abovementioned legal provision personally by the Judge, after examination under
was invoked, it was ruled that machinery which is oath and affirmation of the complainant, and that
movable by nature becomes immobilized when placed witnesses he may produce and particularly describing
by the owner of the tenement, property or plant, but the place to be searched and the persons and things to
not so when placed by a tenant, usufructuary, or any be seized. The probable cause must be in connection
other person having only temporary right, unless such with one specific offense and the Judge must, before
person acted as the agent of the owner. In the case at issuing Search Warrant, personally examine in the
bar, petitioners do not claim to be the owners of the form of searching questions and answers, In writing
land and/or building on which the machineries were and under oath, the complainant and any witnesses
placed. This being the case, the machineries in he may produce, on facts personally known to them
question, while in fact bolted to the ground remains and attach to the record their sworn statements
movable property susceptible to seizure under a together with any affidavits submitted.
search warrant.
“Probable Cause” for a valid search warrant, has been
12.17 PRUDENTE V. DAYRIT 180 SCRA 69 defined as such facts and circumstances which would
lead a reasonably discreet and prudent man to believe
FACTS: The Chief of the Intelligence Special Action that an offense has been committed, and that objects
Division (ISAD) filed with the Regional Trial Court sought in connection which the offense are in the
(RTC) Manila, Judge Abelardo Dayrit, for the place sought to be searched. This probable case must
issuance of Search Warrant for violation of PD No. be shown to be personal knowledge and of the

Consti II (section 2) 2
complainant and witnesses he may produce and not arrest was correspondingly issued. Respondent judge,
based on mere hearsay. in a motion to quash warrant of arrest, dismissed the
case against Billy for insufficiency of evidence,
12.18 UNITED STATES V. JONES JANUARY 23, recalled the warrant issued, and ordered the
2012 prosecution to withdraw its amended information and
to file a new one charging Jonathan Cerbo only. The
Facts: Antoine Jones was arrested on Oct. 24, 2005, private prosecutor's motion for reconsideration was
for drug possession after police attached a tracker to denied, hence, his resort to the Court of Appeals. The
Jones's Jeep -- without judicial approval -- and used it appellate court, in affirming the trial court held that
to follow him for a month. A jury found Jones not the trial court RTC has authority to reverse the
guilty on all charges save for conspiracy, on which prosecutor's finding of probable cause and dismiss the
point jurors hung. District prosecutors, upset at the information on the ground that the evidence
loss, re-filed a single count of conspiracy against Jones presented did not substantiate the charge.
and his business partner, Lawrence Maynard. Jones
owned the "Levels" nightclub in the District of Issue: Whether the Court of Appeals erred in finding
Columbia. Jones and Maynard were then convicted, that no probable cause exists to merit the filing of
but a three-judge panel of the U.S. Court of Appeals charges against private respondent Billy Cerbo.
for the D.C. Circuit ruled that the Supreme Court
specifically stated in a 1983 case regarding the use of Ruling: Yes. The determination of probable cause to
a beeper to track a suspect that the decision could not hold a person for trial is a function that belongs to the
be used to justify 24-hour surveillance without a public prosecutor. It is an executive function, the
warrant. correctness of the exercise of which is a matter that
the trial court itself does not and may not be
Issue: W/N the warrantless use of a tracking device compelled to pass upon. The judge should not override
on Jones's vehicle to monitor its movements on public the public prosecutor's determination of probable
streets violate Jones' Fourth Amendment rights? cause on the ground that the evidence presented to
substantiate the issuance of a warrant of arrest was
Ruling: Yes. The Court affirmed the judgment of the insufficient. As a general rule, if the information is
lower court, and held that the installation of a GPS valid on its face and there is no showing of manifest
tracking device on Jones' vehicle, without a warrant, error, grave abuse of discretion or prejudice on the
constituted an unlawful search under the Fourth part of the public prosecutor, courts should not
Amendment. The Court rejected the government's dismiss it for want of evidence because evidentiary
argument that there is no reasonable expectation of matters should be presented and heard during the
privacy in a person's movement on public trial.
thoroughfares and emphasized that the Fourth
Amendment provided some protection for trespass III. Kinds of Evidence Needed to Establish Probable
onto personal property. Cause

Justice Sonia Sotomayor wrote a concurring opinion, 12.20 MICROSOFT CORP. V. MAXICORP, GR
agreeing that the government had obtained 140946
information by usurping Jones' property and by
invading his privacy. However, she further reasoned FACTS : This case involves the issuance of search
that the Fourth Amendment was not only concerned warrant to the respondent MAXICORP Inc for alleged
with trespasses onto property. She stated that a violation of Section 29 of Intellectual Property and
Fourth Amendment search occurs whenever the Article 189 of the RPC (unfair competition). Armed
government violates a subjective expectation of with the search warrants, NBI agents conducted a
privacy that society recognizes as reasonable, which is search of Maxicorp’s premises and seized property
particularly important in an era where physical fitting the description stated in the search warrants.
intrusion is unnecessary to many forms of Maxicorp filed a motion to quash the search warrants
surveillance. alleging that there was no probable cause for their
issuance and that the warrants are in the form of
II Who determines Probable Cause "general warrants." Where the RTC denied the motion
and at the same time denied their motion for
12.19 PEOPLE V. CA GR 126005 JAN. 21, 1999 reconsideration. According to RTC they’ve found a
probable cause to issue such warrant after examining
Facts: Jonathan Cerbo was charged in an information
the NBI agent and the computer technician who
for murder in connection with the fatal shooting of
visited Maxicorp. Maxicorp filed a petition for
Rosalinda Dy inside the office of his father, Billy
certiorari with the Court of Appeals seeking to set
Cerbo. The information was amended to include Billy
aside the RTC’s order. The Court of Appeals reversed
Cerbo as one of the accused and a warrant for his
Consti II (section 2) 3
the RTC’s order denying Maxicorp’s motion to quash judge the computer unit that he purchased from
the search warrants. Petitioners moved for Maxicorp, in which computer unit Maxicorp had pre-
reconsideration. The Court of Appeals denied installed petitioners’ software.
petitioners’ motion on 29 November 1999. The Court
of Appeals held that NBI Agent Samiano failed to IV. In General
present during the preliminary examination
conclusive evidence that Maxicorp produced or sold 13.11 Nala v. Barroso, GR 153087 Aug. 7, 2003
the counterfeit products. The Court of Appeals
pointed out that the sales receipt NBI Agent Samiano
presented as evidence that he bought the products FACTS: On June 2001, PO3 Alcoser together with
from Maxicorp was in the name of a certain "Joel his witness applied for the issuance of a warrant to
Diaz." search the person and residence of petitioner Bernard
R. Nala, who was referred to in the application as
ISSUE: Whether or not there’s a probable cause on
“Rumolo Nala alias Long” of “Purok 4, Poblacion,
the part of CA to quash the search warrants issued by
Kitaotao, Bukidnon.” The application was filed in
RTC
connection with petitioner’s alleged illegal possession
RULING: Probable cause means "such reasons, of one caliber .22 magnum and one 9 mm. pistol in
supported by facts and circumstances as will warrant violation of Illegal Possession of Firearms. On the
a cautious man in the belief that his action and the same day, respondent Presiding Judge of RTC of
means taken in prosecuting it are legally just and Malaybalay City, issued Search and Seizure Warrant.
proper."Thus, probable cause for a search warrant The fact that the items seized were not exactly the
requires such facts and circumstances that would lead items listed in the warrant does not invalidate the
a reasonably prudent man to believe that an offense same because the items seized bear a direct relation
has been committed and the objects sought in to the crime of illegal possession of firearms.
connection with that offense are in the place to be
(Respondent judge also found that petitioner was
searched. The testimonies of these two witnesses,
sufficiently identified in the warrant although his first
coupled with the object and documentary evidence
name was erroneously stated therein as “Romulo” and
they presented, are sufficient to establish the
not “Bernard”, considering that the warrant was
existence of probable cause. From what they have
couched in terms that would make it enforceable
witnessed, there is reason to believe that Maxicorp
against the person and residence of petitioner and no
engaged in copyright infringement and unfair
other.)
competition to the prejudice of petitioners. Both NBI
Agent Samiano and Sacriz were clear and insistent ISSUE: Was petitioner sufficiently described in the
that the counterfeit software were not only displayed search and seizure warrant?
and sold within Maxicorp’s premises, they were also
produced, packaged and in some cases, installed RULING: YES. the failure to correctly state in the
there. The determination of probable cause does not search and seizure warrant the first name of
call for the application of rules and standards of proof petitioner, which is “Bernard” and not “Romulo” or
that a judgment of conviction requires after trial on “Rumolo”, does not invalidate the warrant because the
the merits. As implied by the words themselves, additional description “alias Lolong Nala who is said
"probable cause" is concerned with probability, not to be residing at Purok 4, Poblacion, Kitaotao,
absolute or even moral certainty. The prosecution Bukidnon” sufficiently enabled the police officers to
need not present at this stage proof beyond reasonable locate and identify the petitioner. . What is
doubt. The standards of judgment are those of a prohibited is a warrant against an unnamed party,
reasonably prudent man,not the exacting calibrations and not one which, as in the instant case, contains a
of a judge after a full-blown trial. No law or rule description personae that will enable the officer to
states that probable cause requires a specific kind of identify the accused without difficulty.
evidence. No formula or fixed rule for its
determination exists.Probable cause is determined in WHEREFORE, in view of all the foregoing, the
the light of conditions obtaining in a given situation. 26 petition is GRANTED. Search and Seizure Warrant
Thus, it was improper for the Court of Appeals to is declared VOID and the articles seized by virtue
reverse the RTC’s findings simply because the sales thereof are declared inadmissible in evidence.
receipt evidencing NBI Agent Samiano’s purchase of
counterfeit goods is not in his name. For purposes of 13.12 Betoy v. Judge MAMERTO Y. COLIFLORES,
determining probable cause, the sales receipt is not A.M. No. MTJ-05-1608, February 28, 2006
the only proof that the sale of petitioners’ software
occurred. During the search warrant application Facts: In a Letter-Complaint dated July 2000, signed
proceedings, NBI Agent Samiano presented to the by Bernardo Betoy, Sr. (complainant) charges Judge
Consti II (section 2) 4
Mamerto Y. Coliflores (respondent) with Grave Abuse The Court DISMISSED the petition and AFFIRMED
of Discretion and Authority, Conduct Unbecoming as the questioned decision and resolution of the CA.
a Judge and Gross Negligence Resulting to Procedural
Lapses (Dereliction of Duty).Complainant attached to 13.14 Columbia Pictures v. CA, 262 SCRA 219
the Letter-Complaint an Affidavit executed by his
wife Lucia Betoy. Facts:In 1986, the Videogram Regulatory Board
(VRB) applied for a warrant against Jose Jinco
Issue: Whether respondent retired Judge was found (Jingco), owner of Showtime Enterprises for allegedly
guily of gross ignorance of the law. pirating movies produced and owned by Columbia
Pictures and other motion picture companies. Jingco
Ruling: Yes. Respondent retired Judge Mamerto Y. filed a motion to quash the search warrant but the
Coliflores is found guilty of gross ignorance of the law. same was denied in 1987. Subsequently, Jinco filed an
He is fined P20,000.00 to be deducted from his Urgent Motion to Lift the Search Warrant and Return
retirement benefits. the Articles Seized. In 1989, the RTC judge granted
the motion. The judge ruled that based on the ruling
Under Section 8(9), Rule 140 of the Rules of Court, as in the 1988 case of 20th Century Fox Film
amended, gross ignorance of the law or procedure is Corporation vs CA, before a search warrant could be
classified as a serious charge. Section 11(A) of the issued in copyright cases, the master copy of the films
same Rule provides that the penalty to be imposed if a alleged to be pirated must be attached in the
respondent is found guilty of a serious charge is either application for warrant.
a fine of more than P20,000.00 but not more
thanP40,000.00, suspension from office without salary ISSUE: Whether the 20th Century Fox ruling may be
and other benefits for more than three (3) but not applied retroactively in this case.
exceeding six (6) months, or dismissal from the service,
forfeiture of all or part of the benefits as the Court may HELD: No. In 1986, obviously the 1988 case of 20th
determine, and disqualification from reinstatement or Century Fox was not yet promulgated. The lower
appointment to any public office, including court could not possibly have expected more evidence
government-owned or controlled corporations. from the VRB and Columbia Pictures in their
application for a search warrant other than what the
13.13 20th Century Fox v. CA, 162 SCRA 655; G.R. law and jurisprudence, then existing and judicially
Nos. 76649-51, August 19, 1988 accepted, required with respect to the finding of
probable cause.
Facts: Petitioner 20th Century Fox Film Corporation
sought the assistance of the NBI in conducting B. Personally Determined by the Judge
searches and seizures in connection with the NBI’s
anti-film piracy campaign. Petitioner alleged that 13.15 Placer v. Villanueva, 126 SCRA 463
certain videotape outlets all over Metro Manila are
engaged in the unauthorized sale and renting out of Facts: Petitioners filed information in the city court
copyrighted films in violation of PD No. 49 (the old and they certified that Preliminary Investigation and
Intellectual Property Law). Examination had been conducted and that prima facie
cases have been found. Upon receipt of said
The NBI conducted surveillance and investigation of information, respondent judge set the hearing of the
the outlets pinpointed by the petitioner and criminal cases to determine propriety of issuance of
subsequently filed three (3) applications for search warrants of arrest. After the hearing, respondent
warrants against the video outlets owned by the issued an order requiring petitioners to submit to the
private respondents. The lower court issued the court affidavits of prosecution witnesses and other
desired search warrants. The NBI, accompanied by documentary evidence in support of the information to
the petitioner's agents, raided the video outlets and aid him in the exercise of his power of judicial review
seized the items described in the three warrants. of the findings of probable cause by petitioners.
Petitioners petitioned for certiorari and mandamus to
Issue: Did the judge properly lift the search warrants compel respondent to issue warrants of arrest. They
he issued earlier? contended that the fiscal’s certification in the
information of the existence of probable cause
Ruling: YES, the judge properly lifted the search constitutes sufficient justification for the judge to
warrants he issued earlier. This linkage of the issue warrants of arrest.
copyrighted films to the pirated films must be
established to satisfy the requirements of probable Issue: Whether respondent city judge may, for the
cause. Mere allegations as to the existence of the purpose of issuing warrants of arrest, compel the
copyrighted films cannot serve as basis for the fiscal to submit to the court the supporting affidavits
issuance of a search warrant.
Consti II (section 2) 5
and other documentary evidence presented during the permission or clearance from COMELEC as required
preliminary investigation. by law.

Held: Judge may rely upon the fiscal’s certification After a preliminary investigation of Barba’s
for the existence of probable cause and on the basis complaint, Atty. Lituanas found a prima facie case.
thereof, issue a warrant of arrest. But, such Hence, on September 1988, he filed with the
certification does not bind the judge to come out with respondent trial court a criminal case for violation of
the warrant. The issuance of a warrant is not a mere section 261, Par. (h), Omnibus Election Code against
ministerial function; it calls for the exercise of judicial the OIC-Mayor. In an Order dated September 30,
discretion on the part of issuing magistrate. Under 1988, the respondent court issued a warrant of arrest
Section 6 Rule 112 of the Rules of Court, the judge against the accused OIC Mayor.
must satisfy himself of the existence of probable cause
before issuing a warrant of arrest. If on the face of the Issue: Does a preliminary investigation conducted by
information, the judge finds no probable cause, he a Provincial Election Supervisor involving election
may disregard the fiscal’s certification and require offenses have to be coursed through the Provincial
submission of the affidavits of witnesses to aid him in Prosecutor, before the Regional Trial Court may take
arriving at the conclusion as to existence of probable cognizance of the investigation and determine
cause. Petition dismissed. whether or not probable cause exists?

13.16 Lim v. Judge Fenix, 194 SCRA 292 Held: The 1987 Constitution empowers the
COMELEC to conduct preliminary investigations in
FACTS: On March 1989, at about 7:30 o'clock in the cases involving election offenses for the purpose of
morning, at the vicinity of the airport road of the helping the Judge determine probable cause and for
Masbate Domestic Airport, located at the municipality filing an information in court. This power is exclusive
of Masbate province of Masbate, Congressman Moises with COMELEC. The evident constitutional
Espinosa, Sr. and his security escorts, namely intendment in bestowing this power to the COMELEC
Provincial Guards were attacked and killed by a lone is to insure the free, orderly and honest conduct of
assassin. Mr. Siblante another security escort of elections, failure of which would result in the
Congressman Espinosa, Sr. survived the frustration of the true will of the people and make a
assassination plot, although, he himself suffered a mere idle ceremony of the sacred right and duty of
gunshot wound. An investigation of the incident then every qualified citizen to vote. To divest the
followed. COMELEC of the authority to investigate and
prosecute offenses committed by public officials in
After conducting the preliminary investigation, the relation to their office would thus seriously impair its
court issued an order concluding that a probable effectiveness in achieving this clear constitutional
cause has been established for the issuance of a mandate.
warrant of arrest of named accused.
13.18 People v. Delgado, 189 SCRA 715, 189 SCRA
ISSUE : Whether or not a judge may issue a warrant 715, 1990
of arrest without bail by simply relying on the
prosecution's certification and recommendation that a Facts: On January 1988 the COMELEC received a
probable cause exists. report-complaint from the Election Registrar of Toledo
City against private respondents for alleged violation
HELD: If a Judge relies solely on the certification of of the Omnibus Election Code. The COMELEC
the Prosecutor as in this case where all the records of directed the Provincial Election Supervisor of Cebu to
the investigation are in Masbate, he or she has not conduct the preliminary investigation of the case who
personally determined probable cause. The eventually recommended the filing of an information
determination is made by the Provincial Prosecutor. against each of the private respondents for violation of
The constitutional requirement has not been satisfied. the Omnibus Election Code. The COMELEC en banc
The Judge commits a grave abuse of discretion. resolved to file the information against the private
respondents as recommended.
13.17 People v. Inting, 187 SCRA 788
Private respondents filed motions for reconsiderations
Facts: Mrs. Editha Barba filed a letter-complaint and the suspension of the warrant of arrest with the
against OIC-Mayor Dominador Regalado of Tanjay, respondent court on the ground that no preliminary
Negros Oriental with the COMELEC for allegedly investigation was conducted. Later, an order was
transferring her, a permanent Nursing Attendant, issued by respondent court directing the COMELEC
Grade I, in the office of the Municipal Mayor to a very through the Regional Election Director of Region VII
remote barangay and without obtaining prior to conduct a reinvestigation of said cases. The

Consti II (section 2) 6
COMELEC Prosecutor filed a motion for probable cause even if the accused was not in the
reconsideration and opposition to the motion for custody of the court.
reinvestigation alleging therein that it is only the
Supreme Court that may review the decisions, orders, Held: Yes. The Supreme Court issued a temporary
rulings and resolutions of the COMELEC. restraining order enjoining the PACC from enforcing
the warrant of arrest and the respondent judge
Issue: Whether the (RTC) has the authority to review therein from further proceeding in the case on the
the actions of the Commission on Elections ground of lack of probable cause. As with other earlier
(COMELEC) in the investigation and prosecution of cases resolved by the high court, the accused is
election offenses filed in said court. deemed to have submitted himself to the jurisdiction
of the court upon seeking affirmative relief.
Held: Based on the Constitution and the Omnibus Notwithstanding such, there is no requirement that
Election Code, it is clear that aside from the the accused be in the custody of the law. Various
adjudicatory or quasi-judicial power of the COMELEC reliefs can be granted by the Supreme Court to
to decide election contests and administrative accused even if they are not in the custody of the law.
questions, it is also vested the power of a public
prosecutor with the exclusive authority to conduct the 13.20 Gozos v. Tac-an – GR 123191, Dec. 17, 1998
preliminary investigation and the prosecution of
election offenses punishable under the Code before Facts: On February 1995, the officials, teachers, and
the competent court. Thus, when the COMELEC, students of the Concepcion Aguila Memorial College
through its duly authorized law officer, conducts the in San Jose, Batangas organized a school party. While
preliminary investigation of an election offense and the party was going on, the principal, Felizardo
upon a prima facie finding of a probable cause, files Aguila, was informed that several men, who appeared
the information in the proper court, said court thereby to be drunk, were trying to force their way through
acquires jurisdiction over the case. Consequently, all the main gate. One of the men seemed armed with a
the subsequent disposition of said case must be handgun. After calling the police, Aguila went to the
subject to the approval of the court. The COMELEC main gate, where he asked the men what their
cannot conduct a reinvestigation of the case without business was.
the authority of the court or unless so ordered by the
court.

13.19 Allado v. Diokno – 232 SCRA 192 At this point, private respondents Blanco and Atienza
arrived at the school. They were shortly joined by
Facts: On September 1993, a Security Guard and a private respondents Pedro Castillo, Sulit, and
discharged Philippine Constabulary named Ildefonso Castillo, who were all members of the
Escolastico Umbal executed a sworn statement Philippine National Police of San Jose, Batangas.
implicating petitioners Allado and Mendoza who are They demanded from the man armed with a handgun,
partners in the Law Firm of Salonga, Hernandez and who later turned out to be the victim Gilbert Dyogi,
Allado. He accused them as the brains behind the that he surrender his gun and go with them to the
alleged kidnapping and slaying of Eugen Van Twest, station. Gilbert Dyogi gave the handgun to the
a German national. Based on that confession of respondents, who then asked to see his license. He
Umbal, a search warrant was issued by Judge Roberto produced a sheet of paper from his wallet which he
Barrios of the RTC of Manila. handed to private respondents. After allowing them to
inspect the weapon and the alleged license, Gilbert
Then, the operatives of the Presidential Anti-Crime Dyogi asked the private respondents to give them
Commission (PACC), armed with the search warrant back to him. However, private respondent Blanco,
issued separately raided the dwellings of police who had the gun, refused to do so.
officers who were also pointed by Umbal as the
perpetrators of the crimes. Several firearms and What exactly followed is unclear. Before long the two
ammunitions were found in the raid including Van were grappling for possession of the gun. Apparently,
Twest's Cartier sunglasses. So, the two lawyers and Blanco pulled out his sidearm and fired at Gilbert
their other co-defendants were charged with illegal Dyogi twice. Order dated January 1996 Respondent
possession of firearms and ammunitions, carnapping, judge Tac-an denied the second motion for
kidnapping for ransom with murder, and usurpation reconsideration filed by complainant Edna Dyogi,
of authority. questioning the authority of respondent to require the
Provincial Prosecutor to amend the information. (long
Issue: Whether a warrant of arrest without bail can facts to be substantial)
be set aside and the case be dismissed for lack of

Consti II (section 2) 7
Issue: Whether or not there is probable cause against 4. There was no cause under the law to arrest
all the accused before the issuance of a warrant of them as the possibility of fleeing to escape the
arrest hands of justice is remote as they are
barangay officials.
Ruling: The established rule is that a preliminary
investigation is not the occasion for the full and In response to the charge, the respondent judge said
exhaustive display of the parties evidence; it is for the that the quantum of evidence required in preliminary
presentation of such evidence only as may engender a investigation had likewise been observed. He
well-grounded belief that an offense has been contended that, by posting bail, complainants waived
committed and that the accused is probably guilty objection to any irregularities which might have been
thereof. committed in the course of the preliminary
investigation.
Hence, notwithstanding the contrary opinion of the
judge regarding the designation of the offense ISSUE:When may an investigating judge issue a
committed, for as long as he finds probable cause for warrant of arrest of the accused?
the offense charged, he should issue a warrant of
arrest against the accused for the crime charged in HELD:What differentiates the present rule from the
the information.Wherefore, the petition is hereby previous one before the 1985 revision of the Rules on
granted and the orders, dated October 1995 , Criminal Procedure is that while before, it was
November 1995, and January 1996, of respondent mandatory for the investigating judge to issue a
Judge Paterno tac-an are annulled and set aside. warrant for the arrest of the accused if he found
probable cause, the rule now is that the investigating
14.11 FLORES vs. SUMALJAG (290 SCRA 568) judges power to order the arrest of the accused is
limited to instances in which there is a necessity for
Facts: This is an administrative case against Judge placing him in custody in order not to frustrate the
Antonio C. Sumaljag, Acting Presiding Judge of ends of justice. The arrest of the accused can be
Branch 5, Municipal Trial Court of Baybay, Leyte, for ordered only in the event the prosecutor files the case
gross ignorance of the law in connection with the and the judge of the Regional Trial Court finds
preliminary investigation of three criminal cases and probable cause for the issuance of a warrant of
the arrest of complainants. arrest. It is entirely new rule, and it is plain to see
that it is not obligatory, but merely discretionary,
Complainants Domingo Veloso, et al, were charged for upon the investigating judge to issue a warrant for
conspiring together in preparing a spurious and the arrest of the accused, even after having personally
falsified excerpt from the alleged minutes of the examined the complainant and his witnesses in the
regular session dated August 24, 1996 to unduly form of searching questions and answers, for the
protest the application of another person in the name determination of whether a probable cause exists and
of Gualberto Parmis to own a certain lot in Sitio whether it is necessary to arrest the accused in order
Hayahay in Baybay, Leyte. The charge arises from not to frustrate the ends of justice, is left to his sound
the fact that no regular session was held on the above- judgment or discretion.
mentioned date.
14.12 BACHE & CO. vs RUIZ (37 SCRA 823)
Basically, the complainants instituted this
administrative case because of the four main points: Facts: On 24 Feb 1970, Commissioner Vera of
Internal Revenue, wrote a letter addressed to J Ruiz
1. The testimonies during the preliminary requesting the issuance of a search warrant against
examinations failed to establish probable petitioners for violation of Sec 46(a) of the NIRC, in
cause; relation to all other pertinent provisions thereof,
particularly Sects 53, 72, 73, 208 and 209, and
2. In Criminal Case Nos. R-3227-A and R-3228-
authorizing Revenue Examiner de Leon make and file
A, the complainant was not personally
the application for search warrant which was
examined by respondent, the ones who
attached to the letter. The next day, de Leon and his
testified being only complainants witnesses;
witnesses went to CFI Rizal to obtain the search
3. In Criminal Case No. R 3231-A, there was warrant. At that time J Ruiz was hearing a certain
absence of searching questions and answers case; so, by means of a note, he instructed his Deputy
during the preliminary examination, the Clerk of Court to take the depositions of De Leon and
questions propounded being answerable by Logronio. After the session had adjourned, J Ruiz was
Yes, Judge; informed that the depositions had already been taken.
The stenographer read to him her stenographic notes;
and thereafter, J Ruiz asked respondent Logronio to

Consti II (section 2) 8
take the oath and warned him that if his deposition when the description expresses a conclusion of fact
was found to be false and without legal basis, he could not of law by which the warrant officer may be guided
be charged for perjury. J Ruiz signed de Leon’s in making the search and seizure or when the things
application for search warrant and Logronio’s described are limited to those which bear direct
deposition. The search was subsequently conducted. relation to the offense for which the warrant is being
issued.
ISSUE: Whether or not there had been a valid search
warrant. 14.13 SOLIVEN vs. MAKASIAR (GR 8287, Nov.
14, 1981)
HELD: The SC ruled in favor of Bache on three
grounds. Pres. Cory Aquino filed a criminal complaint for libel
against Beltran. Beltran argues that "the reasons
1. J Ruiz failed to personally examine the which necessitate presidential immunity from suit
complainant and his witness. Personal impose a correlative disability to file suit". He
examination by the judge of the contends that if criminal proceedings ensue by virtue
complainant and his witnesses is of the President's filing of her complaint-affidavit, she
necessary to enable him to determine the may subsequently have to be a witness for the
existence or non-existence of a probable prosecution, bringing her under the trial court's
cause. jurisdiction. This would in an indirect way defeat her
privilege of immunity from suit, as by testifying on
2. The search warrant was issued for more than the witness stand, she would be exposing herself to
one specific offense. The search warrant in possible contempt of court or perjury. Beltran also
question was issued for at least four distinct contends that he could not be held liable for libel
offenses under the Tax Code. As ruled in because of the privileged character of the publication.
Stonehill “Such is the seriousness of the He also says that to allow the libel case to proceed
irregularities committed in connection with would produce a “chilling effect” on press freedom.
the disputed search warrants, that this Court
deemed it fit to amend Section 3 of Rule 122 of ISSUE:Whether or not the constitutional rights of
the former Rules of Court that ‘a search Beltran were violated when respondent RTC judge
warrant shall not issue but upon probable issued a warrant for his arrest without personally
cause in connection with one specific offense.’ examining the complainant and the witnesses, if any,
Not satisfied with this qualification, the Court to determine probable cause
added thereto a paragraph, directing that ‘no
search warrant shall issue for more than one HELD: What the Constitution underscores is the
specific offense. exclusive and personal responsibility of the issuing
judge to satisfy himself of the existence of probable
3. The search warrant does not particularly cause. In satisfying himself of the existence of
describe the things to be seized. The probable cause for the issuance of a warrant of arrest,
documents, papers and effects sought to be the judge is not required to personally examine the
seized are described in the Search Warrant complainant and his witnesses. Following established
“Unregistered and private books of accounts doctrine and procedure, he shall: (1) personally
(ledgers, journals, columnars, receipts and evaluate the report and the supporting documents
disbursements books, customers ledgers); submitted by the fiscal regarding the existence of
receipts for payments received; certificates of probable cause and, on the basis thereof, issue a
stocks and securities; contracts, promissory warrant of arrest; or (2) if on the basis thereof he
notes and deeds of sale; telex and coded finds no probable cause, he may disregard the fiscal's
messages; business communications, report and require the submission of supporting
accounting and business records; checks and affidavits of witnesses to aid him in arriving at a
check stubs; records of bank deposits and conclusion as to the existence of probable cause.
withdrawals; and records of foreign
remittances, covering the years 1966 to 1970.” Sound policy dictates this procedure, otherwise judges
would be unduly laden with the preliminary
The description does not meet the requirement in Art examination and investigation of criminal complaints
III, Sec. 1, of the Constitution, and of Sec. 3, Rule 126 instead of concentrating on hearing and deciding
of the Revised Rules of Court, that the warrant should cases filed before their courts.
particularly describe the things to be seized. A search
warrant may be said to particularly describe the 14.14 LUNA vs. PLAZA (26 SCRA 310)
things to be seized when the description therein is as
specific as the circumstances will ordinarily allow or

Consti II (section 2) 9
Facts: Luna was charged with the crime of murder Additional notes: The term "searching questions and
before the court of respondent-judge Lorenzo Plaza. answers" means only, taking into consideration the
Supporting the complaint were sworn statements of purpose of the preliminary examination which is to
the witnesses for the prosecution, in the form of determine "whether there is a reasonable ground to
questions and answers taken by investigator T-Sgt. believe that an offense has been committed and the
Patosa, and subscribed and sworn to before the accused is probably guilty thereof so that a warrant of
respondent Judge at the time of the filing of the arrest may be issued and the accused held for trial.”
complaint. The respondent Judge examined the What would be searching questions would depend on
prosecution witnesses by reading to them "all over what is sought to be inquired into, such as: the nature
again the questions and answers" in their statements of the offense, the date, time, and place of its
in writing, and the witnesses-affiants declared before commission, the possible motives for its commission;
said Judge that the questions were propounded by T- the subject, his age, education, status, financial and
Sgt. Candido Patosa, and that the answers were made social circumstances, his attitude toward the
by them. Considering the answers of the affiants to investigation, social attitudes, opportunities to
the questions contained in their sworn statements, commit the offense; the victim, his age, status, family
together with the post-mortem and autopsy report on responsibilities, financial and social circumstances,
the dead body of the victim, the respondent Judge characteristics, etc. The points that are the subject of
opined that there was reasonable ground to believe inquiry may differ from case to case. The questions,
that the crime of murder had been committed and the therefore, must to a great degree depend upon the
accused was probably guilty thereof. Respondent Judge making the investigation. At any rate, the court
Judge issued the order and warrant of arrest, a quo found that respondent Judge was "satisfied that
specifying therein that no bail should be accepted for the questions and answers contained in the sworn
the provisional release of the accused. statements taken by T-Sgt. Patosa partake of the
Petitioner contends that Republic Act No. 3828 nature of his searching questions and answers as
imposes on a municipal judge, before he can issue a required by law," so the respondent Judge adopted
warrant of arrest, two specific duties, to wit: (1) he them.
must examine the witnesses personally; (2) the
examination must be under oath; and (3) the
examination must be reduced to writing in the form of 14.15 KHO vs. JUDGE MAKALINTAL (GR 94902-06)
searching questions and answers. The record of the
instant case, according to petitioner, does not show Facts: May 15, 1990, NBI agents Max Salvador and
that said examination was performed by respondent Eduardo Arugay applied for the issuance of search
Judge. warrants by the respondent Judge against petitioner
Kho, in his residences in BF homes and Moonwalk,
ISSUE: Did the respondent judge violate the above- Paranaque. The search warrants were applied after
mentioned requisites for the issuance of a warrant of teams of NBI agents had conducted personal
arrest? surveillances and investigation in the 2 houses on the
basis of confidential information they received that
HELD:The first condition was fulfilled. The trial said places were being used as storage centers for
court found as a fact that "the respondent judge unlicensed firearms and “chop-chop” vehicles. On the
personally examined the witnesses for the same day, respondent Judge conducted the necessary
prosecution; that respondent judge adopted as his own examination of the applicants and their witnesses,
personal examination the questions asked by T-Sgt. after which he issued search warrants. On May 16,
Patosa appearing in the written statements, which he 1990, NBI searched the premises and recovered
read over again the witnesses together with the unlicensed high-powered firearms, ammunitions,
answers given therein, asking the witnesses whether radio transceivers and unregistered motor vehicles.
said answers were theirs, and whether the same On May 28, 1990, petitioners file a Motion to Quash
answers were true, to which the witnesses answered contending that said warrants were issued without
in the affirmative. Republic Act No. 3828 does not probable cause and were in the nature of general
prohibit the municipal Judge from adopting the warrants. On July 26, 1990, respondent Judge denied
questions asked by a previous investigator. The said motion.
second condition for the issuance of a warrant
of arrest was also fulfilled. The trial court found ISSUE:Whether or not there was probable cause in
that the complaint was "supported by statements of the issuance of the search warrants?
the witnesses under oath." The third condition
requiredwas likewise fulfilled. The examination of HELD:Yes. It is within the discretion of the
the witnesses was written down, in the form of examining judge to determine what questions to ask
searching questions and answers. the witnesses so long as the questions asked are
germane to the pivot of inquiry – the existence or
Consti II (section 2) 10
absence of probable cause. The respondent judge true test of sufficiency of an affidavit to warrant
examined the applicants and witnesses under oath, issuance of a search warrant is whether it has
and asked them questions on the facts and been drawn in such a manner that perjury
circumstances personally known to them enough to could be charged thereon and affiant be held
create a probable cause. liable for damages caused. The affidavit, which
served as the exclusive basis of the search warrant, is
On the issue of general warrants, the law does not insufficient and fatally defective by reason of the
require that the things to be seized must be described manner in which the oath was made, and therefore,
in precise and minute detail as to leave no room for the search warrant and the subsequent seizure of the
doubt on the part of the searching authorities. books, documents and other papers are illegal.
Further, the description is required to be specific only Further, it is the practice in this jurisdiction to attach
so far as the circumstance will ordinarily allow. the affidavit of at least the applicant or complainant
to the application. It is admitted that the judge who
14.16 ALVAREZ vs. COURT OF FIRST INSTANCE issued the search warrant in this case, relied
(64 PHIL 33) exclusively upon the affidavit made by agent Almeda
and that he did not require nor take the deposition of
Facts: The Anti-Usury Board of the Department of
any other witness. The Constitution does not provide
Justice presented to Judge David a sworn affidavit
that it is of an imperative necessity to take the
that a certain Narciso Alvarez is in possession of
depositions of the witnesses to be presented by the
books, receipts, chits, lists used by him as money
applicant or complainant in addition to the affidavit of
lender/usurer charging usurious rates in violation of
the latter. The purpose of both in requiring the
law. Affiant Almeda, chief of the task force, didn’t say
presentation of depositions is nothing more than to
that the information was based on his personal
satisfy the committing magistrate of the existence of
knowledge but was only received by him from a
probable cause. Therefore, if the affidavit of the
reliable source. Subsequently, the judge issued the
applicant or complainant is sufficient, the judge may
warrant ordering the search of Alvarez’ house. On
dispense with that of other witnesses. Inasmuch as
June 4, 1936, the agents raided the subject place and
the affidavit of the agent was insufficient because his
seized different documents namely, banknotes,
knowledge of the facts was not personal but merely
bankbooks, stubs, cashbooks, bills of lading, credit
hearsay, it is the duty of the judge to require the
receipts, etc. Thereafter, the articles seized was not
affidavit of one or more witnesses for the purpose of
brought immediately to the custody of the judge who
determining the existence of probable cause to
issued the SW. Alvarez moved that the agents of the
warrant the issuance of the search warrant. When the
Board be declared guilty of contempt and prays that
affidavit of the applicant or complainant contains
all articles in question be returned to him because the
sufficient facts within his personal and direct
SW issued was illegal. On the other hand, the Anti-
knowledge, it is sufficient if the judge is satisfied that
Usury Board pleaded that they be allowed to retain
there exists probable cause; when the applicant’s
custody of the articles seized for further investigation.
knowledge of the facts is mere hearsay, the affidavit
When the judge sustained the latter’s motion. Alvarez
of one or more witnesses having a personal knowledge
elevated the matter to the SC and prayed that the
of the facts is necessary. Thus the warrant issued is
search warrant as well as the order of the judge
likewise illegal because it was based only on the
authorizing the Anti-Usury Board to retain custody be
affidavit of the agent who had no personal knowledge
declared null and void.
of the facts.
ISSUE: Whether the search warrant issued by the
judge is illegal for having solely as basis the affidavit
of Agent Almeda in whose oath the latter declared 14.17 BACHE & CO. vs. RUIZ (37 SCRA 823)
that he had no personal knowledge of the facts which
were to serve as basis for the issuance of the warrant Facts:Commissioner of Internal Revenue, wrote a
but he had knowledge thereof only through letter addressed to respondent Judge Vivencio M.
information secured from a person whom he Ruiz requesting the issuance of a search warrant
considered reliable. against petitioners for violation of Section 46(a) of the
National Internal Revenue Code. Revenue Examiner
HELD:Yes.The oath required must refer to the Rodolfo de Leon and Arturo Logronio went to CFI
truth of the facts within the personal knowledge with proper documents. Judge Vivencio Ruiz asked
of the petitioner or his witnesses, because the his secretary to take the deposition and when done
purpose thereof is to convince the committing stenographer read it to the judge. Logronio took the
magistrate, not the individual making the oath and was warned by judge that he may be
affidavit and seeking the issuance of the charged with perjury if found lying. Search warrant
warrant, of the existence of probable cause. The was issued and served. Petitioners’ lawyers protested
Consti II (section 2) 11
the search on the ground that no formal complaint or bail constitutes a waiver of any irregularity in the
transcript of testimony was attached to the warrant. issuance of a warrant of arrest.
The agents nevertheless proceeded with their search
which yielded six boxes of documents. BIR based on ISSUES:
the documents seized. Petitioner contend that judged
failed to personally examine the complainant and 1. WON the Informations charging the
witnesses. petitioners were validly filed and the warrants
for their arrest were properly issued;
ISSUE:Whether or not search warrant is null and
void on the ground of no personal examination of the 2. WON this Court can, itself, determine
judge? probable cause; and

3. WON the petitioners posting a bail constitutes


a waiver of their right to question the validity
RULING:Yes. If there was an examination at all of of their arrest.
the complainant and his witness, it was the one
conducted by the Deputy Clerk of Court. But, as HELD:First issue: For the issuance of a warrant of
stated, the Constitution and the rules require a arrest, probable cause has been defined as the
personal examination by the judge. It was precisely on existence of such facts and circumstances that would
account of the intention of the delegates to the lead a reasonably discreet and prudent person to
Constitutional Convention to make it a duty of the believe that an offense has been committed by the
issuing judge to personally examine the complainant person sought to be arrested. It is one of the requisites
and his witnesses that the question of how much time for a warrant of arrest to be valid. On the basis of the
would be consumed by the judge in examining them above-stated documents (in the facts) and on the
came up before the Convention, as can be seen from strength of the affidavit executed by the respondent,
the record of the proceedings quoted above. The the prosecutor concluded that probable cause exists.
reading of the stenographic notes to respondent Judge These same affidavit and documents were used by the
did not constitute sufficient compliance with the trial court in issuing the warrant of arrest. The SC
constitutional mandate and the rule; for by that finds the complaint-affidavit and attachments
manner respondent Judge did not have the insufficient to support the existence of probable cause.
opportunity to observe the demeanor of the The respondent’s claims of the falsity of the
complainant and his witness, and to propound initial documents were mere assertions. It must be
and follow-up questions which the judicial mind, on emphasized that the affidavit of the complainant, or
account of its training, was in the best position to any of his witnesses, shall allege facts within their
conceive. These were important in arriving at a sound (affiants) personal knowledge. The allegation of the
inference on the all-important question of whether or respondent that the signatures were falsified does not
not there was probable cause. qualify as personal knowledge. Nowhere in said
affidavit did respondent state that he was present at
14.18 BORLONGAN vs. PENA (GR 143591, Nov. 23, the time of the execution of the documents. Neither
2007) did he claim that he was familiar with the signatures
of the signatories. He simply made a bare assertion
Facts: Respondent Pena instituted a civil case for
recovery of agent’s compensation and expenses, Second issue:Regarding the issuance of the warrant of
damages and attorney’s fees against Urban Bank and arrest, petitioners contend that the warrants were
petitioners before the RTC. Petitioners filed a Motion illegally issued as they were solely based on the
to dismiss, including several documents as evidence. affidavits of the complainant. Section 2 of Article III
Atty Pena claims that the documents were falsified. of the Constitution underscores the exclusive and
He subsequently filed his Complaint-Affidavit with personal responsibility of the issuing judge to satisfy
the City Prosecutor. The prosecutor found probable himself of the existence of probable cause. But the
cause and the Informations were filed before MTCC. judge is not required to personally examine the
Warrants of arrest were issued for the petitioners / complainant and his witnesses. Following established
accused. Upon the issuance of the warrant of arrest, doctrine and procedure, he shall (1) personally
petitioners immediately posted bail as they wanted to evaluate the report and the supporting documents
avoid embarrassment, being then officers of Urban submitted by the prosecutor regarding the existence
Ban. On the scheduled date for the arraignment, of probable cause, and on the basis thereof, he may
despite the petitioners’ refusal to enter a plea, the already make a personal determination of the
court a quo entered a plea of “Not Guilty” for them. existence of probable cause; and (2) if he is not
The accused questioned the validity of the warrant of satisfied that probable cause exists, he may disregard
arrest. However, the trial court ruled that posting of the prosecutor’s report and require the submission of

Consti II (section 2) 12
supporting affidavits of witnesses to aid him in RULING:Yes. In determining probable cause, it is
arriving at a conclusion as to the existence of probable required that (1) the judge must examine the
cause. There is no provision or procedural rule which complaint and his witnesses personally; (2) that the
makes the submission of counter-affidavits mandatory examination must be under oath; and (3) the
before the judge could determine probable cause. examination must be reduced in writing in the form of
searching questions and answers. The prosecution
Third issue:Yes. The erstwhile ruling of this Court failed to prove that the judge who issued the warrant
was that posting of bail constitutes a waiver of any put into writing his examination of the applicant, and
irregularity in the issuance of a warrant of arrest, his witnesses in the form of searching questions and
that has already been superseded by Section 26, Rule answers before issuance of the search warrant. When
114 of the Revised Rule of Criminal Procedure. The the Branch Clerk of Court was required to testify on
principle that the accused is precluded from the available records kept in their office, neither
questioning the legality of the arrest after transcript of the proceedings of a searching question
arraignment is true only if he voluntarily enters his and answer nor the sworn statements of the
plea and participates during trial, without previously complainant and his witnesses showing that the judge
invoking his objections thereto. examined them in writing was presented. Mere
affidavits of the complainant and his witnesses are
Moreover, considering the conduct of the petitioner not sufficient. Such written examination is necessary
after posting her personal bail bond, it cannot be in order that the judge may be able to properly
argued that she waived her right to question the determine the existence and non-existence of probable
finding of probable cause and to assail the warrant of cause.
arrest issued against her by the respondent judge.
There must be clear and convincing proof that the 14.20ORTIZ vs. PALAYPAYON (234 SCRA 391)
petitioner had an actual intention to relinquish her
right to question the existence of probable cause. Facts: Judge Lucio Palaypayon of the Municipal Trial
When the only proof of intention rests on what a party Court of Tinambac, Camarines Sur is
does, his act should be so manifestly consistent with, administratively charged with gross ignorance of the
and indicative of, an intent to voluntarily and law tainted with vindictiveness and oppression
unequivocally relinquish the particular right that no relative to a criminal case pending before respondent
other explanation of his conduct is possible. x x x. judge. The charge stemmed from a complaint for
damage to property thru reckless imprudence filed
14.19 PEOPLE vs. MAMARIL (GR 147607) before the respondent's sala on May 19, 1993
involving a collision between a Toyota Corolla owned
Facts: SPO2 Chito Esmenda applied before the RTC by one Rosalinda Tanay and then driven by her
for a search warrant authorizing the search for husband, Roberto Tanay, and a mini-truck owned by
marijuana at the family residence of appellant one Juliana Lu which was entrusted to herein
Mamaril. During the search operation, the searching complainant, and then driven by one Rodrigo
team confiscated sachets of suspected marijuana Vasquez. It appears that on May 19, 1993, respondent
leaves. Police officers took pictures of the confiscated judge issued an order for the arrest of accused Juliana
items and prepared a receipt of property seized and Lu, Rodrigo Vasquez and herein complainant, David
certified that the house was properly searched, which Ortiz, on the basis of mere affidavits by the offended
was signed by the appellant and the barangay officials party and without conducting the preliminary
who witnessed the search. The PNP Crime Laboratory investigation required by Rule 112 of the Revised
issued a report finding the seized specimens positive Rules of Court
for the presence of marijuana. Moreover, the
examination on the urine sample of appellant Respondent judge, instead of filing his comment on
affirmed that it was positive for the same. Appellant the complaint, filed a Motion to Dismiss alleging lack
denied that he was residing at his parent’s house, and of factual or legal basis. claims that pursuant to the
that he was at his parent’s house when the search said rule, he personally examined in writing and
was conducted only because he visited his mother. He under oath the private complainant and his witnesses
also said that he saw the Receipt of Property Seized by asking the same questions propounded to them in
for the first time during the trial, although he their sworn statements and the same answers were
admitted that the signature on the certification that given. At the same time, respondent judge maintains
the house was properly searched was his. that it is customary for him to adopt the sworn
statements of the complainant and the witnesses if he
ISSUE:Whether or not the trial court erred in issuing is satisfied of the existence of probable cause. This
a search warrant. practice, he claims, will expedite the proceedings and
save the party litigants' time and money.

Consti II (section 2) 13
ISSUE:Whether or not Palaypon erred in ordering the organization known as the Parliamentary Club. Jose
arrest of the accused Juliana Lu, et al.? Ma. Veloso was at that time a member of the House of
Representative of the Philippine Legislature. He was
HELD:The respondent judge's action clearly violates also the manager of the club. The police of Manila had
constitutional provisions and established rules of reliable information that the so-called Parliamentary
procedure. Article III, Section 2 of the 1987 Club was nothing more than a gambling house.
Constitution provides: “…no search warrant or Indeed, on May 19, 1923, J. F. Townsend, the chief of
warrant of arrest shall issue except upon probable the gambling squad, had been to the club and verified
cause to be determined personally by the judge after this fact. As a result, on May 25, 1923, Detective
examination under oath or affirmation of the Andres Geronimo of the secret service of the City of
complainant and the witnesses he may produce...” In Manila, applied for, and obtained a search warrant
relation thereto, Rule 112, Section 6 (b) of the 1985 from Judge Garduño of the municipal court.
Rules on Criminal Procedure, as amended, provides
that “If the municipal trial judge conducting the This is an appeal from a judgment of the
preliminary investigation is satisfied after an Court of First Instance of Manila finding the accused,
examination in writing and under oath of the Jose Ma. Veloso, guilty of the crime of resistance of
complainant and his witnesses in the form of the agents of the authority, in violation of article 252
searching questions and answers, that a probable of the Penal CodeThe errors assigned by counsel for
cause exists and that there is a necessity of placing the accused as appellant, go to the proposition that
the respondent under immediate custody in order not the resistance of the police was justifiable on account
to frustrate the ends of justice, he shall issue a of the illegality of the John Doe search warrant.
warrant of arrest.”
I:Whether there was a valid SW?
From the foregoing factual and legal milieu, it is
evident that there was no preliminary investigation R:YES. John Doe warrants are valid if the best
conducted. Respondent judge did not personally description possible is given in the arrest warrant. It
examine the complainant and her witnesses by asking must be sufficient to indicate clearly on whom it is to
searching questions and answers to satisfy himself of be served, by stating his occupation, personal
the existence of probable cause as mandated by law. appearance or peculiarities, place of residence or other
He simply ignored the constitutional requirement of circumstances which he may be identified.
procedural due process. This Court cannot
countenance such blatant practice of disregarding The Court agree with the trial judge and with
fairly elementary legal principles and substituting it the Attorney-General in their conclusions to the effect
with an unorthodox and highly irregular practice that the search warrant was valid, and that the
which appears convenient only to respondent judge. defendant has been proved guilty beyond a reasonable
doubt, of the crime of resistance of the agents of the
The purpose of a preliminary investigation is to authority. Undoubtedly, Jose Ma. Veloso, as Juan de
secure the innocent against hasty, malicious and la Cruz, would have resisted the police just as stoutly,
oppressive prosecution, and to protect him from an as the Honorable Jose Ma. Veloso did. The penalty,
open and public accusation of crime, from the trouble, accordingly, falls within that provided by the Penal
expense and anxiety of a public trial, and also to Code.
protect the state from useless and expensive trials.
(Trocio v. Manta, 118 SCRA 241; citing Hashim v. EMPHASIS ON MAIN POINT SUPPLIED
Boncan, 71 Phil. 216). The right to a preliminary
15.12 Alvarez versus CFI
investigation is a statutory grant, and to withhold it
would be to transgress constitutional due process. F: The Anti-Usury Board of the Department of
(See People v. Oandasan, 25 SCRA 277). However, in Justice presented to Judge David a sworn affidavit
order to satisfy the due process clause it is not enough that a certain Narciso Alvarez is in possession of
that the preliminary investigation is conducted in the books, receipts, chits, lists used by him as money
sense of making sure that a transgressor shall not lender/usurer charging usurious rates in violation of
escape with impunity. A preliminary investigation law. Affiant Almeda, chief of the task force, didn’t say
serves not only the purposes of the State. that the information was based on his personal
knowledge but was only received by him from a
D. Particularity of Description
reliable source. Subsequently, the judge issued the
15.11 People versus Veloso warrant ordering the search of Alvarez’ house. On
June, the agents raided the subject place and seized
F:In May, 1923, the building located at No. 124 Calle different documents namely, banknotes, bankbooks,
Arzobispo, City of Manila, was used by an stubs, cashbooks, bills of lading, credit receipts, etc.

Consti II (section 2) 14
Thereafter, the articles seized was not brought F: Respondent Judge issued a search warrant for
immediately to the custody of the judge who issued the seizure of articles allegedly used by petitioner in
the Search Warrant (SW). Alvarez moved that the committing the crime of sedition. Seized were printed
agents of the Board be declared guilty of contempt copies of the Philippine Times, newspaper dummies,
and prays that all articles in question be returned to typewriters, mimeographing machines and tape
him because the SW issued was illegal. On the other recorders, video machines and tapes. The petitioner
hand, the Anti-Usury Board pleaded that they be moved to quash the warrant but his motion was
allowed to retain custody of the articles seized for denied.
further investigation. When the judge sustained the
latter’s motion. Alvarez elevated the matter to the SC I: Whether the SW was valid?
and prayed that the search warrant as well as the
order of the judge authorizing the Anti-Usury Board R: The statements made in the affidavits are
to retain custody be declared null and void. mere conclusions of law and do not satisfy the
requirement of probable cause. The language used is
Issue: Whether the SW issued by the judge is illegal all embracing as to include all conceivable words and
for having solely as basis the affidavit of Agent equipment of petitioner regardless of whether they
Almeda in whose oath the latter declared that he had are legal or illegal. The search warrant under
no personal knowledge of the facts which were to consideration was in the nature of a general warrant
serve as basis for the issuance of the warrant but he which is objectionable. Conclusions of law of
had knowledge thereof only through information military officers will not satisfy probable cause
secured from a person whom he considered reliable. requirement for issuance of search warrants.

EMPHASIS ON MAIN POINT SUPPLIED

Ruling: The Constitution requires that there be not 15.14 Pangandaman versus Casar
only probable cause before the issuance of a search
warrant but that the search warrant must be based F: The shooting incident by armed men in Lanao
upon an application supported by oath of the led to the issuance of a warrant of arrest. Petitioners
applicant and the witnesses he may produce. In its assert that the respondent Judge issued a warrant of
broadest sense, an oath includes any form of arrest against fifty “John Does” transgressing the
attestation by which a party signifies that he is bound Constitutional provision requiring that such warrants
in conscience to perform an act faithfully and should particularly describe the persons or things to
truthfully; and it is sometimes defined as an outward be seized.
pledge given by the person taking it that his
I: Whether said warrant is valid?
attestation or promise is made under an immediate
sense of his responsibility to God. The oath required R: NO. Insofar as said warrant is issued against
must refer to the truth of the facts within the personal fifty “John Does” not one of whom the witnesses to the
knowledge of the petitioner or his witnesses, because complaint could or would identify. Clearly such was
the purpose thereof is to convince the committing violative of the constitutional injunction that
magistrate, not the individual making the affidavit warrants of arrest should particularly describe
and seeking the issuance of the warrant, of the the person or persons to be seized, the warrant
existence of probable cause. The true test of must, as regards its unidentified subjects, be
sufficiency of an affidavit to warrant issuance voided.
of a search warrant is whether it has been
drawn in such a manner that perjury could be Wherefore, the warrant complained of is
charged thereon and affiant be held liable for upheld and declared valid insofar as it orders the
damages caused. The affidavit, which served as the arrest of the petitioners. Said warrant is voided to the
exclusive basis of the search warrant, is insufficient extent that it is issued against fifty “John Does.” The
and fatally defective by reason of the manner in which respondent Judge is directed to forward to the
the oath was made, and therefore, the search warrant Provincial Fiscal of Lanao del Sur the record of the
and the subsequent seizure of the books, documents preliminary investigation of the complaint of his court
and other papers are illegal. for further appropriate action.

EMPHASIS ON MAIN POINT SUPPLIED EMPHASIS ON MAIN POINT SUPPLIED

Particularity of Decision 15.15 Stonehill versus Diokno

15.13 Corro versus Lising F: Respondents issued, on different dates, 42


search warrants against petitioners personally, and

Consti II (section 2) 15
corporations for which they are officers directing F: September 2, 2006 at around 1245 PM, PO1
peace officers to search the persons of petitioners and Bernard Azarden was on duty at the Police
premises of their offices, warehouses and residences Community Precinct along Arellano St., Dagupan City
to search for personal properties “books of accounts, when a concerned citizen reported that a pot session
financial records, vouchers, correspondence, receipts, was underway in the house of accused Rafael
ledgers, journals, portfolios, credit journals, Gonzales in Trinidad Subdivision, Dagupan City. PO1
typewriters, and other documents showing all Azardan, PO1 Alejandro dela Cruz and members of
business transactions including disbursement Special Weapons and Tactics (SWAT) proceeded to
receipts, balance sheets and profit and loss aforesaid house. Upon inquiry from people in the
statements and Bobbins (cigarettes)” as the subject of area, the house of Gonzales was located. As the team
the offense for violations of Central Bank Act, Tariff entered the house, accused Orlando Doria was
and Customs Laws, Internal Revenue Code, and arrested while coming out. Inside the house were
Revised Penal Code. Gonzales, Arnold Martinez, Edgar Dizon, and Rezin
Martinez. Seized from the accused were open plastic
Upon effecting the search in the offices of the sachets (containing shabu residue), pieces of rolled
aforementioned corporations and on the respective used aluminum foil and pieces of used aluminum foil.
residences of the petitioners, there seized documents, The accused were arrested and brought to police
papers, money and other records. Petitioners then station, seized items were sent to the Pangasinan
were subjected to deportation proceedings and were Provincial Police Crime Laboratory. All accused,
constrained to question the legality of the searches except for Doria, were found positive for methyl
and seizures as well as the admissibility of those amphetamine HCL. On February 13, 2008, RTC found
seized as evidence against them. Arnold Martinez, Edgar Dizon, Rezin Martinez and
Rafael Gonzales guilty beyond reasonable doubt
On March 1962, the SC issued a writ of under Sec. 13 in relation to Sec. 11, Art. II of RA
preliminary injunction and partially lifted the same 9165and sentenced each to life imprisonment and
on June with respect to some documents and papers. fined PHP 500,000 plus cost of suit. The CA supported
the findings of the lower court.(IN SHORT THE
I: Whether the SW were vaid?
ACCUSED HEREIN ARE CAUGHT IN A POT
R: NO. Search warrants issued were violative of SESSION INFLAGRANTE DELICTO)
the Constitution and the Rules, thus, illegal or
I: Whether the accused is estopped from
being general warrants. There is no probable
assailing the legality of his arrest.
cause and warrant did not particularly specify
the things to be seized. The purpose of the R: YES. Section 2 as a constitutional guarantee,
requirement is to avoid placing the sanctity of the however, is not a blanket prohibition against all
domicile and the privacy of communication and searches and seizures without warrant. Arrests and
correspondence at the mercy of the whims, caprice or seizures in the following instances are allowed even in
passion of peace officers. the absence of a warrant (i) warrantless search
incidental to a lawful arrest; (ii) search of evidence in
Document seized from an illegal search
"plain view;" (iii) search of a moving vehicle; (iv)
warrant is not admissible in court as a fruit of a
consented warrantless search; (v) customs search; (vi)
poisonous tee. However, they could not be returned,
stop and frisk; and (vii) exigent and emergency
except if warranted by the circumstances.
circumstances.
Petitioners were not the proper party to question the
This case would appear to fall under either a
validity and return of those taken from the
warrantless search incidental to a lawful arrest or a
corporations for which they acted as officers as they
plain view search, both of which require a lawful
are treated as personality different from that of the
arrest in order to be considered valid exceptions to the
corporation.
constitutional guarantee.

EMPHASIS ON MAIN POINT SUPPLIED

15.17 Microsoft Corp versus Maxicorp


EMPHASIS ON MAIN POINT SUPPLIED
F: In 1996, Dominador Samiano, Jr., an agent of
the National Bureau of Investigation conducted a
surveillance against Maxicorp, Inc. He observed that
15.16 People versus Martinez Microsoft Softwares were being produced and
packaged within the premises of Maxicorp. Samiano,

Consti II (section 2) 16
together with a civilian witness (John Benedict F: Two warrants were issued against petitioners
Sacriz) then bought a computer unit from Maxicorp. for the search on the premises of “Metropolitan Mail”
The unit was pre-installed with a pirated copy of and “We Forum” newspapers and the seizure of items
Windows. For their purchase, they were issued a alleged to have been used in subversive activities.
receipt, however, the receipt was in the name of a Petitioners prayed that a writ of preliminary
certain “Joel Diaz”. mandatory and prohibitory injunction be issued for
the return of the seized articles, and that respondents
Subsequently, Samiano applied for a search be enjoined from using the articles thus seized as
warrant before the RTC. He brought with him Sacriz evidence against petitioner.
as witness. He also brought the computer unit they
bought as evidence as well as the receipt. He even Petitioners questioned the warrants for the
added an additional witness (Felixberto Pante), a lack of probable cause and that the two warrants
computer technician, who showed the judge that the issued indicated only one and the same address. In
software in the computer unit bought by Samiano addition, the items seized subject to the warrant were
from Maxicorp was pirated. real properties.

The RTC judge, convinced that there is a Issue: Whether the two warrants were valid to
probable cause for a case of copyright infringement justify seizure of the items.
and unfair competition committed by Maxicorp,
issued the corresponding warrant. Maxicorp assailed R: NO.What makes the search warrants
the legality of the warrant before the Court of under consideration constitutionally
Appeals. The Court of Appeals ruled in favor of objectionable is that they are in the nature of
Maxicorp and in its decision it highlighted the fact general warrants. Mere generalization will not
that the receipt issued was not in Samiano’s or Sacriz’ suffice. Thus, the broad statement in Col. Abadilla's
name hence the proceeding in the trial court was application that petitioner "is in possession or has in
infirm from the onset. his control printing equipment and other
paraphernalia, news publications and other
I: Whether or not the Court of Appeals is correct documents which were used and are all continuously
in ruling in favor of Maxicorp.? being used as a means of committing the offense of
subversion punishable under Presidential Decree 885,
R: NO. The testimonies of the two witnesses, as amended ..." is a mere conclusion of law and does
coupled with the object and documentary evidence not satisfy the requirements of probable cause.
they presented, are sufficient to establish the
existence of probable cause. From what they have The Search Warrants issued by respondent
witnessed, there is reason to believe that Maxicorp judge are hereby declared null and void and are
engaged in copyright infringement and unfair accordingly set aside. The prayer for a writ of
competition to the prejudice of Microsoft. Both NBI mandatory injunction for the return of the seized
Agent Samiano and Sacriz were clear and insistent articles is hereby granted and all articles seized
that the counterfeit software were not only displayed thereunder are hereby ordered released to petitioners.
and sold within Maxicorp’s premises, they were also
produced, packaged and in some cases, installed 15.19EMPHASIS ON MAIN POINT SUPPLIED
there.
Frank Uy versus BIR
The fact that the receipt issued was not in
Samiano’s name nor was it in Sacriz’ name does F: In Sept 1993, Rodrigo Abos, a former
not render the issuance of the warrant void. No employee of UPC reported to the BIR that Uy Chin Ho
law or rule states that probable cause requires a aka Frank Uy, manager of UPC, was selling
specific kind of evidence. No formula or fixed thousands of cartons of canned cartons without
rule for its determination exists. Probable cause issuing a report. This is a violation of Sec 253 & 263 of
is determined in the light of conditions the Internal Revenue Code. In Oct 1993, the BIR
obtaining in a given situation. Thus, it was requested before RTC Cebu to issue a search warrant.
improper for the Court of Appeals to reverse the Judge Gozo-Dadole issued a warrant on the same day.
RTC’s findings simply because the sales receipt A second warrant was issued which contains the same
evidencing NBI Agent Samiano’s purchase of substance but has only one page, the same was dated
counterfeit goods is not in his name. Oct 1st 2003. These warrants were issued for the
alleged violation by Uy of Sec 253. A third warrant
EMPHASIS ON MAIN POINT SUPPLIED was issued on the same day for the alleged violation of
Uy of Sec 238 in relation to sec 263. On the strength
15.18 Burgos versus Chief of Staff, AFP of these warrants, agents of the BIR, accompanied by

Consti II (section 2) 17
members of the PNP, on 2 Oct 1993, searched the On April, the police searched Apartment No.
premises of the UPC. They seized, among other 8, in the same compound and found one (1) .45 caliber
things, the records and documents of UPC. A return of pistol. Found in Apartment No. 2 were firearms,
said search was duly made by Labaria with the RTC ammunitions and explosives. Petitioners were
of Cebu. UPC filed a motion to quash the warrants charged before the RTC of Caloocan City accusing
which was denied by the RTC. They appealed before them with illegal possession of firearms, ammunitions
the CA via certiorari. The CA dismissed the appeal for and explosives, pursuant to Presidential Decree No.
a certiorari is not the proper remedy. 1866.6 Thereafter, petitioners were arrested and
detained.
I: Whether there was a valid search warrant
issued? Petitioners contend that the search and
seizure orders violated Sections 2 and 3 of the Bill of
R: NO. The SC ruled in favor of UPC and Uy in a Rights as well as Section 3 of Rule 126 of the Rules of
way for it ordered the return of the seized items but Court on Criminal Procedure because the place
sustained the validity of the warrant. The SC ruled searched and articles seized were not described with
that the search warrant issued has not met some particularity. They argue that the two-witness
basic requisites of validity. A search warrant must requirement under Section 10 of Rule 126 was ignored
conform strictly to the requirements of the foregoing when only one witness signed the receipt for the
constitutional and statutory provisions. These properties seized during the search, and said witness
requirements, in outline form, are was not presented at the trial.

(1) the warrant must be issued upon probable I: Whether the items described in the warrant
cause; were sufficiently described with particularity.

(2) the probable cause must be determined by R: NO. The place to be searched cannot be
the judge himself and not by the applicant or changed, enlarged nor amplified by the police.
any other person; Policemen may not be restrained from pursuing their
task with vigor, but in doing so, care must be taken
(3) in the determination of probable cause, the that constitutional and legal safeguards are not
judge must examine, under oath or affirmation, disregarded. Exclusion of unlawfully seized evidence
the complainant and such witnesses as the is the only practical means of enforcing the
latter may produce; and constitutional injunction against unreasonable
searches and seizures. Hence, we are constrained to
(4) the warrant issued must particularly
declare that the search made at Apartment No. 8 is
describe the place to be searched and persons
illegal and the .45 caliber pistol taken thereat is
or things to be seized.
inadmissible in evidence against petitioners.
The SC noted that there has been
Now, in contrast, the search conducted at
inconsistencies in the description of the place to be
Apartment No. 2 could not be similarly faulted. The
searched as indicated in the said warrants. Also the
search warrants in question specifically mentioned
thing to be seized was not clearly defined by the
Apartment No. 2. The search was done in the
judge. He used generic itineraries. The warrants were
presence of its occupants, herein petitioners, in
also inconsistent as to who should be searched. One
accordance with Section 7 of Rule 126, Revised Rules
warrant was directed only against Uy and the other
of Court. Petitioners allege lack of particularity in the
was against Uy and UPC. The SC however noted that
description of objects to be seized pursuant to the
the inconsistencies were cured by the issuance of the
warrants. That the articles seized during the search of
latter warrant as it has revoked the two others.
Apartment No. 2 are of the same kind and nature as
EMPHASIS ON MAIN POINT SUPPLIED those items enumerated in the search warrant
appears to be beyond cavil. The items seized from
15.20 Yousex Al Ghoul versus CA Apartment No. 2 were described with specifity in the
warrants in question.

The nature of the items ordered to be seized


F: Judge Geronimo S. Mangay, presiding judge of did not require a technical description. Moreover, the
the RTC, Caloocan City, issued 2 search warrants for law does not require that the things to be seized must
the search and seizure of certain items in Apartment be described in precise and minute details as to leave
No. 2 at 154 Obiniana Compound, Deparo Road, no room for doubt on the part of the searching
Caloocan City. authorities, otherwise, it would be virtually
impossible for the applicants to obtain a search

Consti II (section 2) 18
warrant as they would not know exactly what kind of sufficiently established and it was of the nature of a
things they are looking for. Substantial similarity of general search warrant and on the additional ground
those articles described as a class or species would that the warrant was unlawfully served or
suffice. implemented.

EMPHASIS ON MAIN POINT SUPPLIED ISSUE: Whether the warrant is valid

16.11 People v CA RULING: NO. In the present case, the search


warrant is invalid because (1) the trial court failed to
FACTS; A petition for certiorari has been filed to examine personally the complainant and the other
invalidate the order of Judge Casanova which deponents; (2) SPO3 Cicero Bacolod, who appeared
quashed search warrant issued by Judge Bacalla and during the hearing for the issuance of the search
declared inadmissible for any purpose the items warrant, had no personal knowledge that petitioners
seized under the warrant. An application for a search were not licensed to possess the subject firearms; and
warrant was made by S/Insp Brillantes against Mr. (3) the place to be searched was not described with
Azfar Hussain who had allegedly in his possession particularity. Search Warrant No. 799 is declared null
firearms and explosives at Abigail Variety Store, Apt and void.
1207 Area F. Bagon Buhay Avenue, Sarang Palay,
San Jose Del Monte, Bulacan. The following day 16.13 Malalaon v CA
Search Warrant No. 1068 was issued but was served
not at Abigail Variety Store but at Apt. No. 1, FACTS: Filed with the Regional Trial Court of
immediately adjacent to Abigail Variety Store Kalookan City an application for search warrant in
resulting in the arrest of 4 Pakistani nationals and connection with an alleged violation of P.D. 1866
the seizure of a number of different explosives and (Illegal Possession of Firearms and Ammunitions)
firearms. perpetrated at Quezon City. Respondent RTC Judge
of Kalookan City issued Search Warrant No. 95-90,
ISSUE: Whether the search warrant is valid the same was implemented and resulted to the
indictment of herein petitioners. Petitioners raise the
RULING: No. What was done was to substitute for issue of WHETHER A COURT MAY TAKE
the place that the Judge had written down in the COGNIZANCE OF AN APPLICATION FOR A
warrant and the premises that the executing officers SEARCH WARRANT IN CONNECTION WITH AN
had in their mind. This should not have been done. It OFFENSE ALLEGEDLY COMMITTED OUTSIDE
is neither fair nor licit to allow police officers to search ITS TERRITORIAL JURISDICTION AND TO ISSUE
a place different from that stated in the warrant on A WARRANT TO CONDUCT A SEARCH ON A
the claim that the place actually searched — although PLACE LIKEWISE OUTSIDE ITS TERRITORIAL
not that specified in the warrant — is exactly what JURISDICTION.
they had in view when they applied for the warrant
and had demarcated in their supporting ISSUE: Whether a branch of a RTC has the authority
evidence. What is material in determining the validity to issue a warrant for the search of a place outside its
of a search is the place stated in the warrant itself, territorial jurisdiction
not what the applicants had in their thoughts, or had
represented in the proofs they submitted to the court RULING: YES. A search warrant is but a judicial
issuing the warrant. The place to be searched, as set process, not a criminal action. No legal provision,
out in the warrant, cannot be amplified or modified by statutory or reglamentary, expressly or impliedly
the officers' own personal knowledge of the premises, provides a jurisdictional or territorial limit on its area
or the evidence they adduced in support of their of enforceability. The Interim or Transitional Rules
application for the warrant. Such a change is and Guidelines expressly authorizes its enforcement
proscribed by the Constitution which requires inter anywhere in the country. Contrary to what petitioners
alia the search warrant to particularly describe the aver, there was no violation of their constitutional
place to be searched as well as the persons or things right which is translated into specifically enumerated
to be seized safeguards in Rule 126 of the 1985 Rules on Criminal
Procedure for the issuance of a search warrant, and
16.12 Paper Industries v Asuncion all these have to be observed regardless of whatever
court in whichever region is importuned for or
FACTS: Petitioners pray, inter alia for the actually issues a search warrant.
nullification of Search Warrant No. 799. Petitioners
submit that Judge Asuncion committed grave abuse of 16.14 People v Estrada
discretion or has exceeded his jurisdiction in refusing
to quash such warrant as probable cause has not been FACTS: Atty. Cabanlas, Chief of the Legal,
Information and Compliance Division (LICD) of the
Consti II (section 2) 19
Bureau of Food and Drugs (BFAD), filed with the RTC ISSUE: Whether the search order was valid?
of Quezon City, an application for the issuance of a
search warrant against "Aiden Lanuza of Cebu City," RULING: No. Under the new Constitution, “. . . no
for violation of The Consumer Act of the Philippines search warrant or warrant of arrest shall issue except
(selling medicines without appropriate license from upon probable cause to be determined personally by
DOH). Judge Estrada issued Search Warrant No. 958 the judge after examination under oath or affirmation
(95). Private respondent Aiden Lanuza filed a motion of the complainant and the witnesses he may produce,
to quash the warrant which the respondent Judge and particularly describing the place to be searched
granted. and the persons or things to be seized”. The Closure
and Seizure Order was based on Article 38 of the
ISSUE: Whether the quashal of the warrant was Labor Code. The Supreme Court held, “We reiterate
valid that the Secretary of Labor, not being a judge, may no
longer issue search or arrest warrants. Hence, the
RULING: YES. The respondent Judge acted correctly authorities must go through the judicial process.
in granting the motion to quash the search warrant. Furthermore, the search and seizure order was in the
The search warrant merely indicated the address of nature of a general warrant. The court held that the
the compound which is 516 San Jose de la Montana warrant is null and void, because it must identify
St., Mabolo, Cebu City. This description of the place to specifically the things to be seized.
be searched is too general and does not pinpoint the
specific house of private respondent. Thus, the
inadequacy of the description of the residence of
private respondent sought to be searched has 16.16 Republic (PCGG) v Sandiganbayan
characterized the questioned search warrant as a
general warrant, which is violative of the FACTS: The PCGG, established to recover
constitutional requirement. the alleged ill-gotten wealth of former
President Ferdinand Marcos, his family and
4. Only a Judge May Issue Warrant his cronies, filed with the Sandiganbayan a
complaint for reversion, reconveyance,
16.15 Salazar v Achcoso restitution, accounting and damages and
sequestration against Respondents Tan
FACTS: Rosalie Tesoro in a sworn statement filed (among others). The PCGG filed a motion to
with the POEA, charged petitioner with illegal disqualify Respondent Mendoza as counsel of
recruitment. Public respondent Atty. Marquez Respondents Tan on the basis that Mendoza
directed petitioner to appear to the POEA regarding was then the SolGen and counsel to Central
the complaint against him. After knowing that Bank who actively interfered in the
petitioner had no license to operate a recruitment liquidation of GENBANK which was acquired
agency, respondent Administrator Achacoso issued a by Respondents Tan. Sandiganbayan issued a
Closure and Seizure Order No. 1205 to petitioner. It resolution denying PCGGs motion to
stated that there will a seizure of the documents and disqualify respondent Mendoza since it failed
paraphernalia being used or intended to be used as to prove the existence of an inconsistency
the means of committing illegal recruitment, having between respondent Mendoza’s former
verified that petitioner has— (1) No valid license or function as Solicitor General and his present
authority from the Department of Labor and employment as counsel of the Lucio Tan
Employment to recruit and deploy workers for group. PCGG invokes Rule 6.03 of the Code of
overseas employment; (2) Committed/are committing Professional Responsibility which prohibits
acts prohibited under Article 34 of the New Labor former government lawyers from accepting
Code in relation to Article 38 of the same code. A team engagement or employment in connection
was then tasked to implement the said Order. The with any matter in which he had intervened
group went to petitioner’s residence. They served the while in said service.
order to a certain Mrs. For a Salazar, who let them in. ISSUE: Whether the Sandiganbayan erred in
The team confiscated assorted costumes. Petitioner denying the motion to disqualify
filed with POEA a letter requesting for the return of RULING: No. The Court in interpreting Rule
the seized properties, because she was not given prior 6.03 was not unconcerned with the prejudice
notice and hearing. The said Order violated due to the client which will be caused by its
process. She also alleged that it violated sec 2 of the misapplication. It cannot be doubted that
Bill of Rights, and the properties were confiscated granting a disqualification motion causes the
against her will and were done with unreasonable client to lose not only the law firm of choice,
force and intimidation. but probably an individual lawyer in whom
the client has confidence. The client with a
Consti II (section 2) 20
disqualified lawyer must start again often absolute sale purportedly executed in her favor by
without the benefit of the work done by the Domingo. RTC granted Sy’s petition. Domingo filed a
latter The effects of this prejudice to the right petition seeking the annulment or cancellation of titles,
to choose an effective counsel cannot be injunction and damages as well as a writ of preliminary
overstated for it can result in denial of due injunction and a TRO. This was granted by the Court which
led the Sy to claim that her right to due process was
process.
violated
1.
ISSUE: Whether there was a violation of due process

RULING:

16.17 Morano v Vivo

FACTS: Chan Sau Wah and her minor son Fu Yan


Fun were permitted only into the Philippines under a
temporary visitor's visa for two (2) months. In a letter
dated August 31, 1962, the Commissioner of
Immigration ordered Chan Sau Wah and her son, to
leave the country on or before September 10, 1962
with a warning that upon failure so to do, he will
issue a warrant for their arrest.

ISSUE: Whether the Commissioner of Immigration


can issue a warrant of arrest

RULING: YES. The constitutional guarantee set


forth in Section 1(3), Article III of the Constitution ,
requiring that the issue of probable cause be
determined by a judge (prior issuance of a warrant),
does not extend to deportation proceedings. The view
we here express finds support in the discussions
during the constitutional convention. The convention
recognized, as sanctioned by due process, possibilities
and cases of deprivation of liberty, other than by order
of a competent court.

RATIO: The stay as a temporary visitor is subject to


certain contractual stipulations as contained in the
cash bond put up by him, among them, that in case of
breach the Commissioner may require the
recommitment of the person in whose favor the bond
has been filed. The Commissioner did nothing but to
enforce such condition. Such a step is necessary to
enable the Commissioner to prepare the ground for
his deportation under section 37(a) of Commonwealth
Act 613. A contrary interpretation would render such
power nugatory to the detriment of the State.

16.18 Sy v Domingo

FACTS: The property in dispute was a vacant unfenced lot


situated in White Plains, Quezon City and covered by
Transfer Certificate of Title (TCT) No. N-165606 issued in
the name of respondent Lilia V. Domingo who learned that
construction activities were being undertaken on her
property without her consent. She soon unearthed the
series of anomalous transactions affecting her property. Sy
representing herself as the owner of the property,
petitioned the RTC for the issuance of a new owner’s copy
of Domingo’s TCT appending to her petition, a deed of
Consti II (section 2) 21
16.19 Lucien Tran Van Nghia vs. Liwag Lucien Tran Van order of deportation. A warrant of arrest issued by the
Nghia vs. Liwag, 175 SCRA 318, G.R. No. 78596 July 13, Commissioner of Immigration for purposes of
1989 investigation only, as in the case at bar, is null and
void for being unconstitutional since the same was
FACTS: Petitioner Lucien Tran Van Nghia is a French
issued only for purposes of investigation of the
national temporarily in Manila. Respondent Commissioner
suspects, William Gatchalian included.
of Immigration and Deportation (CID) Commissioner Ramon
J. Liwag received a sworn complaint from a certain Dionisio
17.11 Harvey v. Santiago 162 SCRA 840
G. Cabrera, Jr., allegedly petitioner’s landlord, accusing
petitioner of being an undesirable alien for “committing
Facts: This is a petition for Habeas Corpus.
acts inimical to public safety and progress.” A warrant of
Petitioners are the following: American Nationals
arrest was issued by respondent Commissioner. Nghia now
Andrew Harvey, 52 and John Sherman 72. Dutch
alleges that he was arrested without warrant and deprived
of his liberty by respondent Commissioner of Immigration Citizen Adriaan Van Den Elshout, 58. All reside at
and Deportation and his agents. Pagsanjan Laguna. Respondent Commissioner
Miriam Defensor Santiago issued Mission Orders to
ISSUE: Whether there was a valid warrant the Commission of Immigration and Deportation
(CID) to comprehend petitioners at their residences.
RULING: YES. The requirement of probable cause to be The “Operation Report” read that Andrew Harvey was
determined by a Judge, does not extend to deportation found together with two young boys. Richard
proceedings.’ What is essential is that there should be a Sherman was found with two naked boys inside his
specific charge against the alien intended to be arrested room. While Van Den Elshout in the “after Mission
and deported, that a fair hearing be conducted. Secondly, Report” read that two children of ages 14 and 16 has
records show that formal deportation proceedings have
been under his care and subjects confirmed being live-
been initiated against petitioner before the Board of Special
in for some time now. Seized during the petitioner’s
Inquiry of the CID.10 The restraint (if any) against
petitioner’s person has therefore become legal. The writ of apprehension were rolls of photo negatives and photos
habeas corpus has served its purpose. of suspected child prostitutes shown in scandalous
poses as well as boys and girls engaged in sex. Poster
16. 20 Board of Commissioners v Judge De La Rosa and other literature advertising the child prostitutes
were also found. Petitioners were among the 22
FACTS: On January, 1962, the then Secretary of suspected alien pedophiles. They were apprehended
Justice issued Memorandum No. 9 which directed the February 17, 1988, after close surveillance for 3
Board of Commissioners to review all cases where months of the CID in Pagsanjan., Laguna. 17 of the
entry (to the Philippines) was allowed on the ground arrested aliens opted for self-deportation. One
that the entrant was a Philippine citizen. Among released for lack of evidence, no other charge but
those cases was that of William Gatchalian and working with NO VISA, the 3 petitioners chose to face
others. The new Board of Commissioners, after a deportation proceedings. On March 4, 1988,
review motu proprio of the proceedings in the Board of deportation proceedings were instituted against aliens
Special Inquiry ordered the exclusion of, among for being undesirable aliens under Section 69 of
others, respondent Gatchalian. Gatchalian filed a Revised Administrative Code. Warrants of arrest were
petition for certiorari and prohibition with injunction issued on March 7, 1988 against petitioners for
before the RTC of Manila as presided by respondent violation of Sec. 37, 45 and 46 of Immigration Act and
Judge De La Rosa. The petition is anchored on inter Sec. 69 of Revised Administrative Code. Trial by the
alia, the proposition that: respondent judges have no Board of Special Inquiry III commenced the same
jurisdiction over petitioners (Board of Commissioners, date. Petition for Bail was filed on March 11, 1988 but
et al.,) and the subject matter of the case, appellate was not granted by the Commissioner of Immigration.
jurisdiction being vested by BP 129 with the Court of April 4, 1988, Petitioners filed a petition for Writ of
Appeals Habeas Corpus. The court heard the case on oral
argument on April 20, 1988.
ISSUE: Whether the warrant is valid

RULING:NO. From a perusal of the Immigration Act,


it is clear that in matters insofar as deportation of Issue: Whether the Commissioner has the power to
aliens are concerned, the Commissioner of arrest and detain petitioners pending determination
Immigration may issue warrants of arrest only after a of existence of probable cause.
determination by the Board of Commissioners of the
existence of the ground for deportation as charged Ruling: While pedophilia is not a crime under the
against the alien. In other words, a warrant of arrest Revised Penal Code, it violates the declared policy of
issued by the Commissioner of Immigration, to be the state to promote and protect the physical, moral,
valid, must be for the sole purpose of executing a final spiritual and social well-being of the youth. The arrest

Consti II (section 2) 22
of petitioners was based in the probable cause Issue: Whether a judge may determine probable
determined after close surveillance of 3 months. The cause and issue a warrant of arrest solely on the basis
existence of probable cause justified the arrest and of the resolution of the prosecutor who conducted the
seizure of articles linked to the offense. The articles preliminary investigation, without having before him
were seized as an incident to a lawful arrest; therefore any of the evidence (such as complainant’s affidavit,
the article are admissible evidences (Rule 126 Section respondent’s counter-affidavit, exhibits, etc.) which
12 of Rules on Criminal Procedure). may have been submitted at the preliminary
investigation.
17.12 Ho v. People 280 SCRA 365
Ruling: Article III Section 2, 1987 Constitution: The
Facts: On August 8, 1991, the Anti-Graft League of right of the people to be secure in their persons,
the Philippines, represented by its chief prosecutor houses, papers, and effects against unreasonable
Atty. Reynaldo L. Bagatsing filed with the Office of searches and seizures of whatever nature and for any
the Ombudsman a complaint against Doris Teresa Ho, purpose shall be inviolable, and no search warrant or
Rolando S. Narciso, Anthony Marden, Arsenio warrant of arrest shall issue except upon probable
Benjamin Santos and Leonardo Odoño. The complaint cause to be determined personally by the judge after
was for alleged violation of Section 3 (g) of Republic examination under oath or affirmation of the
Act 3019 prohibiting a public office from entering into complainant and the witnesses he may produce and
any contract or transaction on behalf of the particularly describing the place to be searched and
government if it is manifestly and grossly the persons or things to be seized.
disadvantageous to the latter, whether or not the
public officer profited or will not profit thereby.

According to the information, Rolando The word “personally” does not appear in the
Narciso, being then the Vice-President of the National corresponding provisions of our previous
Steel Corporation (NSC), a government-owned or Constitutions. This emphasis shows the present
controlled corporation organized and operating under Constitution’s intent to a place a greater degree of
the Philippine laws, and Doris Ho, the President of responsibility upon trial judges than that imposed
National Marine Corporation (NMC), a private under the previous charters.
corporation organized and operating under our
Corporation law, was said to have entered without The Court, in this case, reiterated and
legal justification into a negotiated contract of elaborated on the doctrine laid down in People vs.
affreightment disadvantageous to the NSC for the Inting and ruled that:
haulage of its products at the rate of P129.50/MT,
from Iligan City to Manila. Such contract was entered “First, as held in Inting, the determination of
into despite their full knowledge that the rate they probable cause by the prosecutor is for a purpose
have agreed upon as much higher than those offered different from that which is to be made by the judge.
by the Loadstar Shipping Company, Inc. (LSCI) and Whether there is reasonable ground to believe that
Premier Shippng Lines, Inc. (PSLI), in the amounts of the accused is guilty of the offense charged and should
P109.56 and P123.00 per Metric Ton, respectively, in be held for trial is what the prosecutor passes upon.
the public bidding, thereby giving unwarranted The judge, on the other hand, determines whether a
benefits to the National Marine Corporation. warrant of arrest should be issued against the
accused, i.e. whether there is a necessity for placing
Ho and Narciso alleged that the him under immediate custody in order not to frustrate
Sandiganbayan, in determining probable cause for the the ends of justice. Thus, even if both should base
issuance of the warrant of their arrest, merely relied their findings on one and the same proceeding or
on the information and the resolution attached evidence, there should be no confusion as to their
thereto, filed by the Ombudsmand without other distinct objectives.
supporting evidence, in violation of the requirements
of Section 2, Article III of the Constitution, and Second, since their objectives are different,
settled jurisprudence. They contend that a judge, in the judge cannot rely solely on the report of the
personally determining the existence of probable prosecutor in finding probable cause to justify the
cause, must have before him sufficient evidence issuance of a warrant of arrest. Obviously and
submitted by the parties, other than the information understandably, the contents of the prosecutor’s
filed by the investigating prosecutor, to support his report will support his own conclusion that there is
conclusion and justify the issuance of an arrest reason to charge the accused of an offense and hold
warrant. Such evidence should not be “merely him for trial. However, the judge must decide
described in a prosecutor’s resolution.” independently. Hence, he must have supporting
evidence, other than the prosecutor’s bare report,

Consti II (section 2) 23
upon which to legally sustain his own findings on the 17.13 Board of Commissioners v. Judge De La
existence of probable cause to issue an arrest order. Rosa 197 SCRA 853
Their responsibility of determining personally and
independently the existence or nonexistence of Facts: On July 12, 1960, Santiago Gatchalian,
probable cause is lodged in him by no less than the grandfather of William Gatchalian, was recognized by
most basic law of the land. Parenthetically, the the Bureau of Immigration as a native born Filipino
prosecutor could ease the burden of the judge and citizen following the citizenship of natural mother
speed up the litigation process by forwarding to the Mariana Gatchalian. On June 27, 1961, William, the
latter not only the information and his bare resolution 12 years old, arrived in Manila from Hongkong
finding probable cause, but also so much of the together with a daughter and a son of Santiago. They
records and the evidence on hand as to enable his had with them certificate of registration and identity
honor to make his personal and separate judicial issued by the Philippine consulate in Hongkong based
finding on whether to issue a warrant of arrest. on a cablegram bearing the signature of the secretary
of foreign affairs, Felixberto Serrano, and sought
Lastly, it is not required that the complete or admission as Filipino citizens. On July 6, 1961, the
entire records of the case during the preliminary board of special inquiry admitted the Gatchalians as
investigation be submitted to and examined by the Filipino citizens and issued an identification
judge. We do not intend to unduly burden trial courts certificate to William. The Board of Commissioners
by obliging them to examine te complete records of directed by the Secretary of Justice to Review all
every case all the time simply for the purpose of cases where entry was allowed on the grounds that
ordering the arrest of an accused. What is required, the entrant was a Filipino citizen such included the
rather, is that the judge must have sufficient case of William. As a result of the decision of the
supporting documents (such as the complaint, board of special inquiry which recommended for the
affidavits, counter-affidavits, sworn statements of reversal of the decision of the Board of
witnesses or transcripts of stenographic notes, if any) Commissioners. Acting Commissioner issued an order
upon which to make his independent judgment or, at affirming the decision of the Board of Special Inquiry.
the very least, upon which to verify the findings of the On August 15, 1990, the Commission on Immigration
prosecutor as to the existence of probable cause. The and Deportation ordered the arrest of William and
point is: he cannot rely solely and entirely on the was released upon posting P200,000 cash bond. Thus
prosecutor’s recommendation, as Respondent Court on the 29th of the same month, he filed a petition for
did in this case enjoys the legal presumption or certiorari and prohibition before the RTC of Manila. A
regularity in the performance of his official duties and motion to dismiss was filed but denied.
functions, which in turn gives his report the
presumption accuracy, the Constitution, we repeat, Issue: Whether William Gatchalian is to be declare as
commands the judge to personally determine probable a Filipino citizen.
cause in the issuance of warrants of arrest. This
Court consistently held that judge fails in his Ruling: William Gatchalian is declared as a Filipino
bounden duty if he relies merely on the citizen. Having declared the assailed marriage as
certification or the report of the investigating valid, respondent William Gatchalian follows the
officer.” citizenship of his father, a Filipino as legitimate child.
Respondent belongs to a class of Filipinos who are
citizens of the Philippines at the time of the adoption
of the Constitution.
IN THE INSTANT CASE, the public respondent
relied fully and completely upon the resolution of the 5. “Of Whatever Nature and for Any Purpose”
graft investigation officer and the memorandum of the
reviewing prosecutor, attached to the information 17.14 Material Distribution v. Judge 84 Phil 127
filed before it, and its conjecture that the Ombudsman (1989)
would not have approved their recommendation
Facts: On March 24, 1947, Lope Sarreal filed a
without supporting evidence. It had no other
complaint (amended on April 10, 1947, to include
documents from either the complainant (the Anti-
Harry Lyons) seeking a money judgment against
Graft League of the Philippines) Court palpably
petitioners on three causes of action in the total of
committed grave abuse of discretion in ipso facto
P1,256,229.30. On May 27, 1947, Sarreal filed a
issuing the challenged warrant of arrest on the sole
motion for the production and inspection of
basis of the prosecutor’s findings and
documents. On June 4, 1947, Sarreal filed a
recommendation, and without determining on its own
supplemental motion for the production and
the issue of probable cause based on evidence other
inspection of the originals of Annexes A and B of the
than such bare findings and recommendation.
complaint. On June 12, 1947, petitioner filed a

Consti II (section 2) 24
memorandum and opposition to Sarreal’s original and to investigate conditions and practices of employment
supplemental motion on the ground that he failed to in any industry subject to the Act, and of Section 9,
show good cause and that the motion were evidently incorporating the provisions of Section 9 of the
filed for the purpose of fishing evidence. On July 16, Federal Trade Commission Act authorizing the
1947, respondent judge, granting both motions, issuance and judicial enforcement of subpoenas, were
required petitioners to produce the documents and challenge by the petitioners, contending that they
annexes in question on July 24, 1947. On account of violate the prohibitions of the Fourth Amendment
the absence in the Philippines of Harry Lyons, against unreasonable searches and seizures or any
petitioner moved, reserving whatever rights they have other provision of the Constitution. Said law applies
under the Rules of Court, to postpone the inspection of to business of publishing and distributing newspapers
the documents and annexes in question and like the herein petitioners. Respondent conducted an
accordingly respondent judge postponed it to August investigation pursuant to the above-mentioned law
15, 1947. Petitioners impugn the validity of the orders against the petitioners.
of July 16 and September 27, 1947, as were issued by
the respondent judge in excess of his jurisdiction or Issue: Whether the Fair Labor Standard Act violates
with grave abuse of his discretion, and prayed for the the fourth amendment.
annulment or modificationOf the order of July 16,
1947. Ruling: No. The Fourth Amendment, if applicable,
guards against abuse only by way of too much
Issue: Whether the order of the trial judge violated indefiniteness or breadth in the things required to be
petitioner’s constitutional rights against self- “particularly described,” if the inquiry is one the
incrimination. demanding agency is authorized by law to make and
the materials specified are relevant, the gist of the
Ruling: This Court have considered carefully persons protection being the requirement that the disclosure
advanced by petitioners and memoranda in support of sought shall not be unreasonable. There was no
this allegation and found nothing in them to show violation of petitioner’s rights in these cases, since
how, without the inspection of Annexes A and B of the both petitioners were corporations; the only records or
complaint petitioners may incriminate themselves. documents sought were corporate ones; no element of
The Court have, therefore, to dismiss such contention. self-incrimination was presented or claimed; all the
records sought were relevant to an inquiry for the
The orders in question, issued in virtue of the purpose of determining whether petitioners were
provisions of Rule 21, pertain to a civil procedure that subject to the Act and, if so, whether they were
cannot be identified or confused with the violating it; and such an inquiry was authorized by
unreasonable searches prohibited by the Constitution. Section 11 (a) of the Fair Labor Standards Act.
But in the erroneous hypothesis that the production
and inspection of books and documents in question is 17.16 Camara v. Municipal Court, 387 US 523
tantamount to a search warrant, the procedure (1967)
outlined by Rule 21 and followed by respondent judge
place them outside the realm of the prohibited Facts: On November 6, 1963, an inspector of the
unreasonable searches. Division of Housing Inspection of the San Francisco
Department of Public Health entered an apartment
The constitutional guarantee of privacy of building to make a routine annual inspection for
communication and correspondence will not be possible violations of the city’s Housing Code.” The
violated, because the trial court has power and inspector was informed that the Appellant was using
jurisdiction to issue an order for the production and part of his leasehold as a personal residence. The
inspection of the books and documents in question in inspector confronted the Appellant and demanded to
virtue of the constitutional guarantee making an inspect the premises because residential use was not
express exception in favor of the disclosure of allowed on the first floor of the apartment building.
communication and correspondence upon lawful order The Appellant did not allow the inspector to enter
of a court of justice. After a careful consideration of because he did not have a warrant. The inspector
the legal question raised by petitioners, this Court has attempted to obtain access to Appellant’s apartment a
arrived at the conclusion that the trial judge, in second time two days later, and again the Appellant
issuing the order of July 16, 1947, has not exceed his refused to grant him access. The Appellant then was
jurisdiction or acted with grave abuse of discretion. sent a summons ordering him to appear at the district
attorney’s office. The Appellant did not appear and a
17.15 Oklahoma Press v. Walling, 327 US 186 few weeks later two other inspectors attempted to
gain access to his apartment and were again refused
Facts: The provisions of Section 11(a) of the Fair because they did not have a search warrant. A
Labor Standards Act, authorizing the Administrator complaint was then filed against the Appellant for
Consti II (section 2) 25
violation of the Housing Code. His demurrer was standards for conducting an area inspection are
denied and he filed a writ of prohibition. The court of satisfied with respect to a particular dwelling.
Appeals held the housing section “does not violate
Fourth Amendment rights because it is part of a
regulatory scheme which is essentially civil rather
than criminal in nature, inasmuch as that section
creates a right of inspection which is limited in scope
6. Warrantless Searches and Seizures
and may not be exercised under reasonable
conditions.” A. General Rule: Get a Search Warrant
Issue: Whether administrative inspection programs, 17.17 People v. Aminuddin, 163 SCRA 402
as presently authorized and conducted, violate Fourth
Amendment rights as those rights are enforced Facts: Idel Aminuddin, accused-appellant was
against the States through Fourteenth Amendment. arrested on June 25, 1984, shortly after disembarking
from the M/V Wilcon 9 at about 8:30 in the evening, in
Ruling: Yes. (Frank v. Maryland) to the exten that it Iloilo City. The PC officers who were in fact waiting
sanctioned such warrantless inspections, must be for him because of a tip from one of their informers
overruled.” In Frank v. Maryland, the Supreme Court simply accosted him, inspected his bag and finding
upheld the conviction of one who refused to permit a what looked like marijuana leaves took him to their
warrantless inspection of private premises for the headquarters for investigation. The two bundles of
purposes of locating and abating a suspected public suspect articles were confiscated from him and later
nuisance.” The Frank opinion has been generally been taken to the NBI laboratory for examination. It was
interpreted as carving out an additional exception to found to contain three kilos of what were later
the rule that warrantless searches are unreasonable analyzed as marijuana leaves by an NBI forensic
under the Fourth Amendment.” The majority here examiner. An information for violation of the
observed, the practical effect of this system is to leave Dangerous Drugs Act was filed against him. Later,
the occupant subject to the discretion of the official in the information was amended to include Farida Ali y
the field. This is precisely the discretion to invade Hassen, who had also been arrested with him that
private property which we have consistently same evening and likewise investigated. Both were
circumscribed by a requirement that a disinterested arraigned and pleaded not guilty. Subsequently, the
party warrant the need to search. We simply cannot fiscal filed a motion to dismiss the charge against Ali
say that the protections provided by the warrant on the basis of a sworn statement of the arresting
procedure are not needed in this context; broad officers absolving her after a thorough investigation.
statutory safeguards are no subtle for individualized The motion was granted, and trial proceeded only
review, particularly when those safeguards may only against the accused-appellant, who was eventually
be invoked at the risk of a criminal penalty. convicted. In his defense, Aminuddin disclaimed
marijuana, averring that all he had in his bag was his
Unfortunately, there can be no ready test for
clothing consisting of a jacket, two shirts and two
determining reasonableness of a search other than by
pairs of pants. He alleged that he was arbitrarily
balancing the need to search against the invasion
arrested and immediately handcuffed. His bag was
which the search entails. But the majority thought
confiscated without a search warrant. At the PC
that a number if persuasive inspections. First, such
headquarters, he was manhandled to force him to
programs have a long history of judicial and public
admit he was carrying the marijuana, the investigator
acceptance. Second, the public interest demands that
hitting him with a piece of wood in the chest and arms
all dangerous conditions be prevented or abated, yet it
even as he parried the blows while he was still
is doubtful that any other canvassing technique would
handcuffed. He insisted he did not even know what
achieve acceptable results. Many such conditions –
marijuana looked like and that his business was
faulty wiring is an obvious example – are not
selling watches and sometimes cigarettes. However,
observable from outside the building and indeed may
the RTC rejected his allegations. Saying that he only
not be apparent to the inexpert occupant himself.
has two watches during that time and that he did not
Finally, because the inspections are neither personal
sufficiently proved the injuries allegedly sustained.
in nature nor aimed at the discovery of evidence of
crime, they involve a relatively limited invasion of the Issue: Whether the search of defendant’s bad is legal.
urban citizen’s privacy. Further, after concluded that
the area inspection is a reasonable search of private Ruling: the search is illegal. Defendant was not
property within the meaning of the Fourth caught in flagrante delicto, which could allow
Amendment, the majority observed it is obvious that warrantless arrest or search. At the moment of his
“probable cause” to issue a warrant to inspect must arrest, he was no committing a crime. Nor was he
exist if reasonable legislative or administrative about to do so or had just done so. To all appearances,
Consti II (section 2) 26
he was like any of the other passengers innocently followed. The Regional Trial Court found the
disembarking from the vessel. The said marijuana appellant guilty beyond reasonable doubt of illegal
therefore could not be appreciated as evidence against possession of shabu under section 11, Article II of R.A.
the defendant, and furthermore he is acquitted of the No. 9165, and sentenced to suffer the penalty of life
crime charged. imprisonment. It also ordered him to pay P500,000.00
fine. The Court of Appeals affirmed the RTC decision.
17.18 People v. Valdez, 341 SCRA 85 Appellant move to reconsider this decision, but the CA
denied his motion.
Facts: SPO3 Marcelo Tipay, a member of the police
force of Villaverde, Nueva Vizcaya, testified that at Issue: Whether the warrantless arrest was valid, and
around 10:15 a.m. of September 24, 1996, he received if so, whether the seized items were admissible.
a tip from an unnamed informer about the presence of
a marijuana plantation, allegedly owned by Abe Ruling: No. the shabu purportedly seized from the
Valdez y De La Cruz at Sitio Bulan, Ibung, Villaverde, appellant is inadmissible in evidence for being the
Nueva Vizcaya. The prohibited plants were allegedly proverbial fruit of the poisonous tree. Section 5 (a),
planted close to appellant’s hut. Police Inspector Rule 113 of the Rules of Criminal Procedure provides
Alejandro R. Parungao, Chief of Police of Villaverder, that a peace officer or a private person may, without a
Nueva Vizcaya then formed a reaction team from his warrant, arrest a person, when in his presence the
operatives to verify the report. At approximately 5:00 person to be arrested has committed, is actually
o’clock A.M. the following day, said police team, committing, or is attempting to commit an offense.
accompanied by their informer, left for the site where This is known as arrest in flagrante delicto.
the marijuana plants were allegedly being grown.
After a three-hour, uphill trek from the nearest “For a warrantless arrest of an accused caught
barangay road, the police operatives arrived at the in flagrante delicto to be valid, two requisites must
place pinpointed by their informant. The police found concur: (1) THE PERSON TO BE ARRESTED MUST
appellant alone in his nipa hut. They then proceeded EXECUTED AN OVERT ACT INDICATING THAT
to look around the area where appellant had his HE HAS JUST COMMITTED, IS ACTUALLY
kaingin and saw seven five-foot high, flowering COMMITTING, OR WAS ATTEMPTING TO
marijuana plants in two rows, approximately 25 COMMIT A CRIME. In fact, SPO3 Corbe testified
meters from appellant’s hut. The appellant admitted that the appellant and the informant were just
that the said marijuana was owned by him. The police talking with each other when he approached them.
took photos of appellant standing beside the cannabis There was no exchange of money and drugs when he
plants. Appellant was then arrested. The accused was approached the car.
found guilty beyond reasonable doubt by trial court of
cultivating marijuana plants punishable under 17.20 Dale Grady v. North Carolina, March 30, 2015
section 9 of Dangerous Drugs Act of 1972, as amended
Facts: Between 1997 and 2006, Dale Grady was
and was sentenced to suffer the penalty of death by
convicted of two sexual offenses. After being released
lethal injection. The accused-appellant contended
for the second time, a trial court civilly committed
there was unlawful search and that the court erred in
Grady to take part in North Carolina’s satellite-based
declaring the marijuana plants, as evidence despite
monitoring program for the duration of his life. The
that was the product of an illegal search.
program required participants to wear a GPS
Issue: Whether the search and seizure of the monitoring bracelet so that authorities can make sure
marijuana plants in this case lawful. that participants are complying with prescriptive
schedule and location requirements. Grady challenged
Ruling: No. in the instant case, there was no search the constitutionality of the program and argued that
warrant issued by a judge after personal the constant tracking amounted to unreasonable
determination of the existence of probable cause. The search that was prohibited under the Fourth
Supreme Court Reversed the decision of the RTC and Amendment. Both the trial court and North Carolina
the appellant was acquitted. Court of Appeals held that wearing a GPS monitor did
not amount to a search.
17.19 People v. Oliver Edano, GR No.188133, 729
SCRA 255, July 7, 2014 Issue: Whether wearing of a GPS monitor constitute
a search under the Fourth Amendment.
Facts: The prosecution charged appellant Edaño and
Godofredo Siochi with violation of Section 11, Article Ruling: Yes. In a per curiam opinion, the Court held
II of R.A No. 9165 under two separate Informations. that the trial court and appellate court both failed to
The appellant and Siochi pleaded not guilty to the apply correct law based on the Court’s decision in
charge on arraignment. Joint trial on the merits United States v. Jones, which held that placing a GPS

Consti II (section 2) 27
tracker on the bottom of a vehicle constituted a search seizures areprohibited. Those, which are reasonable,
under the Fourth Amendment. The Court held that are not forbidden.
participation in the North Carolina program
amounted to a search because requiring someone to Between the inherent right of thestate to protect its
wear a bracelet that tracks the person’s whereabouts existence and promote public welfare and an
constitutes what the Jones decision termed a individual’s right against awarrantless search w/c is,
“physical occupation of private property for the however, reasonably conducted, the former should
purpose of obtaining information.” The Court prevail. True,the manning of checkpoints by the
remanded the case back to the trial court for a military is susceptible of abuse by the military in
determination of whether or not this “search” was thesame manner that all governmental power is
unreasonable under the Fourth Amendment. susceptible of abuse. But, at the cost ofoccasional
inconvenience, discomfort and even irritation to the
B. When is search a “search” citizen, the checkpointsduring these abnormal times,
when conducted w/in reasonable limits, are part of the
18.11 Valmonte v. General de Villa – 178 SCRA 211 pricewe pay for an orderly society and a peaceful
(Main) and 185 SCRA 655 (MR) community.

FACTS: The National Capital Region District 18.12 Guazon v. De Villa – 181 SCRA 623
Command (NCRDC) was activated pursuant to Letter
of Instruction 02/87 of the Philippine General The military and police officers conducted “Areal
Headquarters, AFP. As part of its duty to maintain Target Zonings” or “saturation drives” in Metro
peace and order, the NCRDC installed checkpoints in Manila, specifically on places where the subversives,
various parts of Valenzuela, Metro Manila. as pinpointed by said authorities, were hiding. During
Petitioners aver that, because of the installation of these saturation drives, police and military units
said checkpoints, the residents of Valenzuela are cordon an area o! more than one residence and
worried of being harassed and of their safety being sometimes the whole barangay or areas of barangays,
placed at the arbitrary, capricious and whimsical without any search warrant or warrant o! arrest.
disposition of the military manning the checkpoints, Petitioners claimed that said saturation drives
considering that their cars and vehicles are being followed a common pattern of human rights abuses,
subjected to regular searches and check-ups, as such, sought for its stoppage.
especially at night or at dawn, without the benefit of a
search warrant and/or court order.

Their alleged fear for their safety increased when, at FACTS: The petitioners, who are of legal age,
dawn of 9 July 1988, Benjamin Parpon, a supply bonafide residents of Metro Manila, and taxpayers
officer of the Municipality of Valenzuela, Bulacan, and leaders in their respective communities, sought to
was gunned down allegedly in cold blood by the prohibit the military and police officers from
members of the NCRDC manning the checkpoint conducting “real Target Zonings" or “saturation
along McArthur Highway at Malinta, Valenzuela, for drives" in Metro Manila.
ignoring and/or refusing to submit himself to the
checkpoint and for continuing to speed off inspire of ISSUE: Whether a search may be conducted legally
warning shots fired in the air. without search warrant.

ISSUE: Whether the installation of checkpoints RULING: No. Nothing in the Constitution denies the
violates the right of the people against unreasonable authority of the Chief Executive to order police
searches and seizures actions to stop unabated criminality, rising
lawlessness and alarming communist activities.
RULING: No. Petitioner's concern for their safety However, all police actions are governed by the
and apprehension at being harassed by the military limitations of the Bill of Rights. The Government
manning the checkpoints are not sufficient grounds to cannot adopt the same reprehensible methods of
declare the checkpoints per se, illegal. No proof has authoritarian systems both of the right and of the left,
been presented before the Court to show that, in the the enlargement of whose spheres of influence it is
course of theirroutine checks, the military, indeed, trying hard to suppress. Our democratic institutions
committed specific violations of petitioners'' may still be fragile but they are not in the least bit
rightsagainst unlawful search and seizure of other strengthened through violations of the constitutional
rights. The constitutional right againstunreasonable protections which are their distinguishing features.
searches and seizures is a personal right invoked only Moreover, the right to be left alone in the privacy of
by those whose rightshave been infringed, or his own house guaranteed under Article IV of the
threatened to be infringed. Not all searches and

Consti II (section 2) 28
1973 Constitution which the Government may not probable cause to make the arrest at the outset of the
touch. search. The question, therefore, is whether the police
herein had probable cause to arrest Tudtud, et. al.
18.13 People v. Tudtud, GR 144037, Sept 26, 2003 The long-standing rule in this jurisdiction, applied
with a great degree of consistency, is that “reliable
Facts: Sometime during the months of July and information” alone is not sufficient to justify a
August 1999, the Toril Police Station, Davao City warrantless arrest under Section 5 (a), Rule 113. The
received a report from a “civilian asset” named rule requires, in addition, that the accused perform
Bobong Solier about a certain Noel Tudtud. Solier some overt act that would indicate that he “has
related that his neighbors have been complaining committed, is actually committing, or is attempting to
about Tudtud, who was allegedly responsible for the commit an offense.” For the exception in Section 5 (a),
proliferation of marijuana in their area. Reacting to Rule 113 to apply, this Court ruled, two elements
the report, PO1 Ronald Desierto, PO1 Ramil Floreta must concur: (1) the person to be arrested must
and their superior, SPO1 Villalonghan, all members execute an overt act indicating he has just committed,
of the Intelligence Section of the Toril Police Station, is actually committing, or is attempting to commit a
conducted surveillance in Solier’s neighborhood in crime; and (2) such overt act is done in the presence or
Sapa, Toril, Davao City. For 5 days, they gathered within the view of the arresting officer. Reliable
information and learned that Tudtud was involved in information alone is insufficient. The search of
illegal drugs. PO1 Floreta and PO1 Desierto then Tudtud's box does not come under the recognized
approached the suspects and identified themselves as exceptions to a valid warrantless search, the
police officers. PO1 Desierto informed them that the marijuana leaves obtained thereby are inadmissible
police had received information that stocks of illegal in evidence. And as there is no evidence other than
drugs would be arriving that night. The man who the hearsay testimony of the arresting officers and
resembled Tudtud’s description denied that he was their informant, the conviction of Tudtud, et. al.
carrying any drugs. PO1 Desierto asked him if he cannot be sustained.
could see the contents of the box. Tudtud obliged,
saying, “it was alright.” Tudtud opened the box 18.14 Sony Music v. Judge Espanol, GR 156804,
himself as his companion looked on. The box yielded March 14, 2005
pieces of dried fish, beneath which were two bundles,
one wrapped in a striped plastic bag and another in The presumption juris tantum of regularity cannot, by
newspapers. PO1 Desierto asked Tudtud to unwrap itself, prevail against the constitutionally protected
the packages. They contained what seemed to the rights of an individual because zeal in the pursuit of
police officers as marijuana leaves. The police thus criminals cannot ennoble the use of arbitrary methods
arrested Tudtud and his companion, informed them of that the Constitution itself detests.
their rights and brought them to the police station.
The two did not resist. The confiscated items were FACTS: In a criminal complaint filed with the DOJ,
turned over to the Philippine National Police (PNP) the VRB charged Respondents with violation of PD
Crime Laboratory for examination. Forensic tests on 1987. As alleged in the complaint, the four were
specimens taken from the confiscated items confirmed engaged in the replication, reproduction and
the police officers’ suspicion. The plastic bag distribution of videograms without license and
contained 3,200 grams of marijuana leaves while the authority from Videograms Regulatory Board. The
newspapers contained another 890 grams. Noel NBI through Agent Lavin applied on with the RTC
Tudtud and his companion, Dindo Bulong, were presided by respondent judge for the issuance of
subsequently charged before the Regional Trial Court search warrants against private respondents which
(RTC) of Davao City with illegal possession of was then issued.
prohibited drugs.
Individual private respondents, through counsel filed
Issue: Whether the warrantless arrest was proper. a motion to quash search warrant and to release
seized properties grounded on lack of probable cause,
RULING:No. The RTC justified the warrantless that the warrant did not sufficiently describe the
search of appellants’ belongings under the first items to be seized and that the warrant was
exception, as a search incident to a lawful arrest. A improperly enforced.
search incidental to a lawful arrest is sanctioned by
the Rules of Court. It is significant to note that the ISSUE: Whether the warrant was issued without
search in question preceded the arrest. Recent probable cause.
jurisprudence holds that the arrest must precede the
search; the process cannot be reversed. Nevertheless, RULING: In the determination of probable cause, the
a search substantially contemporaneous with an court must necessarily resolve whether or not an
arrest can precede the arrest if the police have offense exist to justify the issuance or quashal of the

Consti II (section 2) 29
warrant. Moreover, a core requisite before a warrant 18.16 Padilla v. CA, GR 121917 March 12, 1997
shall validly issue is the existence of a probable cause,
meaning “the existence of such facts and FACTS: A vehicle heading north with plate number
circumstances which would lead a reasonably discreet PMA 777 was involved in a hit and run accident.
and prudent man to believe that an offense has been SPO2 Borja and SPO2 Miranda of Mobile 3 were able
committed and that the objects sought in connection to intercept the vehicle by cutting into the latter's
with the offense are in the place to be searched.” path forcing it to stop. SPO2 Miranda went to the
vehicle with plate number PMA 777 and instructed its
In the exercise of this mandate which we can allow as driver to alight. The driver rolled down the window
being akin to conducting a preliminary investigation and put his head out while raising both his hands.
abuse of discretion cannot plausibly be laid at the They recognized the driver as Robin C. Padilla. SPO2
doorstep of the issuing court on account of its prima Miranda told Padilla to alight to which Padilla
facie holding that no offense has been committed, complied. Padilla was wearing a short leather jacket
even if consequent to such holding a warrant is such that when he alighted with both his hands
recalled and the private complainant is incidentally raised, a gun tucked on the left side of his waist was
deprived of vital evidence to prove his case. revealed, its butt protruding. SPO2 Borja made the
move to confiscate the gun but Padilla held the
D. Instances of Warrantless Searches and former's hand alleging that the gun was covered by
Seizures legal papers. SPO2 Borja disarmed Padilla and told
the latter about the hit and run incident. Padilla,
18.15 People v. Sevilla– 339 SCRA 625 however, arrogantly denied his misdeed and, instead,
played with the crowd by holding their hands with
FACTS: Automatic review after the trial court find
one hand and pointing to SPO2 Borja with his right
both the accused guilty beyond reasonable doubt of
hand saying "iyan, kinuha ang baril ko." Because
the crime of violation of Sec 8 , Art II, Republic Act
Padilla's jacket was short, his gesture exposed a long
6425, as amended by R.A 7659, An Act To Impose The
magazine of an armalite rifle tucked in his back right
Death Penalty On Certain Heinous Crimes,
pocket. SPO Mercado saw this and so when Padilla
Amending For That Purpose The Revised Penal Laws,
turned around as he was talking and proceeding to his
As Amended, Other Special Penal Laws, And For
vehicle, Mercado confiscated the magazine from
Other Purposes both of them are hereby sentenced to
Padilla. SPO Mercado modified the arrest of Padilla
suffer the penalty of Death with all the accessory
by including as its ground illegal possession of
penalties provided by law. Accused are now
firearms. SPO Mercado then read to appellant his
contending that the court erred in convicting the
constitutional rights. The police officers brought
accused despite the absence of search warrant.
Padilla to the Traffic Division at Jake Gonzales
ISSUE: Whether the warrantless search and seizure Boulevard where Padilla voluntarily surrendered a
was valid. third firearm, a pietro berreta pistol with a single
round in its chamber and a magazine loaded with 7
RULING: YES. There are certain cases where the other live bullets.
law itself allows a search even in the absence of a
warrant. Jurisprudence mentions the following ISSUE: Whether the firearms and ammunition
instances under which a warrantless search and confiscated during a warrantless search and seizure,
seizure may be effected, to wit: 1. Search which is especially the baby armalite, are admissible as
incidental to the arrest. 2. Seizure of evidence in plain evidence against Robin Padilla.
view 3. Search of moving vehicle 4 Consented
RULING: Yes. When in pursuing an illegal action or
warrantless search 5. Customs search 6. Stop and
in the commission of a criminal offense, the police
frisk 7. Exigent and emergency circumstances.
officers should happen to discover a criminal offense
i. Incidental to a Lawful Arrest being committed by any person, they are not
precluded from performing their duties as police
Sec. 12 Rule 16, Rules of Court officers for the apprehension of the guilty person and
the taking of the corpus delicti. Objects whose
Two Requisites: possession are prohibited by law inadvertently found
in plain view are subject to seizure even without a
1. Item to be searched was within the warrant." With respect to the Berreta pistol and a
arrestee’s custody or area of immediate black bag containing assorted magazines, Padilla
control. voluntarily surrendered them to the police. This latter
gesture of Padilla indicated a waiver of his right
2. Search was contemporaneous with an against the alleged search and seizure, and that his
arrest. failure to quash the information estopped him from
Consti II (section 2) 30
assailing any purported defect. Even assuming that Espano during his arrest were valid under the
the firearms and ammunitions were products of an doctrine of search made incidental to a lawful arrest.
active search done by the authorities on the person The warrantless search made in his house, however,
and vehicle of Padilla, their seizure without a search which yielded ten cellophane bags of marijuana
warrant nonetheless can still be justified under a became unlawful since the police officers were not
search incidental to a lawful arrest (first instance). armed with a search warrant at the time. Moreover, it
Once the lawful arrest was effected, the police may was beyond the reach and control of Espano. The right
undertake a protective search of the passenger of the people to be secure in their persons, houses,
compartment and containers in the vehicle which are papers and effects against unreasonable searches and
within Padilla's grabbing distance regardless of the seizures of whatever nature and for any purposes
nature of the offense. shall be inviolable, and no search warrant or warrant
of arrest shall issue except upon probable cause to be
18.17 Espano v. CA 288 SCRA 558 (1998) determined personally by the judge after examination
under oath or affirmation of the complainant and the
FACTS: On 14 July 1991, at about 12:30 a.m., Pat. witnesses he may produce, and particularly describing
Romeo Pagilagan and other police officers, namely, the place to be searched and the persons or things to
Pat. Wilfredo Aquilino, Simplicio Rivera, and Erlindo be seized." An exception to the said rule is a
Lumboy of the Western Police District (WPD), warrantless search incidental to a lawful arrest for
Narcotics Division went to Zamora and Pandacan dangerous weapons or anything which may be used as
Streets, Manila to confirm reports of drug pushing in proof of the commission of an offense. It may extend
the area. They saw Rodolfo Espano selling beyond the person of the one arrested to include the
"something" to another person. After the alleged premises or surroundings under his immediate
buyer left, they approached Espano, identified control. Herein, the ten cellophane bags of marijuana
themselves as policemen, and frisked him. The search seized at petitioner's house after his arrest at
yielded two plastic cellophane tea bags of marijuana . Pandacan and Zamora Streets do not fall under the
When asked if he had more marijuana, he replied that said exceptions.
there was more in his house. The policemen went to
his residence where they found ten more cellophane 18.18 People v. De Lara – 236 SCRA 291
tea bags of marijuana. Espano was brought to the
police headquarters where he was charged with FACTS: This is an appeal from the decision of the
possession of prohibited drugs. On 24 July 1991, Regional Trial Court finding appellant guilty beyond
Espano posted bail and the trial court issued his order reasonable doubt for the sale of prohibited drugs. The
of release on 29 July 1991. On 14 August 1992, the National Criminal Investigation Service were ordered
trial court rendered a decision, convicting Espano of to conduct a surveillance operation in the vicinity of
the crime charged. Espano appealed the decision to Garrido and Zamora Streets at Sta. Ana, Manila,
the Court of Appeals. The appellate court, however, on after receiving reports of rampant drug-pushing in
15 January 1995 affirmed the decision of the trial that area. No arrest was made because the team was
court in toto. Espano filed a petition for review with instructed by their superior to conduct a surveillance
the Supreme Court. operation only. Weeks after, Capt. Cablayan
instructed Sgt. David to plan a buy-bust operation.
ISSUE: Whether the search of Espano’s home after The confidential informant proceeded to the house of
his arrest does not violate against his right against appellant and bought two foils of marijuana. It was
unreasonable search and seizure. after the accused handed the two foils that he sensed
the presence of the police operatives. Thereafter, the
RULING: Espano's arrest falls squarely under Rule team, together with appellant, proceeded to the WPD
113 Section 5(a) of the Rules of Court. He was caught headquarters for investigation. The prohibited drugs
in flagranti as a result of a buy-bust operation seized from appellant were brought to the NBI for
conducted by police officers on the basis of chemical analysis. In his appeal, appellant questions
information received regarding the illegal trade of the legality of his arrest and the seizure of prohibited
drugs within the area of Zamora and Pandacan drugs found inside his house.
Streets, Manila. The police officer saw Espano
handing over something to an alleged buyer. After the ISSUE: Whether or not the arrest without warrant
buyer left, they searched him and discovered two was lawful
cellophanes of marijuana. His arrest was, therefore,
lawful and the two cellophane bags of marijuana RULING: As to the legality of appellant's arrest, we
seized were admissible in evidence, being the fruits of find that the police operatives acted within the
the crime. As for the 10 cellophane bags of marijuana bounds of law. In the case at bench, appellant was
found at Espano's residence, however, the same caught red-handed in delivering two tin foils of
inadmissible in evidence. The articles seized from marijuana to the poseur-buyer. Applying the
Consti II (section 2) 31
aforementioned provision of law, appellant's arrest search obviously cannot be made in a place other than
was lawfully effected without need of a warrant of the place of arrest. In the case at bar, appellants were
arrest. "Having caught the appellant in flagrante as a arrested in Room 504 of the Las Palmas Hotel. The
result of the buy-bust operation, the policemen were piece of paper bearing Leangsiri’s name was obtained
not only authorized but were also under obligation to through a warrantless search of Room 413 of the same
apprehend the drug pusher even without a warrant of hotel, and found tucked within the pages of appellant
arrest" Amidus telephone and address book. Clearly, the
warrantless search is illegal and the piece of paper
18.19 People v. Leangsiri – 252 SCRA 213 bearing Leangsiris name cannot be admitted as
evidence against appellants. The inadmissibility of
FACTS: Leangsiri, a Thai national, was arrested at this evidence will not, however, exculpate
the arrival area of the (NAIA). He was in the act of appellants. Its exclusion does not destroy the
bringing into the country heroin hidden under a black prosecution’s case against appellants.
suitcase. He was brought to the headquarters of the
Narcotics Command (NARCOM) at the Old MIA for 18.20 People v. Cuenco – GR 128277, Nov. 16, 1998
further investigation. The head of the command
formed a team to conduct follow-up operations in the FACTS: Ferdinand Cueno y Mata was accused for the
case. Leangsiri received a telephone call from his sale of Marijuana, a prohibited drug. A surveillance
contact and was told that the black suitcase would be operations was ordered to be conducted on the alleged
picked up at about ten oclock that night. He relayed drug pushing activities of herein accused. During the
the information to the NARCOM agents. At about ten operation, a sale of marijuana has been consummated.
p.m., Amidus co-appellants, Omogbolahan and Bhola, They reported the result of their surveillance to their
arrived at the Las Palmas hotel. They knocked on the commanding officer who instructed them to apply for
door of Room 504. Leangsiri took the black suitcase a search warrant. They were issued a search warrant
and brought it to the dining area of the room where which ordered the search of the premises of Ferdie
appellants stood in full view of NARCOM agents and the seizure of dried marijuana leaves allegedly in
Gapiangao and Balneg and handed it over to the possession of aforesaid person. However, they
appellants. They started to leave the hotel room with were instructed that a buy-bust operation be
the contraband when Agents barged out of the conducted on the accused first before the
washroom and made the arrest. Appellants identified implementation of the search warrant. After the
themselves by presenting their respective passports. operation, they arrested Ferdie. The policemen
Amidu, on the other hand, merely said she was announced that they had with them a search warrant
staying in Room 413 in the same hotel. Accompanied for the search and seizure of marijuana. Before the
by the hotels owner and security officer, appellant raiders proceeded to search the house of the accused,
Amidus room was searched. Tucked within the pages they called for barangay officials in the area to
of her telephone and address book was a piece of witness the conduct of the search. Appellant
paper with the name Suchinda Leangsiri written on questioned his conviction because of the illegality of
it. The paper and Amidu’s other possessions were the search made in his house. He contends that the
confiscated. The trial court convicted appellants, policemen showed the barangay officials the search
finding them guilty of conspiring to transport heroin. warrant which was not shown to the them.
The argument of appellants is that the piece of paper
found in Amidus hotel room should not be admitted to ISSUE: Whether or not the search made in the house
the court because such warrantless search is illegal of the accused was illegal
being made in a place other than the place of arrest.
RULING: The arrest of appellant has been made in
ISSUE: Whether or not the warrantless search made the course of a buy-bust operation, thus, in flagrante
by the agents at Amidu’s room is illegal delicto. A buy-bust operation - a form of entrapment
which has repeatedly been accepted to be a valid
RULING: The Revised Rules of Court provide that (a) means of arresting violators of the Dangerous Drugs
person lawfully arrested may be searched for Law - is far variant from an ordinary arrest. In
dangerous weapons or anything which may be used as lawful arrests, it becomes both the duty and the right
proof of the commission of an offense, without a of the apprehending officers to conduct a warrantless
search warrant. The better and established rule is search not only on the person of the suspect but also
a strict application of the exception provided that is to in the permissible area within his reach, i.e., that
absolutely limit a warrantless search of a person who point which is within the effective control of the
is lawfully arrested to his or her person at the time of person arrested, or that which may furnish him with
and incident to his or her arrest and to dangerous the means of committing violence or of escaping.
weapons or anything which may be used as proof of
the commission of the offense. Such warrantless
Consti II (section 2) 32
This is not yet complete, lacking case digests are the ff: 1.23 People V Evaristo
Facts: Peace officers composed of Sgt. Eladio Romeroso and
13.43-13.45(3 cases) CIC Edgardo Vallarta of Philippine Constabulary together
with Sgt. Daniel Maligaya and 2 other members of the
Integrated National Police were on routine patrol duty in
1.21 PEOPLE V. CHE CHUN TING Barangay III, Mendez, Cavite. At 5:00 in the afternoon, the
officers heard a successive burst of gunfire and they came
FACTS: Standing outside Unit 122, accused handled two
upon Barequiel Rosillo who was firing a gun into the air.
transparent bags of drugs to Mabel Po, in full view of
Seeing the patrol, Rosillo ran to the nearby house of Evaristo
NARCOM agents. Police officers arrested the surprised man prompting the lawmen to pursue him. Upon approaching the
and conducted a search of Unit 122 where they found more immediate perimeter of the house, the patrol chanced upon
bags of shabu so convicted for dispatching in transit and
Evaristo and Carillo. They inquired as to the whereabouts of
having in his possession large amounts of shabu. He contends
Rosillo. The police patrol members were told that he had
that the shabu is inadmissible in evidence as it was seized
already escaped through a window of the house. Vallarta
without a valid search warrant.
noticed a bulge around the waist of Carillo and upon being
frisked he admitted the same to be a revolver. As the patrol
ISSUE: Whether or not the evidence is admissible without a was still in pursuit of Rosillo, Sgt. Romeroso sought
valid search warrant?
Evaristo’s permission to scour through the house which was
granted. Romaroso found a number of firearms and
RULING: No. The search of Unit 122 and the seizure of
paraphernalia supposedly used in the repair and manufacture
drugs found therein are illegal. A warrantless search should
of firearms. Evaristo and Carillo were ound guilty of illegal
be limited to the premises and surroundings that are under the possession of firearms.
immediate control of the accused. As a consequence of the
illegal search, the things seized on the occasion thereof are
Issue: Whether or not the evidence obtained without warrant
inadmissible in evidence under the exclusionary rule. They are
in accidental discovery of evidence is admissible
regarded as having been obtained from a polluted source, the
“fruit of a poisonous tree.” Ruling: Yes, the firearms seized was valid and lawful for
being incidental to a lawful arrest. An offense was committed
in the presence or within the view of an officer, within the
1.22 People V Chi Chan meaning of the rule authorizing an arrest without a warrant.
Facts: Paglicawan and SPO3 Isagani received a radio message
these exceptions is the seizure of evidence in plain view.
from barangay captain of ambil island, looc, oriental Mindoro
saying that there is a suspicious boat in the vicinity of the The records in this case show that Sgt. Romerosa was granted
island and when they responded they saw a fishing boat and a permission by the appellant Evaristo to enter his house. The
speed boat who was transferring cargo to each other, upon
officer’s purpose was to apprehend Rosillo whom he saw had
approachement of the police officer the speed boat sped away
sought refuge therein. Therefore, it is clear that the search for
and was not able to caught except for the fishing boat. Officer
firearms was not Romerosa’s purpose in entering the house,
found shabu inside the boat and asked for its identification
thereby rendering his discovery of the subject as inadvertent
papers but failed to do so. They were apprehended and have and even accidental..
been read their rights. Information was filed against
respondent for violation of dangerous drug act and was found
by the court guilty beyond reasonable doubt. Respondent
contended that the drugs were from the barangay captian and
1.24 People V Tabar
that the search and seizure was invalid. Facts: Respondent-accused was charged, together with her
nephew, for violation of the Dangerous Drugs Act in an
Issue: Whether or not there was unreasonable search and
information, That on or about the 8th day of February 1989, at
seizure and if the warrantless arrest was unjustified as to make
about 3:00 PM. in the City of Cebu, Philippines, and within
the bag inadmissible?
the jurisdiction of this Honorable Court, the said accused,
conniving and confederating together and mutually helping
Ruling: Court ruled that the appellant were actually each other, with deliberate intent, did then and there sell and
committing a crime and were caught by the apprehending deliver, without authority of law, Three (3) sticks of marijuana
officer in flagrante delicto. While acting upon the report of the
cigarettes, a (sic) prohibited drugs, to a person who posted
barangay captain, spotted appellant transferring cargos and the
himself as a buyer, Dangerous Act of 1972. The accused were
speed boat sped away upon closing naturally arouse suspicion,
then convicted of the offense charged against them in the trial
they failed to provide paper and even offered big big lump
court. On appeal, respondent presented her argument that the
sum of money to the officers. lower court erred in admitting the evidence against her when
there wasn’t any search warrant. Therefore, violating the
constitutional guarantee against unreasonable searches and
seizures.

Consti II (section 2) 33
Issue: Whether or not there was a violation against the included in the list of properties to be seized in the search
constitutional guarantee of individuals against unreasonable warrant and were not illegal per se, like explosives and shabu,
searches and seizures. as to justify their seizure in the course of unlawful search.

Ruling: Court ruled there was a valid search and seizure since Issue:Whether the seizure of the Disudrin and Inoflox
he voluntarily submits to a search and consent to have it made products was justified by the plain view doctrine
of his person or premises. The evidence for the prosecution
discloses that the appellant placed the packs of marijuana Ruling:No. Objects, articles or papers not described in the
sticks under the rolled pair of pants which she was then warrant but on plain view of the executing officer may be
carrying at the time she hurriedly left her shanty after noticing seized by him. However, the seizure by the officer of
the arrest of Rommel. When she was asked to spread it out, objects/articles/papers not described in the warrant cannot be
which she voluntary did, the package containing the packs of presumed as plain view. The State must adduce evidence,
marijuana sticks were thus exposed in plain view to the testimonial or documentary, to prove the confluence of the
member of the team. A crime was thus committed in the essential requirements for the doctrine to apply, namely: (a)
presence of the policemen. Pursuant to Section 5, Rule 113 the executing law enforcement officer has a prior justification
and Section 12 Rule 126 of the Revised Rules of Court, she for an initial intrusion or otherwise properly in a position from
could lawfully be arrested and searched for anything which which he can view a particular order; (b) the officer must
may be used as proof of the commission of an offense without discover incriminating evidence inadvertently; and (c) it must
the corresponding arrest and search warrants be immediately apparent to the police that the items they
observe may be evidence of a crime, contraband, or otherwise
subject to seizure.
1.25 Roan v. Gonzales, 145 SCRA 687 The petitioner and the NBI failed to present any of the NBI
Facts:The challenged search warrant was issued by the agents who executed the warrant, or any of the petitioners
respondent judge on May 10, 1984. The petitioner's house was representative who was present at the time of the enforcement
searched two days later but none of the articles listed in the of the warrant to prove that the enforcing officers discovered
warrant was discovered. However, the officers conducting the the sealed boxes inadvertently, and that such boxes and their
search found in the premises one Colt Magnum revolver and contents were incriminating and immediately apparent. It must
18 live bullets which they confiscated and the bases of the be stressed that only the NBI agent/agents who enforced the
charge against the petitioner. warrant had personal knowledge whether the sealed boxes and
their contents thereof were incriminating and that they were
Issue:Whether those seized from the petitioner could have immediately apparent.
been taken even without a warrant?

Ruling:No. Prohibited articles may be seized but only as long


1.27 People v. Doria
as the search is valid. In this case, the Court found out that
Facts:Doria was entrapped and arrested in a buy-bust
there was no valid search warrant. The applicant who asked
operation after the members of the North Metropolitan
for the issuance of the search warrant was based on mere
District, PNP NARCOM, received information from 2 civilian
hearsay and not of information personally known to him, as
informants that one “Jun” was engaged in illegal drug
required by settled jurisprudence. In short, the military officers
activities. During the arrest, the marked bills handed to him as
who entered the petitioner's premises had no right to be there
payment for the marijuana could not be found on him. He
and therefore had no right either to seize the pistol and bullets.
revelead that he left the money at the house of his associate
It does not follow that because an offense is malum
named “Neneth”, whose house he led the police to. The team
prohibitum, the subject thereof is necessarily illegal per se.
found the door of Neneth’s house open and a woman inside,
Motive is immaterial in mala prohibita, but the subjects of this
whom Jun identified as his associate. While asked about the
kind of offense may not be summarily seized simply because
marked bills, a carton box under the dining table was noticed
they are prohibited. A search warrant is still necessary.
by one of the policemen, PO3 Manlangit. The content of
which is wrapped in plastic similar to the marijuana sold to
them. The box contained 10 bricks of what appeared to be
1.26 United Laboratories v. Isip dried marijuana leaves. Simultaneous with the box’s
discovery, was the recovery of the marked bills. Neneth was
Facts:NBI raided the first and second floors of a building for arrested and the box together with its contents and marked
the seizure of finished or unfinished UNILAB products bills were turned over to the investigators at the headquarters.
particularly counterfeit Revicon multivitamins. No fake This was
Revicon multivitamins were found; instead there were boxes
of Disudrin and Inoflox, which were granted by the court to be Issue:Whether the box that contained bricks of marijuana is
turned over to the custody of BFAD for examination. The considered in plain view during the seizure
respondents filed an Urgent Motion to Quash the Search
Warrant or to Suppress Evidence. They asserted that the NBI Ruling:No. Since the warrantless arrest of accused-appellant
officers seized Disudrin and Inoflox products which were not Gaddao was illegal, it follows that the search of her person
Consti II (section 2) 34
and home and the subsequent seizure of the marked bills and
marijuana cannot be deemed legal as an incident to her arrest. 1.29 Hizon v. Court of Appeals
Facts: Fishermen represented by Richard Hizon, were charged
In his direct examination, PO3 Manlangit said that he was sure with illegal fishing penalized under sections 33 and 38 of P.D.
that the contents of the box were marijuana because he himself 704 for using of obnoxious or poisonous substance. The trial
checked and marked the said contents. On cross-examination, court found the 31 petitioners guilty and sentenced them to
however, he admitted that he merely presumed the contents to imprisonment and also ordered the confiscation and forfeiture
be marijuana because it had the same plastic wrapping as the of the F/B Robinson, the 28 sampans and the ton of assorted
"buy-bust marijuana." A close scrutiny of the records reveals live fishes and instruments and proceeds of the offense. They
that the plastic wrapper was not colorless and transparent as to claimed that they are legitimate fishermen of the First
clearly manifest its contents to a viewer. Each of the 10 bricks Fishermen Industries, Inc., a domestic corporation licensed to
of marijuana in the box was individually wrapped in old engage in fishing. They alleged that they catch fish by the
newspaper and placed inside plastic bags-- white, pink or blue hook and line method and that they were intimidated by the
in color. PO3 Manlangit himself admitted on cross- policemen. The trial court found the petitioners guilty of the
examination that the contents of the box could be items other crime of illegal fishing with the use of obnoxious or poisonous
than marijuana. He did not know exactly what the box substance commonly known as sodium cyanide.
contained that he had to ask appellant Gaddao about its
contents. It was not immediately apparent to PO3 Manlangit Issue: Whether the evidence against petitioner in view of the
that the content of the box was marijuana. The marijuana was warrantless search of the fishing boat and the subsequent
not in plain view and its seizure without the requisite search arrest of the petitioners is admissible.
warrant was in violation of the law and the Constitution. It
was fruit of the poisonous tree and should have been excluded Ruling: YES. As a general rule, any evidence obtained
and never considered by the trial court. without judicial warrant is inadmissible for any purpose in any
proceeding. The rule is, subject to certain exceptions which
are: (1) a search incident to a lawful arrest; (2) seizure of
evidence in plain view; (3) search of a moving vehicle; and (4)
1.28 Del Rosario v. People search in violation of customs laws. Search and seizure
Facts: In 1996, the police received a report that Rosario was without search warrant of vessels and aircrafts for violations
in possession of firearms without the necessary licenses. of customs laws have been the traditional exception to the
Police applied for a SW to enable them to search the house of constitutional requirement of a search warrant. It is rooted on
appellant. When they entered, they informed him that they had the recognition that a vessel and an aircraft, like motor
a SW and that they were authorized to search his house. The vehicles, can be quickly moved out of the locality or
search yielded the following items: a caliber .45 pistol, 5 jurisdiction in which the search warrant must be sought and
magazines of 5.56 M-16 rifle and two radios found in the secured. The same exception ought to apply to seizures of
room of the daughter; and a caliber .22 revolver with 8 pieces fishing vessels and boats breaching our fishery laws. The
of live ammunition found in the kitchen of the house. When Court holds as valid the warrantless search on the F/B
asked about his license to possess, he failed to produce any. Robinson, a fishing boat suspected of having engaged in
This prompted the police to seize the firearms. For his illegal fishing. The fish and other evidence seized in the
defense, appellant contends that he had a license for the course of the search were properly admitted by the trial court.
caliber .45 pistol recovered in his bedroom and that the other
items seized were planted by the police officers. Appellant
likewise assails the manner in which the search was carried
out, the officers just barged into his house without asking 1.30 Bagalihog V Fernandez
permission. Furthermore, he claimed that the barangay Facts: Congressman Espinosa, Sr. and his escorts were
officials arrived only after the police already had finished the attacked and killed by a lone assassin. Only Dante Siblante
search. survived although he suffered a gunshot wound. Investigation
of the incident was made. investigator Tandiado, filed an
Issue: Whether or not the seizure of items not mentioned in amended complaint with MTC of Masbate accusing the
the search warrant was illegal. petitioners of the crime multiple murder and frustrated murder
in connection with the incident. After conducting PI, the court
Ruling: No. Seizure is limited to those items particularly issued an order stating that: ". . . after weighing the affidavits
described in a valid search warrant. Searching officers are and answers given by the witnesses for the prosecution during
without discretion regarding what articles they shall seize. the preliminary examination in searching questions and
Evidence seized on the occasion of such an unreasonable answers, concludes that a probable cause has been established
search and seizure is tainted and excluded for being the for the issuance of a warrant of arrest of named accused in the
proverbial "fruit of a poisonous tree." It shall be inadmissible amended complaint, namely
in evidence for any purpose in any proceeding. the firearm
was not found inadvertently and in plain view. It was found as Issue: May a Judge without ascertaining the facts through his
a result of a meticulous search in the kitchen of petitioner's own personal determination and relying solely on the
house. This firearm, to emphasize, was not mentioned in the certification or recommendation of a prosecutor that a
search warrant. Thus, the seizure is illegal. probable cause exists issue a warrant of arrest?
Consti II (section 2) 35
arrival, the agents then waited at the port for the vessel. Upon
Ruling: Court reiterate that in making the required personal arrival of the vessel and when the suspect disembarked, they
determination, a Judge is not precluded from relying on the immediately frisked him and searched his bag which
evidence earlier gathered by responsible officers. The extent contained the marijuana. Subsequently, the Aminnudin was
of the reliance depends on the circumstances of each case and arrested. During the trial, the accused that alleged that he was
is subject to the Judge's sound discretion. However, the Judge arbitrarily arrested and immediately handcuffed and that his
abuses that discretion when having no evidence before him, he bag was confiscated without a search warrant.
issues a warrant of arrest.
Indubitably, the respondent Judge committed a grave error Issue: Is the marijuana found in the accused bag admissible
when he relied solely on the Prosecutor's certification and evidence?
issued the questioned Order dated July 5, 1990 without having
before him any other basis for his personal determination of Ruling: No. The police agents had enough time to secure a
the existence of a probable cause. warrant to arrest and search the accused but did not do so. In
addition to this, the arrest did not fall into any of the
exceptions of a valid warrantless arrest because the accused-
1.31 Aniag jr V comelec appellant was not, at the moment of his arrest, committing a
Facts: Upon the issuance of declaration of gun ban by the crime nor was it shown that he was about to do so or that he
Comelec in connection to the national & local election, the had just done so.
Sgt-at-Arms of the House of Representatives requested
petitioner to return the 2 firearms issued by the House to him.
In compliance, petitioner ordered his driver Arellano to pick
up the firearms in his house to return them to Congress. On his 1.33 People V Malmstedt
way back to the Batasan Complex, Arellano was flagged down Facts: The appellant is sending a package to Switzerland thru
in a check point and police search the car. Upon finding the the packing and export forwarder business owned by Job
guns, he was apprehended and detained and his case was Reyes. Following standard operating procedure, the owner
referred for inquest to the City prosecutor office. Petitioner opened the package for inspection prior to delivery to the
was not made a party to the charge but was invited to shed Bureau of Post/Customs and smelled a peculiar odor from the
light on the incident. Petitioner explained the purpose how box which prompted him to investigate further and found
Arellano came to have the firearms boarded on the car and some dried leaves inside. He took samples and forwarded the
wrote the prosecutor to exonerate Arellano from the charges. content of the shipment to NBI for lab test. He was then
The prosecutor recommended dismissing the case. The investigated by the narcotics division of the NBI and was
Comelec however issued a resolution filing information in requested to bring the package to their office. The lab result
violation of the gun ban against petitioner. Petitioner moves showed the samples were that of marijuana leaves. Thereafter,
for reconsideration to the Comelec which was denied hence NBI took charge of the inventory of the package and an
this petition contending that the search on his car was illegal information was filed against the appellant in violation of RA
and that he was not impleaded as respondent in the 6425 (The Dangerous Drugs Act). It was contended that the
preliminary investigation and his constitutional rights for due court erred in admitting the evidence in a violation of his
process was violated. constitutional rights against illegal search and seizure and
privacy of communication.
Issue: Whether or not the search on his car was illegal?
Issue: Whether or not the evidence obtained should be
Ruling: Yes, An extensive search without warrant could only inadmissible to court?
be resorted to if the officers conducting the search had
reasonable or probable cause to believe there is evidence Ruling: The court held that as a general rule evidence
pertaining to the commission of a crime. However in this case obtained thru illegal and unreasonable search and seizures by
there was no sufficient evidence that would impel the law enforcers and other government agencies are inadmissible
policemen to suspect Arellano to justify the search they have to court. The case at bar however has a peculiar character
conducted, such action constitutes an unreasonable intrusion where the evidence was obtained by a private person without
of the petitioner’s privacy and security of his property. The the intervention of State authorities. In the absence of
manner by which COMELEC proceeded against petitioner government interference, the Constitutional guarantee against
runs counter to the due process clause of the Constitution. unreasonable search and seizure cannot be invoked against the
Thus the court declared the warrantless search and seizure of State. The Bill of Rights provided by the Constitution is not
the firearms as illegal hence inadmissible to court as evidence meant to be invoked against act of private individuals and
in any proceeding against the petitioner. such restraint is only directed against the government and any
of its agencies. Thus the court affirmed that the evidence is
deemed admissible to the case and affirmed the decision of the
1.32 People V Aminuddin lower court finding appellant guilty beyond reasonable doubt.
Facts: The police agents in Iloilo City received a tip from a
reliable informer that the accused, Aminnudin, was on a vessel
bound for Iloilo and is carrying with him marijuana. The said
vessel was to arrive few days after such tip. On the day of the
Consti II (section 2) 36
1.34 People v. Lo Ho Wing 1.36 People v. CFI
Facts: This case involves the transport of shabu by Lo Ho
Wing aka Peter Lo and Lim Cheng Huat aka Antonio Lim, Facts: This case involves the smuggled items apprehended by
from China to Manila. Peter Lo was apprehended by a special government officers in the course of warrantless search and
operations group under the Philippine Constabulary while seizure on a moving light blue dodge car driven by Jessie
riding a taxi cab from the naia. He was brought to the Criminal Hope. Inside the car were 4 boxes which supposedly to be
Investigation Service Headquarters for interrogation. During transported from Angeles City to Manila.
the investigation, the travelling bag of Peter Lo was inspected
and recovered from it are shabu, packed in tea bags. Intelligence officers who apprehended the car driven by Jessie
The trial court convicted Peter Lo and Antonio Lim, applying Hope opened the boxes and recovered wrist watches and
the well-settled presumption of regularity in the performace of bracelets.
official duties.

Issue: Whether the warrantless search and seizure can be Issue: Whether or not the seizure of the merchandise in a
invoked by the convicted Peter Lo and Antonio Lim in their moving vehicle by authorized agents commissioned to enforce
appeals, asserting that they have already planned the arrest 2 customs laws without warrant of seizure breaches the
days before the expected arrival in the airport. constitutional immunity against unreasonable search and
seizure and therefore, such merchandise are inadmissible in
Ruling: The Supreme Court said that the rule on search and evidence.
seizure supported by a valid a warrant is not absolute. There
are 3 exceptions: 1. A search incidental to an arrest; 2. A
search on a moving vehicle; and 3. Seizure of evidence in
plain view. Since the case at bar shows that the search was Ruling: Under the Tariffs and Customs Code, the the
conducted on a moving vehicle, a valid warrant was not authority of persons duly commissioned to enforce tariff and
necessarily. customs laws is quite exceptional when it pertains to the
domain of searches and seizures of goods suspected to have
been introduced in the country in violation of the customs
laws. Said persons were granted to have police power limited
1.35 People v. Saycon to the search and seizure that violated the customs laws.
Facts: Alvaro Saycon, while on board a courier named Doña
Virginia bound for Dumaguete City, he was apprehended by
the coast guards and his bag was opened and recovered from it
is a Marlboro pack containing a pack of shabu. While he was It cannot be said that there is a violation to the right to travel,
searched and arrested, the coast guards did not have a warrant. as long as there is a reasonable cause to believe that
merchandise contained inside a moving vehicle are illegal.
Issue: Whether a valid warrant is necessary in apprehending a
passenger of a sea courier while on board.

Ruling: The general rule is that the search and seizure must be 1.37 People v. Barros
carried out through or with a judicial warrant; otherwise, such
Facts: M/Sgt. Yag-as and S/Sgt. Ayan, boarded to
search and seizure shall be unreasonable. However, the rule is chackchakan, bontok were seated at the back of the bus, they
not absolute. There are exceptions recognized by law. The
saw accused carrying a carton, board the bus and seated
instant case falls under once of the exceptions relating to the
himself on seat No. 18 after putting the carton under his seat.
search on a moving vehicle. Peace officers may lawfully
Thereafter, the bus continued and upon reaching Sabangan,
conduct searches of moving vehicles —automobiles, trucks,
M/Sgt. Yag-as and S/Sgt. Ayan before they alighted, it being
etc. — without need of a warrant, it not being practicable to
their station, called C2C [Fernando] Bongyao to inspect the
secure a judicial warrant before searching a vehicle, since such carton under seat No. 18. After C2C Bongyao inspected the
vehicle can be quickly moved out of the locality or jurisdiction
carton, he found out that it contained marijuana. Accused was
in which the warrant may be sought.
charged with violating Dangerous drug act of 1972 and was
sentenced by the RTC of Bontoc, Barros now appeals from the
In carrying out warrantless searches of moving vehicles, peace
judgement of conviction.
officers are limited to ocular inspection. However, when a
vehicle is stopped and subjected to an extensive search, such a
warrantless search would be constitutionally permissible only
if the officers conducting the search have reasonable or Issue: Whether or not the court deprived him of his right to
probable cause to believe, before the search, that a passenger due process by ignoring manifest absence of the mandatory
or a cargo has illegal contents. warrant in the arrest and search?

Consti II (section 2) 37
Ruling: Court held not all search and seizure are prohibited. emanating from it. PO3 Valenzuela opened one of the boxes
Those which are reasonable are not forbidden. It is determined and saw dried marijuana leaves. He told appellant and co-
according to the facts of each case which in this case a in accused that the contents of the bag were marijuana, which co-
carrying a warrantless arrest it is recognized that in search of a accused readily affirmed. Accused contended that there was a
moving vehicle the officer can do plain view inspection and in violation of their right since they were only riding the taxicab.
order for a intensive search the officer must have probable
cause to conduct such search which Both officers here failed
to do so, they were not given any information or tips from an
informant or manifested that the stench of marijuana inside Issue: Whether or not the search and seizure was valid?
provided them probable cause to search.
Ruling: Confident that they [the accused] have not done
anything wrong, they allowed to be searched." This
declaration of appellant is a confirmation of his intelligent and
1.38 People v. Mustang voluntary acquiescence to the search. The marijuana bricks
were, therefore, obtained legally through a valid search and
Facts: The crime alleged is a violation of sec. 68 of PD No. seizure. They were admissible in evidence; there was no
705 (as amended by E.O. NO. 277). Acting on a tip received, poisonous tree to speak of.
the Special Actions and Investigations Division (SAID) of the
DENR conducted surveillance at petitioner’s lumberyard. In 1.40De Garcia v. Locsin
the course thereof, the team members seize petitioner’s truck,
which was coming out from the lumberyard carrying lauan, Facts: Leona de Garcia’s office were searched and some
almaciga lumber of assorted sizes and dimensions. When packages of records and documents were seized by Mariano
asked for the necessary documents, the driver was unable to Almeda, an agent of the Anti-Usury Board. He conducted the
provide them. Two days later, SAID executed a search search and seizure with a warrant. The search warrant was
warrant. They seized four truckloads of narra shorts, issued upon the affidavit executed by Almeda saying that he
trimmings, and slabs. A day after, the team returned and had a probable cause to believe that Leona de Garcia keeps
placed under administrative seizure the remaining stockpile of and conceals in her house and office certain books, receipts
almaciga, supa, and lauan lumber because petitioner failed to and other pertinent papers relating to her activities as a usurer,
produce upon demand the corresponding documents. The all of which is contrary to the Anti-Usury Law.
Secretary of DENR suspended the permit of petitioner.
Mustang Lumber filed for a TRO against Factoran and
Robles,and questioned the validity of the seizure.
After the seizure, Leona de Garcia on several occasions,
Issue: Whether or not the search and seizure after the demanded from the Anti-Usury Board the return of her
following day was valid? documents. When 6 criminal cases were initiated against her,
she challenged the legality of the search warrant. The trial
Ruling: Yes. The search and seizures made is valid. Search judge denied her motion because there was a waiver on her
the day after was valid since it was a continuation of the part.
search done under and by virtue of the search warrant issued.
Under ROC Rule 126 Sec 9, a search warrant has a lifetime of
10 days. Hence, it could be served.
Issue: Whether there has been a waiver on the part of Leona
de Garcia.

1.39 People v. Lacerna

Facts: (PO3) Carlito P. Valenzuela, a member of the Mobile Ruling: There was implied waiver on her constitutional
Patrol Division of the Western Police District (WPD), immunity against unreasonable search and seizure. She could
appellant and co-accused, who were aboard a taxicab, passed have objected the legality of the search warrant when the
by PO3 Valenzuela's place of assignment, which was then search was conducted without her presence because she was
heavy with traffic, looking suspicious PO3 Valenzuela looked sich, but no objection was made. Since the constitutional right
at the occupants of said taxi, the latter bowed their heads and against unreasonable search and seizure is a personal one, it
slouched, refusing to look at him. Feeling that something was can be waived, expressly or impliedly.
amiss, PO3 Valenzuela and his companion stopped the
vehicle, signalling the driver to park by the side of the road.
PO3 Valenzuela and his co-police officer asked permission to It is well-settled that to constitute a waiver of a constitutional
search the vehicle. As the occupants readily agreed, the police right, it must appear, first, that the right exists; secondly, that
officers went about searching the luggages. They asked the persons involved had knowledge, either actual or
appellant what the contents of the plastic bag were, Co- constructive, of the existence of such right; and, lastly, that
accused NorielLacerna immediately answered that the bag said person had an actual intention to relinquish the right.
contained his vomit. PO3 Carlito found several blocks
wrapped in newspaper, with the distinct smell of marijuana
Consti II (section 2) 38
Issue: Whether there was consent on part of occupant of
hotelroom rented by velasco?
1.41 Caballes v. Court of Appeals
Ruling: Yes, because there was a person inside who gave their
Facts: Police Officers apprehended a jeepney believing that it request. Thus, the unreasonable search may be waived by
may contain something suspicious. The officers told the driver somebody other than the person himself. Therefore such
that they will look at the vehicle. Recovered from it were consent is sufficient in law to dispense with the need for a
cabled wires owned by the National Power Corporation. The search warrant.
driver was convicted of theft.
1.44 People v. Damaso
He then filed a petition for certiorari asserting the illegality of
the search warrant, thus, the evidence obtained shall be Facts: Accsused-appellant charged in an information of
inadmissible. violation of PD 1866 in connection with the crime of
subversion assailed the legality of a search and seizure
Issue: Whether there is consented search when the police conducted at his house at night time when he was not around,
officers did not ask the driver to conduct the search but rather on the ground that it violated constitutional rights against
told him that they will inspect the vehicle. unreasonable search and seizure.
Ruling: The Supreme Court said that the evidence is lacking Issue: Whether or not a search on a house of a person without
that the driver intentionally surrendered his right against the owner’s presence is valid.
unreasonable searches. The manner by which the police
officers obtained consent of the driver for them to conduct the Ruling: No. The search in the dwelling of the accused-
search is not in the manner of asking or requesting, but rather appellant without his knowledge is a violation of the
the manner is imposing upon the driver that they will search constitutional immunity from unreasonable searches and
the vehicle. seizures. The right against unreasonable searches and seizures
is a personal right. The constitutional immunity from
The "consent" given under intimidating or coercive unreasonable searches and seizures, being personal one,
circumstances is no consent within the purview of the cannot be waived by anyone except 1) the person whose rights
constitutional guaranty. are invaded or 2) one who is expressly authorized to do so in
his or her behalf.

1.45 People v. Asis


1.42 People v. Agbot
Facts: Danilo Asis y Fonperada and Gilbert Formento y
Facts: Antonio Agbot was charged and convicted of murder Saricon were charged in an Informationthat the said accused,
for the death of her sister Leona Agbot-Subat. When the police stabbed and robbed one YU HING GUAN @ ROY CHING.
officers went to the scene of the crime, took the “paltik” gun both accused pleaded not guilty. Found to be deaf-mutes
as well as brought him to the station. He admitted the crime "although no witnesses to the actual killing and robbery were
and thus, he was convicted. He appealed his case asserting that presented, the circumstantial evidence including the recovery
there was no search warrant when the gun was taken from the of bloodstained clothing from both accused definitely proved
house and cannot be used as an evidence against him. that the two (2) x x x committed the crime. arguing that the
search was illegally done, making the obtainment of the pair
Issue: Whether or not the seizure was valid? of shorts illegal and taints them as inadmissible. The
prosecution, on the other hand, contends that it was
Ruling: The Supreme Court said that Agbot’s admission is an
Formento's wife who voluntarily surrendered the bag that
express consent. His admission does not constitute a violation
contained the bloodstained trousers of the victim, and thus
of the constitutional guarantee against inadmissibility of
claims that her act constituted a valid consent to the search
illegally seized objects as evidence against an accused. Also,
without a warrant.
the Supreme Court found that the confession itself can only be
supplied by the accused only and the police officers cannot Issue: Whether Formento, a deaf-mute, has given consent to
make up such stories according to their imaginations. the recovery of the bloodstained pair of short, in his
possession during the warrantless search.
1.43 Lopez v. Commissioner of Customs

Facts: The Davao anti-smuggling team apprehended the


vessel MV Jolo Lima which was chatered by Velasco. The Ruling: Primarily, the constitutional right against
MV Jolo lema carried smuggled copra and coffee. The captain unreasonable searches and seizures, being a personal one,
told the team that Velasco was in a hotel. The anti-smuggling cannot be waived by anyone except the person whose rights
team entered the hotel but, Velasco was not there. But, the are invaded or who is expressly authorized to do so on his or
wife of Velasco gave the documents of the smuggled product. her behalf. Because to constitute a valid waiver, it must be
shown that first, the right exists; second, the person involved
had knowledge, actual or constructive, of the existence of such

Consti II (section 2) 39
a right; and third, the person had an actual intention to waived and such waiver may be made either expressly or
relinquish the right. Herein, Formento could not have impliedly.
consented to a warrantless search when, in the first place, he
did not understand what was happening at that moment. There
was no interpreter to assist him -- a deaf-mute -- during the
arrest, search and seizure. 2.23 PEOPLE vs. CORREA

2.21 SPOUSES VEROY v LAYAGUE Facts: The respondents were placed under surveillance on
account of confidential and intelligence reports that
Facts: The Police raided the house of herein petitioners in respondents were engaged in illegal drug trade.The Police
Davao City on information that the said residence was being Operatives from the Drug Enforcement Unit tailed the subject
used as a safehouse of rebel soldiers. They were able to enter jeepney and upon reaching the intersection of Bambang
the yard with the help of the caretakers but did not enter the Extension and Jose Abad Santos Avenue, the operatives
house since the owner was not present and they did not have a stopped and parked their vehicles around the suspect's vehicle
search warrant. The following day they conducted the search and accosted the respondents who were the passengers of the
pursuant to the authority granted by petitioner Ma. Luisa owner-type jeepney. The operatives inspected and able to
Veroy. They recovered a .45 cal. handgun with a magazine, a seized the suspected contrabands. The respondents together
bag etc. The spouse Veroy were held liable for Violation of with their cargo of drugs and their vehicle, were brought to the
Presidential Decree No. 1866 (Illegal Possession of Firearms police station for investigation.
and Ammunitions in Furtherance of Rebellion).
Issue: Whether the evidences presented before the
Issues: Whether the herein petitioner will be held liable for court is inadmissible being the fruit of an illegal search
violation of Presidential Decree No. 1866 following the conducted without any search warrant.
unreasonable searches and seizures in their dwelling.
Held: No. The respondents never protested when
Held: No. The Constitution guarantees the right of the the operative opened the tin can loaded in the appellants'
people to be secure in their persons, houses, papers and effects vehicle and found eight (8) bundles of marijuana. As well
against unreasonable searches and seizures. The rule having they did not raise any protest when they, together with their
been violated and no exception being applicable, the articles cargo of drugs and their vehicle, were brought to the police
seized were confiscated illegally and are therefore protected station for investigation. When one voluntarily submits to a
by the exclusionary principle. They cannot be used as search or consents to have it made on his person or premises,
evidence against the petitioners in the criminal action against he is precluded from later complaining thereof. The right to be
them for illegal possession of firearms. Besides, assuming that secure from unreasonable search may, like every right, be
there was indeed a search warrant, still in mala prohibita, waived and such waiver may be made either expressly or
while there is no need of criminal intent, there must be impliedly.
knowledge that the same existed. Without the knowledge or
voluntariness there is no crime. The criminal case against the
petitioners for illegal possession of firearms is DISMISSED
2.24 PEOPLE vs. RAMOS

Facts: A police team went to the residence of respondent on


2.22 PEOPLE vs OMAWENG the declaration of a drug suspect identifying him as the source
of the of the marijuana leaves. Thereby, arresting him and
Facts: The PC constable put up a checkpoint at the junction immediately brought to the Drugs Enforcement Section of
of the roads, one going to Sagada and the other to Bontoc. WPD Headquarters for investigation. The respondent made an
They stopped and checked all vehicles that went through the admission before the police investigators.
checkpoint. The vehicle was driven by respondent and had no
passengers was stopped and inspected with the permission of Issue: Whether the respondent who made verbal
the latter. A travelling bag was noticed and with the consent of admission before the investigating officers. is not adequately
the respondent to open and see it, it was found to be contained educated to understand fairly and fully the significance of his
forty-one (41) plastic packets of different sizes containing constitutional rights to silence and to counsel be made guilty
pulverized substances of marijuana. of his

Issue: Whether in the case at bar, respondent after Held: No. As mandated, it is not enough that the
voluntarily submits to a search or consented to have it made police investigator merely informs him of his constitutional
waived his the right to be secure from unreasonable search. rights to silence and to counsel, and then taking his statements
down, the interrogating officer must have patience in
Held: Yes. He willingly gave prior consent to the explaining these rights to him. In consonance with Section 20
search and voluntarily agreed to have it conducted on his of the Bill of Rights which states that "any confession
vehicle and travelling bag. Thus, the accused waived his right obtained in violation of this section shall be inadmissible in
against unreasonable searches and seizures. The right to be evidence," We hold that the verbal admissions of appellant
secure from unreasonable search may, like every right, be

Consti II (section 2) 40
during custodial investigation may not be taken in evidence his person or premises, he is precluded from later complaining
against him. thereof . The right to be secure from unreasonable seach may,
like every right, be waived and such waiver may be made
either expressly or impliedly.
2.25 PEOPLE vs TUDTUD
2.27 PEOPLE vs ENCINADA
Facts: A man who identified himself as a police officer
approached the respondent, pointing a .38 caliber revolver. Facts: The police officer received a tip from an informant
The man told him not to run. The Police officer tried to inspect that the respondent would be arriving bringing with him
the plastic bag Tudtud was carrying, and instructed him to “marijuana”. They were not able to secure warrant of arrest
open the bag, which revealed no contrabands. The officer then because the office was already closed. However, they still
directed him to open a carton box some two meters away decided to pursue the apprehension of the appellant. The
which according to him not his, but proceeded to open it out of officers followed the respondent carrying two small colored
fear after the man again pointed his revolver at him. They plastic chairs and boarded a tricycle. The appellant was chased
discovered pieces of dried fish, underneath which was and ordered the driver to stop, they inspect the plastic chairs
something wrapped in cellophane and turned out to be and discovered that a package was place between; tearing the
marijuana. The police thus arrested Tudtud and his package they were convinced that it is marijuana because of
companion, informed them of their rights and brought them to the smell. They apprehended the appellant brought him to the
the police station. The two did not resist. police station and in the presence of a member of the local
media, they opened the package and saw that indeed it
Issue: Whether there was a violation of constitutional rights contains dried leaves of marijuana.
on unreasonable search and seizure in the case at bar.
Issue: Whether the evidence seized during the warrantless
Held: Yes. Unreasonable searches and seizures are the arrest inadmissible?
menace against which the constitutional guarantees afford full
protection. While the power to search and seize may at times Held: Yes. The court ruled acquitting the appellant, it
be necessary to the public welfare, still it may be exercised reiterates the constitutional proscription that evidence seized
and the law enforced without transgressing the constitutional without a valid search warrant is inadmissible in any
rights of the citizens, for the enforcement of no statute is of proceeding. A guild of incriminating evidence will not
sufficient importance to justify indifference to the basic legitimize an illegal search. Indeed, the end never justifies the
principles of government. The officers who are supposed to means. In this case, appellant was not committing a crime in
enforce the law are not justified in disregarding the rights of the presence of the policemen. Moreover, the Lawmen did not
the individual in the name of order. Order is too high a price to have personal knowledge of the facts indicating that the
pay for the loss of liberty. It is simply not allowed in free person to be arrested had committed an offense. The search
society to violate a law to enforce another, especially if the cannot be said to be merely incidental to a lawful arrest. Raw
law violated is the Constitution itself. intelligence information is not sufficient ground for a
warrantless arrest.
2.26 PEOPLE vs TABAR 2.28 PEOPLE vs. ARUTA

Facts: The respondent on her appeal assailed that the Facts: The respondent was crossing the street and was not
marijuana cigarettes seized from her are inadmissible in acting in any manner that would engender a reasonable ground
evidence because they were obtained in violation of the for the NARCOM agents to suspect and conclude that she was
constitutional guarantee against unreasonable search and committing a crime. It was only when the informant pointed to
seizure without warrant. The evidence for the prosecution the respondent and identified her to the agents as the carrier of
discloses that the respondent placed the packs of marijuana the marijuana. Upon inspection of her bag, it was found to
sticks under the rolled pair of pants which she was then contain dried marijuana. The team confiscated the bag and the
carrying at the time she hurriedly left her shanty. When she respondent was then brought to the NARCOM office for
was asked to spread it out, which she voluntary did, the investigation.
package containing the packs of marijuana sticks were thus
exposed in plain view to the member of the officers. Issue: Whether the discovery of marijuana inside the bag of
the respondent by the NARCOM officers justified as seizure
Issue: Whether the respondent can invoke her constitutional of evidence in plain view to effect a valid and legal
guarantee against unreasonable search and seizure without warrantless arrest.
warrant.
Held: No. In the absence of probable cause to effect a valid
and legal warrantless arrest, the search and seizure of
Held: No. Even assuming ex gratia argumenti that the seach respondent bag would also not be justified as seizure of
and seizure were without a warrant, the respondent had evidence in plain view under the second exception. The
effectively waived her constitutional right relative thereto by marijuana was obviously not immediately apparent as shown
voluntarily submitting to the seach and seizure. When one
voluntarily submits to a search and consent to have it made of
Consti II (section 2) 41
by the fact that the NARCOM agents still had to request her to the aircraft and to have his checked-in luggage, if any,
open the bag to ascertain its contents. unloaded. The manager acceded to the request to off-load
Gatward but not to the unloading of his check-in bag as the
plane was about to depart and to do so would unduly delay the
flight. However, Erece made an assurance that the bag would
2.29 PAPA vs MAGO be returned immediately to the Philippines on the first
available flight from Bangkok. The following day, Gatward's
Facts: The head of the counter-intelligence unit of the luggage was brought back to the NAIA from Bangkok through
Manila Police Department, acting upon a reliable information the Thai Airways, pursuant to the request. Upon examination
received that upon orders of the Chief of Police of Manila and of the bag it was observed to contain two cardboard envelopes,
a duly deputized agent of the Bureau of Customs that a certain with a net weight of 5,237.70 grams of heroin.
shipment of personal effects, allegedly misdeclared and
undervalued, would be released from the customs zone of the Issue: Whether the evidence gathered from a warrantless
port of Manila and upon orders of the Chief of Police of search is admissible?
Manila and a duly deputized agent of the Bureau of Customs
seized the load of the two alleged trucks. Held: Yes. Even though no search warrant had been
obtained for that purpose, when appellant checked in his bag
Issue: Whether the police officers policemen had authority as his personal luggage as a passenger of KLM Flight No. 806
to effect the seizure without any search warrant issued by a he thereby agreed to the inspection thereof in accordance with
competent court. customs rules and regulations, an international practice of
strict observance, and waived any objection to a warrantless
Held: Yes. The Tariff and Customs Code does not require search. His subsequent arrest, although likewise without a
said warrant in the instant case. The Code authorizes persons warrant, was justified since it was effected upon the discovery
having police authority under Section 2203 of the Tariff and and recovery of the heroin in his bag, or in flagrante delicto.
Customs Code to stop and search and examine any vehicle,
beast or person suspected of holding or conveying any
dutiable or prohibited article introduced into the Philippines
contrary to law, without mentioning the need of a search 2.32 PEOPLE vs. SUSAN CANTON
warrant in said cases.
Facts: SUSAN was at the Ninoy Aquino International
Airport (NAIA), being a departing passenger bound for
Saigon, Vietnam. The frisker on duty at that time conducted
2.30 PACIS vs PAMARAN the frisking and felt something at Susan’s abdominal, genital
area and thighs packages containing what felt like rice
Facts: The Acting Collector of Customs for the Port of granules. The examiner brought SUSAN to a comfort room
Manila, issued a warrant of seizure and detention for an for a thorough physical examination and was able to collect
automobile owned by respondent, who, according to the the packages that turned out to be a regulated drug, Shabu.
records in his office, had not paid the customs duty collectible
thereon. The respondent invoking what he alleged was a Issue: Whether the strip search conducted on Susan in the
violation of the constitutional provision that only a judge, ladies’ room was constitutionally invalid because it was not
under the 1935 Constitution could issue a search warrant, and "incidental to an arrest."
filed the aforesaid complaint for usurpation.
Held: No. The arrest could not be said to have been made
Issue: Whether the petitioner violated the Constitution and before the search because at the time of the strip search, the
committed usurpation when he issued such warrant of seizure. arresting officers could not have known what was inside the
plastic containers hidden on her body. They could not have
Held: Based on such an assumption and with petitioner determined whether SUSAN was actually committing a crime.
clearly not being a member of the judiciary, there was The strip search was therefore nothing but a fishing
plausibility in the claim that he ran afoul of the penal law. expedition. Verily, it is erroneous to say that she was caught
While the matter was pressed with vigor by counsel for flagrante delicto and that the warrantless search was incidental
respondent, the applicable legal doctrine is on the side of to a lawful arrest.
petitioner. It is a well-settled principle that for violations of
customs laws, the power to issue such a warrant is conceded.
On the undisputed facts and in accordance with the controlling
legal doctrine, no such offense as usurpation of judicial 2.33 PEOPLE vs. JOHNSON
function could have been committed.
Facts: The respondent entered the pre-departure area of the
Bacolod Airport Terminal bound to Manila. Upon passing to a
walk through metal detector machine the red light switched on
2.31 PEOPLE vs. GATWARD and the alarm sounded. It was because of his hand carried box
of piaya. The respondent was ordered to go to the hand-carried
Facts: Customs Police requested manager of the KLM luggage inspection counter where several PASCOM and
airline at the NAIA, to let passenger Gatward disembark from
Consti II (section 2) 42
NARCOM personnel were present. When it was opened with Issue: Whether or not the warantless arrest and search was
his permission, the officers found illegal contrabands. They valid.
apprehended him and brought to the PASCOM office.
Held: An arrest without a warrant may be effected by a
Issue: Whether the PASCOM and NARCOM personnel peace officer or private person, among others, when in his
have no right to conduct search of prohibited materials or presence the person to be arrested has committed, is actually
substances. committing, or is attempting to commit an offense; or when an
offense has in fact just been committed, and he has personal
Held: No. Travelers are often notified through airport knowledge of the facts indicating that the person arrested has
public address systems, signs and notices in their airline committed it.
tickets that they are subject to search and, if any prohibited
materials or substances are found, such would be subject to
seizure. These announcements place passengers on notice that
ordinary constitutional protections against warrantless 2.36 PEOPLE vs SOLAYAO
searches and seizures do not apply to routine airport
procedures. In the case at bar, the PASCOM agents have the Facts: A CAFGU members was conducting an intelligence
right under the law to conduct search of prohibited materials patrol to verify reports on the presence of armed persons
or substances. To simply refuse passengers carrying suspected roaming around the barangays of Caibiran. On their patrol
illegal items to enter the pre-departure area. they met the group of accused-appellant Nilo Solayao
numbering five. The former became suspicious when they
observed that the latter were drunk and that accused-appellant
himself was wearing a camouflage uniform or a jungle suit.
2.34 TERRY vs OHIO Accused-appellant not to run away and introduced himself as
"PC" after which the latter was carrying and found wrapped in
Facts: Terry and two other men were observed by a plain it a 49-inch long homemade firearm locally known as
clothes policeman in what the officer believed to be "casing a "latong". When he asked accused-appellant who issued him a
job, a stick-up." The officer stopped and frisked the three men, license to carry said firearm or whether he was connected with
and found weapons on two of them. Terry was convicted of the same. There upon, the officer confiscated the firearm and
carrying a concealed weapon and sentenced to three years in turned him over to the custody of the policeman of Caibiran
jail. who subsequently investigated him and charged him with
illegal possession of firearm.
Issue: Was the search and seizure of Terry and the other
men in violation of the Fourth Amendment? Issue: Whether the “stop and frisk” conducted by the
officers were justified.
Held: the Court held that the search undertaken by the
officer was reasonable under the Fourth Amendment and that
Held. Yes. In the case at bar constitutes an instance where a
the weapons seized could be introduced into evidence against
search and seizure may be effected without first making an
Terry. Attempting to focus narrowly on the facts of this
arrest. There was justifiable cause to "stop and frisk" accused-
particular case, the Court found that the officer acted on more
appellant. Under the circumstances, the government agents
than a "hunch" and that "a reasonably prudent man would
could not possibly have procured a search warrant first. Thus,
have been warranted in believing [Terry] was armed and thus
there was no violation of the constitutional guarantee against
presented a threat to the officer's safety while he was
unreasonable searches and seizures.
investigating his suspicious behavior." The Court found that
the searches undertaken were limited in scope and designed to
protect the officer's safety incident to the investigation.
2.37 MALACAT vs CA

Facts: allegedly in response to bomb threats Police Officers


was on foot patrol along Quezon Boulevard, Quiapo, Manila,
2.35 POSADAS vs CA near the Mercury Drug store at Plaza Miranda. They chanced
upon two groups of Muslim-looking men, with each group,
Facts: While Pat. Ungab and Umpar were conducting a comprised of three to four men, posted at opposite sides of the
surveillance along Magallanes Street, Davao City, they spotted corner. They mad a stop and frisk and they seized a grenade
petitioner carrying a "buri" bag and they noticed him to be from Malacat. The trial court thus found Malacat guilty of the
acting suspiciously. They approached the petitioner and crime of illegal possession of explosives.
identified themselves as members of the INP. Petitioner
attempted to flee but his attempt to get away was thwarted by Issue: Whether the search made on Malacat is valid, pursuant
the two notwithstanding his resistance. They then checked the to the exception of stop and frisk.
"buri" bag of the petitioner where they found contrabands. The
petitioner was asked to show the necessary license or authority Held: Yes. The general rule as regards arrests, searches and
to possess the firearms and ammunitions but failed to do so. seizures is that a warrant is needed in order to validly effect
the same. The Constitutional prohibition against unreasonable
Consti II (section 2) 43
arrests, searches and seizures refers to those effected without a 1. Warrantless search incidental to a lawful arrest recognized
validly issued warrant, subject to certain exceptions. As under Section 12, Rule 126of the Rules of Court and by
regards valid warrantless arrests, these are found in Section 5, prevailing jurisprudence
Rule 113 of the Rules of Court. A warrantless arrest under the 2. Seizure of evidence in "plain view," the elements of which
circumstances contemplated under Section 5(a) has been are: (a) a prior valid intrusion based on the valid warrantless
denominated as one in flagrante delicto, while that under arrest in which the police are legally present in the pursuit of
Section 5(b) has been described as a hot pursuit arrest. their official duties; (b) the evidence was inadvertently
Turning to valid warrantless searches, they are limited to the discovered by the police who had the right to be where they
following: (1) customs searches; (2) search of moving are; (c) the evidence must be immediately apparent, and (d)
vehicles; (3) seizure of evidence in plain view; (4) consent "plain view" justified mere seizure of evidence without further
searches; (5) a search incidental to a lawful arrest; and (6) a search;
stop and frisk. The concepts of a stop-and-frisk and of a search
incidental to a lawful arrest must not be confused.
2.40 PEOPLE vs SY CHUA

Facts: Police officer s received a report that the


2.38 MANALIL vs. CA respondent will be delivering drugs that night in a
hotel in Angeles. When they spotted the latter
Facts: Police operatives conducted surveillance along the coming out from his car the police officer
front of Kalookan Cemetery based on the information that confronted him and introduced themselves as Police
drug addicts were roaming around in the area, they saw and subjected him to a body search which yielded for some
Manalili who appeared to be high on drugs and introduced illegal contrabands.
themselves as policemen. The latter avoided them and tried to
resist, when they asked what he was holding in his hand, the Issue: Whether there is a valid warrantless arrest.
man held out his wallet and allowed Espiritu to examine it,
who found what he suspected to be crushed marijuana leaves. Held: No.In the case at bar" neither the in flagrante delicto
nor the stop and frisk principles is applicable to justify the
Issue: Whether the evidence seized during a stop-and-frisk warrantless arrest and consequent search and seizure made by
is admissible. the police operatives on accused-appellant. In in flagrante
delicto arrests, the accused is apprehended at the very moment
Held: Yes. Manalili effectively waived the inadmissibility he is committing or attempting to commit or has just
of the evidence illegally obtained when he failed to raise this committed an offense in the presence of the arresting officer.
issue or object during trial. A valid waiver of right against Accordingly, for this exception to apply two elements must
unreasonable searches and seizures require the concurrence of concur 1) the person to be arrested must execute an overt act
these requisites: 1) the right to be waived existed; 2) the indicating that he has just committed, is actually committing,
person waiving it had knowledge; and 3) he/she had actual or is attempting to commit a crime: and 2) such overt act is
intention to relinquish the right. In this case however, it is done in the presence or within the view of the arresting
deemed that Manalili has waived such right for failure to raise officer.
its violation before the trial court, at the earliest opportunity
possible.

2.39 PEOPLE vs ARUTA 2.41 PEOPLE vs. VICTOR COGAED Y ROMANA


Facts: On Dec. 13, 1988, P/Lt. Abello was tipped off by his informant Facts: Accused Victor Cogaed and his companion was
that a certain “Aling Rosa” will be arriving from Baguio City apprehended by the police officer upon the tip from the
with a large volume of marijuana and assembled a team. The next jeepney driver that the two was carrying an illegal
day, at the Victory Liner Bus terminal they waited for the bus coming contrabands. The police officers identified the alleged
from Baguio, when the informer pointed out who “Aling perpetrator through facts that were not based on their personal
Rosa” was, the team approached her and introduced themselves as knowledge.
NARCOM agents. When Abello asked “aling Rosa” about the
contents of her bag, the latter handed it out to the police. They Issue: Whether the stop and frisk principle in the case at bar
found dried marijuana leaves packed in a plastic bag marked to justify the exception of a warrantless search.
“cash katutak”.
Held: No. The search involved in this case was initially a
Issue: Whether the evidence seized during a stop-and-frisk "stop and frisk" search, but it did not comply with all the
is admissible. requirements of reasonability required by the Constitution.
Clearly, a basic criterion would be that the police officer, with
Held: Yes. The following cases are specifically provided or his or her personal knowledge, must observe the facts leading
allowed by law: to the suspicion of an illicit act. In the case at bar, the
assessment of suspicion was not made by the police officer but

Consti II (section 2) 44
by the jeepney driver. It was the driver who signalled to the Facts: Congress passed RA 9165, Comprehensive Dangerous
police that Cogaed was "suspicious." Drugs Act of 2002, and makes it mandatory for candidates for
public office, students of secondary and tertiary schools,
2.42 PEOPLE vs. DE GRACIA officers and employees of public and private offices, and
persons charged before the prosecutor’s office with certain
Facts: The incidents took place at the height of the coup offenses, among other personalities, to undergo a drug test.
d’etat staged bu ultra-rightiest elements headed by the Reform Hence, Senator Pimentel, who is a senatorial candidate for the
the Armed forces Movemnt-Soldiers of the Filipino People 2004 synchronized elections, challenged Section 36(g) of the
(RAM-SFP) against the Government. Rolando de Gracia was said law.
apprehended by the military operatives, holding a C-4 in the
building of Eurocar Office during the raid. No search warrant Issue: Whether RA 9165, sec. 36(g) violates the
was secured by the raiding team because, according to them, Constitutional right of the citizen to a democratic process of
at the time there was so much disorder considering that the election?
neardy Cmp Aguinaldo was being mopped upby the rebel
forces and there was simultaneous firing within the vicinity of Held: Yes. The COMELEC cannot, in the guise of enforcing
Eurocar office and that the courts were consequently closed. and administering election laws or promulgating rules and
regulations to implement Sec. 36(g), validly impose
Issue: Whether the military operatives made a valid search qualifications on candidates for senator in addition to what the
and seizure during the height of the coup d’etat. Constitution prescribes. If Congress cannot require a candidate
for senator to meet such additional qualification, the
Held: Yes.The raid of, and the consequent seizure COMELEC, to be sure, is also without such power. The right
of firearms and ammunition in, the Eurocar Sales Office at the of a citizen in the democratic process of election should not be
height of coup d’etat was held valid, considering the exigent defeated by unwarranted impositions of requirement not
and emergency situation obtaining. The military operatives otherwise specified in the Constitution.
had reasonable ground to believe that a crime was being
committed, and they had no opportunity to apply for a search
warrant from the courts because the latter were closed. Under
such urgency and exigency, a search warrant could be validly 2.45 PEOPLE vs. DE LA CRUZ
dispensed with"
Facts:After receiving a confidential report, a "buy-bust"
2.43 LASERNA vs. DDB operation was conducted by the 13th Narcotics Regional Unit .
At the scene, it was Juan de la Cruz whom an agent first
Facts: the constitutionality of Section 36 of Republic Act negotiated with on the purchase and when the agnt told De la
No. (RA) 9165, otherwise known as the Comprehensive Cruz that he was buying marijuana, De la Cruz instructed
Dangerous Drugs Act of 2002,insofar as it requires mandatory Reynaldo Beltran to give one aluminum foil of marijuana
drug testing of candidates for public office, students of which Beltran got from his pants' pocket and delivered it to
secondary and tertiary schools, officers and employees of the agent. The agent gave the prearranged signal to his
public and private offices, and persons charged before the teammates and effected the arrest of De la Cruz and Beltran.
prosecutors office with certain offenses, among other The marked bill used by the agent was found in the possession
personalities, is put in issue. Petitioner Atty. Manuel J. of Juan de la Cruz together with two aluminum foils and
Laserna, Jr. assiled that the said law is unconstitutional for containing marijuana.
infringing on the constitutional right to privacy, the right
against unreasonable search and seizure, and the right against Issue:Whether the warrantless seizure incidental to the buy-
self-incrimination, and for being contrary to the due process bust operation violates accused’s constitutional rights against
and equal protection guarantees. unreasonable search and seizure.

Issue: Whether RA 9165 violated the right to privacy, the Held: No. A buy-bust operation is the method employed by
right against unreasonable searches and seizure, and the equal peace officers to trap and catch a malefactor in flagrante
protection clause. delicto. While it is conceded that in a buy-bust operation, there
is seizure of evidence from one's person without a search
Held: No. In the case of students, the constitutional warrant, needless to state a search warrant is not necessary, the
viability of the mandatory, random, and suspicionless drug search being incident to a lawful arrest. A peace officer may,
testing for students emanates primarily from the waiver by the without a warrant, arrest a person when, in his presence, the
students of their right to privacy when they seek entry to the person to be arrested has committed, is actually committing or
school, and from their voluntarily submitting their persons to is attempting to commit an offense. It is a matter of judicial
the parental authority of school authorities. In the case of experience that in the arrest of violators of the Dangerous
private and public employees, the constitutional soundness of Drugs Act in a buy-bust operation, the malefactors were
the mandatory, random, and suspicionless drug testing invariably caught red handed.
proceeds from the reasonableness of the drug test policy and
requirement.

2.44 PIMENTEL JR. vs COMELEC 3.21 People v. Doria – GR 125299, Jan. 22, 1999
Consti II (section 2) 45
Facts: Doria was entrapped and arrested in a buy-bust ng halaga ng spare parts, bilihin at and pagpapalaya sa ating
operation after the members of the North Metropolitan pinuno na si Ka Roda hanggang sa magkagulo na.” Later, at
District, PNP NARCOM, received information from 2 civilian a conference at the National Press Club, he called for a
informants that one “Jun” was engaged in illegal drug nationwide strike. He was arrested without a warrant, for
activities. During the arrest, the marked bills handed to him as inciting to sedition.
payment for the marijuana could not be found on him. He
revelead that he left the money at the house of his associate ISSUE: Was the arrest valid?
named “Neneth”, whose house he led the police to. The team
found the door of Neneth’s house open and a woman inside, RULING: Yes. People may differ as to the criminal character
whom Jun identified as his associate. While asked about the of the speech, which at any rate will be decided in court. But
marked bills, a carton box under the dining table was noticed for the purposes of arrest, not conviction, there was sufficient
by one of the policemen, PO3 Manlangit. The content of ground for the officer to believe that Espiritu was in the act of
which is wrapped in plastic similar to the marijuana sold to committing a crime. For purposes of arrest, the law tilts in
them. The box contained 10 bricks of what appeared to be favor of authority. This is an allowable subsequent
dried marijuana leaves. Simultaneous with the box’s punishment of expression because the speech of Espiritu
discovery, was the recovery of the marked bills. Neneth was creates a dangerous tendency, which the state has the right
arrested and the box together with its contents and marked to prevent. There’s a rational connection between the
bills were turned over to the investigators at the headquarters. speech and the evil apprehended.
This was

Issue: Whether the box that contained bricks of marijuana is


3.23 Umil v. Fidel Ramos, GR 81567, July 9, 1990
considered in plain view during the seizure
FACTS:On 1 February 1988, military agents were dispatched
Ruling: No. Since the warrantless arrest of accused-appellant to the St. Agnes Hospital, Roosevelt Avenue, Quezon City, to
Gaddao was illegal, it follows that the search of her person verify a confidential information which was received by their
and home and the subsequent seizure of the marked bills and office, about a "sparrow man" (NPA member) who had been
marijuana cannot be deemed legal as an incident to her arrest. admitted to the said hospital with a gunshot wound. That the
wounded man in the said hospital was among the five (5) male
In his direct examination, PO3 Manlangit said that he was sure "sparrows" who murdered two (2) Capcom mobile patrols the
that the contents of the box were marijuana because he himself day before, or on 31 January 1988 at about 12:00 o'clock
checked and marked the said contents. On cross-examination, noon, before a road hump along Macanining St., Bagong
however, he admitted that he merely presumed the contents to Barrio, Caloocan City. The wounded man's name was listed
be marijuana because it had the same plastic wrapping as the by the hospital management as "Ronnie Javellon," twenty-two
"buy-bust marijuana." A close scrutiny of the records reveals (22) years old of Block 10, Lot 4, South City Homes, Binan,
that the plastic wrapper was not colorless and transparent as to Laguna however it was disclosed later that the true name of
clearly manifest its contents to a viewer. Each of the 10 bricks the wounded man was Rolando Dural. In view of this
of marijuana in the box was individually wrapped in old verification, Rolando Dural was transferred to the Regional
newspaper and placed inside plastic bags-- white, pink or blue Medical Servicesof the CAPCOM, for security reasons. While
in color. PO3 Manlangit himself admitted on cross- confined thereat, he was positively identified by the
examination that the contents of the box could be items other eyewitnesses as the one who murdered the 2 CAPCOM
than marijuana. He did not know exactly what the box mobile patrols.
contained that he had to ask appellant Gaddao about its
contents. It was not immediately apparent to PO3 Manlangit ISSUE: Whether or Not Rolando was lawfully arrested.
that the content of the box was marijuana. The marijuana was
not in plain view and its seizure without the requisite search
warrant was in violation of the law and the Constitution. It
was fruit of the poisonous tree and should have been excluded RULING:Rolando Dural was arrested for being a member of
and never considered by the trial court. the NPA, an outlawed subversive organization. Subversion
being a continuing offense, the arrest without warrant is
justified as it can be said that he was committing as offense
when arrested. The crimes rebellion, subversion, conspiracy or
proposal to commit such crimes, and crimes or offenses
3.22 Espiritu v. General Lim, GR 85727, October 3, 1991 committed in furtherance therefore in connection therewith
constitute direct assaults against the state and are in the nature
of continuing crimes.
FACTS: Espiritu, in a gathering of drivers and sympathizers
at the corner of Valencia St. and Magsaysay Boulevard, said,
among others: “Bukas tuloy ang welga natin, sumagot na ang 3.24 People v. Sucro, 195 SCRA 388
Cebu at Bicol na kasali sila, at hindi tayo titigil hanggang
hindi binibigay ng gobyerno ni Cory ang gusto nating pagbaba

Consti II (section 2) 46
FACTS: Pat Fulgencio went to Arlie Regalado’s house at C. as a buyer. In the instant case, however, the procedure adopted
Quimpo to monitor activities of Edison Sucro (Accused). by the NARCOM agents failed to meet this qualification. The
Sucro was reported to be selling marijuana at a chapel 2 Narcom agents should have secured a valid search warrant
meters away from Regalado’s house. Accused was monitored prior the raid since they have already been conducting
to have talked and exchanged things three times. These surveillance against the accused for quite sometime already
activities are reported through radio to P/Lt. Seraspi. A third and the urgency of their cause of action cannot be justified in
buyer was transacting with appellant and was reported and court. Hence the accused was acquitted.
later identified as Ronnie Macabante. From that moment,
Seraspi proceeded to the area. While the police officers were 3.26 People v. Yap, 229 SCRA 787
at the Youth Hostel in Maagama St. Fulgencio told Seraspi to
intercept. Macabante was intercepted at Mabini and Maagama FACTS: An informant reported to the office of the Station
crossing in front of Aklan Medical Center. Macabante saw the Drug Enforcement Unit, a person referred to by the alias
police and threw a tea bag of marijuana on the ground. Manong, who was allegedly selling shabu at the vicinity of
Macabante admitted buying the marijuana from accused in B.M.B.A.. With this information, Police Chief Senior
front of the chapel. The police team intercepted and arrested Inspector Jose Valencia formed a team to conduct a buy-bust
accused at the corner of C. Quimpo and Veterans. Recovered operation, in which SPO1 Joseph delos Santos was designated
were 19 sticks and 4 teabags of marijuana from a cart inside as the poseur-buyer. That same night, the team proceeded to
the chapel and another teabag from Macabante. the reported area. The informant, upon seeing Manong,
approached the latter and introduced Delos Santos as a
ISSUE: Whether the arrest made without warrant is lawful customer. Delos Santos then told Manong, Pare, pabili ng
piso, and handed him the marked 100-peso bill. Upon receipt
RULING: Search and seizures supported by a valid warrant of the marked money, Manong took out from his pocket and
of arrest is not an absolute rule. Rule 126, Sec. 12 of Rules on handed Delos Santos a plastic sachet containing white
Criminal Procedure provides that a person lawfully arrested crystalline granules. Delos Santos then scratched his left ear,
may be searched from dangerous weapons or anything, which signaling a positive bust. SPO3 Antonio responded to the
may be used as proof of the commission of an offense, without signal and came to the aid of Delos Santos. They frisked
a search warrant. The failure of the police officers to secure a Manong and found four more plastic sachets of white
warrant stems from the fact that their knowledge required crystalline granules on his body. They also recovered the
from the surveillance was insufficient to fulfill requirements marked money from Manong. They then brought Manong to
for its issuance. However, warrantless search and seizures are the police station for investigation. It was only then that the
legal as long as Probable Cause existed. The police officers police learned that Manong is Danilo Jocson, herein accused-
have personal knowledge of the actual commission of the appellant. Accused-appellant Jocson denied the accusations
crime from the surveillance of the activities of the accused. As against him. He testified that on the night of his arrest, he was
police officers were the ones conducting the surveillance, it is at his residence. While watching a late-night television show
presumed that they are regularly in performance of their duties with his mother and his 11-year old niece, SPO3 Antonio
entered his house, and upon seeing him, shouted
3.25 People v. Rodrigueza, 205 SCRA 791 Positive! Thereafter, five other policemen entered the house,
forced accused-appellant out of his bed and handcuffed him.
FACTS: An informer of the Narcotics Division reported
activity on illegal drug trafficking to the authorities and a buy-
bust operation was conducted where the informer successfully ISSUE: WON the arrest was valid without a warrant of arrest?
bought 100 grams of marijuana for P10.00 from the accused.
The authorities immediately conducted a raid and apprehended RULING: In the case at bar, the police arrested accused-
the accused while confiscating marijuana leaves and syringes. appellant in a buy-bust operation. A buy-bust operation is one
The raid however was not authorized by a search warrant. form of entrapment employed by peace officers as an effective
Accused now contends that the court erred in admitting the way of apprehending a criminal in the act of the commission
evidence seized without any search warrant and in violation of of an offense. Entrapment has received judicial sanction when
his constitutional rights. undertaken with due regard for constitutional and legal
safeguards. Where the criminal intent originates in the mind of
the accused and the criminal offense is completed, the fact that
ISSUE: Whether or not evidence obtained without a valid a person, acting as a decoy for the state, or that public officials
search warrant may be used to prosecute the accused. furnished the accused an opportunity for commission of the
offense, or that the accused is aided in the commission of the
crime in order to secure the evidence necessary to prosecute
him, there is permissible entrapment and the accused must be
RULING: The court held that a buy bust operation is a form convicted.
of entrapment employed by peace officers to trap and catch a
malefactor in flagrante delicto. Applied to the case at bar, the
term in flagrante delicto requires that the suspected drug
dealer must be caught red-handed in the act of selling
marijuana or any prohibited drug to a person acting or posing
Consti II (section 2) 47
3.27 People v. Alolod, 266 SCRA 154 prove that he owned the fire arm or that he was licensed to
possess it but instead, he claimed that the weapon was planted
on him at the time of his arrest. He was convicted for violation
of P.D.1866 and was sentenced to reclusion perpetua. In his
FACTS: Respondent was charged with Robbery with appeal he pleads that the weapon was not admissible as
Homicide when he forcibly took from one Romeo a bag, evidence against him because it had been illegally seized and
which contains a lump sum of money and that respondent therefore the fruit of a poisonous tree.
inflicted serious physical injuries, which caused Romeo’s
death. He argued that there was a commotion going on and ISSUE: Whether or not the warrantless search and arrest was
that he was only a by-passer, after hearing some gunshots, illegal.
people panicked and so did he. Several by-passers claimed
that he was the one who fired the gun and immediately the RULING: An evidence obtained as a result of an illegal
police apprehended him. Upon examination, he demanded to search and seizure inadmissible in any proceeding for any
subject his hands to x-ray to determine if he indeed fired a gun purpose as provided by Art. III of the Constitution. Rule 113
or not, but was denied. Convicted of the crime, since evidence sec.5 of the Rules of Court, provides arrest without warrant
shows that police seized a gun, which was used in the lawful when: (a) the person to be arrested has committed, is
commission of the crime. In addition, the procedure followed actually committing, or is attempting to commit an offense, (b)
by the police officers in effecting the arrest of the accused and when the offense in fact has just been committed, and he has
in procuring evidence against him. The accused argues personal knowledge of the facts indicating the person arrested
that: (a) the police officers did not observe his constitutional has committed it and (c) the person to be arrested has escaped
rights in effecting his arrest and while under custodial from a penal establishment or a place where he is serving final
interrogation; (b) they did not subject the gun taken from him judgment or temporarily confined while his case is pending, or
as well as the slug recovered from the body of the victim to a has escaped while being transferred from one confinement to
ballistics test to determine if indeed the slug came from the another.
gun; and, (c) despite his request, they did not bother to subject
him to a paraffin test to determine if he recently fired the gun.

ISSUE: WON there was a valid arrest?


3.29 People v. Elamparo – 329 SCRA
RULING: Court ruled that contrary to the argument of the
defense, the records fail to disclose that there were violations
by the police of the Constitutional rights of the accused during
his custodial interrogation. We hold that there was no more FACTS: Police Officers conducted a buy-bust operation on
need for any ballistics test on the gun recovered from the the sale of shabu or marijuana by themselves without a
accused and the slug retrieved from the body of the victim; warrant. When the police officers ran after the seller of illegal
neither was it necessary to conduct a paraffin test to determine drugs named Erwin Spencer, the latter ran with the marked
if the accused indeed fired a gun. money towards a bungalow house where the police officers,
when they got inside, saw Joel Elamparo packing bricks of
dried marijuana leaves with a newspaper.

3.28 People v. Mengote – 210 SCRA 174


The 2 were arrested and taken to the precinct to the inquest
fiscal for further investigation. They charged of illegal
FACTS: The Western Police District received a telephone call possession of drugs and Elamparo was convicted with penalty
from an informer that there were three suspicious looking of reclusion perpetua and a P9-million fine.
persons at the corner of Juan Luna and North Bay Boulevard
in Tondo, Manila. A surveillance team of plainclothesmen was
forthwith dispatched to the place. The patrolmen saw two men
looking from side to side, one of whom holding his abdomen. ISSUE:
They approached the persons and identified themselves as
policemen, whereupon the two tried to run but unable 1. W/N the arrest was invalid considering that the police
to escape because the other lawmen surrounded them. The officers had no warrant with them.
suspects were then searched. One of them the accused-
appellant was found with a .38 caliber with live ammunitions
RULING:
in it, while his companion had a fan knife. The weapons were
taken from them and they were turned over to the police 1. As to the warrantless search, Section 2 Article III of
headquarters for investigation. An information was filed the 1987 Constitution prohibits a search and seizure
before the RTC convicting the accused of illegal possession of without a judicial warrant. Section 3 also provides
firearm arm. A witness testified that the weapon was among that any evidence obtained without such warrant is
the articles stolen at his shop, which he reported to the police inadmissible for any purpose in any proceeding.
including the revolver. For his part, Mengote made no effort to
Consti II (section 2) 48
However, not being absolute, the right against *There must be immediacy between the time the offense is
unreasonable searches and seizures is subject to committed and the time of the arrest.
exceptions.

This case falls squarely under the plain view doctrine.


3.30 Rizaldy Cajili v. People, GR 204589
Reiterated the case of People v Doria (1999): "Objects
falling in plain view of an officer who has a right to be in
the position to have that view are subject to seizure even
without a search warrant and may be introduced in FACTS: Accused was found to be in possession of “shabu”,
evidence. The "plain view" doctrine applies when the upon arraignment, accused pleaded not guilty to the offense
following requisites concur: (a) the law enforcement charged. However, RTC ruled that accused was caught in
officer in search of the evidence has a prior justification flagrante delicto, in actual possession of shabu. Accused
for an intrusion or is in a position from which he can view contended that the searched was done without a valid warrant
a particular area; (b) the discovery of the evidence in plain and the shabu seized must be inadmissible as evidence before
view is inadvertent; (c) it is immediately apparent to the the court.
officer that the item he observes may be evidence of a
crime, contraband or otherwise subject to seizure. The law
enforcement officer must lawfully make an initial
ISSUE: WON the accused was caught in flagrante delicto,
intrusion or properly be in a position from which he can
hence, a warrant is not needed.
particularly view the area. In the course of such lawful
intrusion, he came inadvertently across a piece of
evidence incriminating the accused. The object must be
open to eye and hand and its discovery inadvertent." RULING: The Court ruled that there was no warrant needed
since the accused was found to be in flagrante delicto on the
basis of the plain-view doctrine.
When Spencer wrenched himself free from the grasp of
PO2 Gaviola, he instinctively ran towards the house of
appellant. The members of the buy-bust team were 3.31 Go v. CA – 206 SCRA 138
justified in running after him and entering the house
without a search warrant for they were hot in the heels of
a fleeing criminal. Once inside the house, the police
officers cornered Spencer and recovered the buy-bust FACTS: Rolito Go was driving and upon nearly bumping
money from him. They also caught appellant another car driven by Eldon Maguan, Rolito stepped out from
in flagrante delicto repacking the marijuana bricks his car, approached Eldon’s car and shot Eldon with a 9 mm
which were in full view on top of a table. caliber pistol. Rolito went back to his car and drove away. A
security guard nearby the incident was able to witness the
In flagrante delicto arrest is lawful, provided that the 2 alleged crime and was able to take note of the plate number of
elements have concurred: (1) the person to be arrested the car of the assailant. The police shortly arrived thereafter.
must execute an overt act indicating the he has just The car plate number was verified at the LTO showing that the
committed, is actually committing, or is attempting to car driven by the assailant was registered to Elsa Ang Go.
commit a crime; and (2) such overt act is done in the
presence or within the view of the arresting officer. Thus, The following day, the police officer launched a manhunt for
when appellant was seen repacking the marijuana, the Rolito Go, after the Security Guard has given his statement
police officers were not only authorized but also duty- and confirmed the assailant was the petitioner.
bound to arrest him even without a warrant.
6 days later, Rolito Go appeared in the Police Station with his
2 lawyers, to confirm the news that he was being hunted by
the police. He was detained and was also charged with
frustrated homicide by the police on that same day.

B. Hot Pursuit Rolito Go questioned right away the validity of the warrantless
arrest to be unlawful. Hence, the petition.

Two Requisites:
ISSUE: W/N the arrest is lawful considering that the accused
1. An offense had just been committed. personally appeared in the police station 6 days after the
alleged shooting incident and was detained and arrested on
2. The person making the arrest has probable cause to believe, that same day without a warrant.
based on his personal knowledge of facts and circumstances,
that the person to be arrested committed it.
Consti II (section 2) 49
warrant and did not inform the accused Manlulu of his right to
counsel. Manlulu admitted the commission of the crime.
RULING: The instant case does not fall within the terms of
Section 5 of Rule 113 of the 1985 Rules on Criminal
Procedure, which provides as follows:
ISSUE: W/N the convicted Manlulu and Samson can
challenged the validity of the warrantless arrest considering
that the arrest took place 19 hours from the time of the alleged
Sec. 5 Arrest without warrant; when lawful. — A peace commission of the crime [the time of the commission of the
officer or a private person may, without warrant, arrest a crime was 1 AM, May 30, 1986 and the time of the
person: warrantless arrest was 7 PM, May 30, 1986].

(a) When, in his presence, the person to be arrested has RULING: Yes. The SC said that the warrantless arrest in this
committed, is actually committing, or is attempting to commit case cannot be regarded as lawful within the meaning of Sec 5
an offense; (b) Rule 113 of the Criminal Procedure which provides: that
the arresting officer must have "personal knowledge" of
(b) When an offense has in fact just been committed, and he an offense which "has in fact just been committed." The
has personal knowledge of facts indicating that the person to arresting officer does not have personal knowledge since
be arrested has committed it; and “personal gathering of information” is different from
“personal knowledge”, and the rule requires that the arrest
(c) When the person to be arrested is a prisoner who has immediately follows the commission of the offense, not some
escaped from a penal establishment or place where he is nineteen hours later.
serving final judgment or temporarily confined while his case
is pending, or has escaped while being transferred from one
confinement to another.
However, inspite of the nullification of the arrest of Manlulu
In cases falling under paragraphs (a) and (b) hereof, the person and the exclusion of real evidence, the SC affirmed that the
arrested without a warrant shall be forthwith delivered to the prosecution was able to prove the guilt of the accused beyond
nearest police station or jail, and he shall be proceed against in reasonable doubt. After all, the illegality of the warrantless
accordance with Rule 112, Section 7. arrest cannot deprive the state of its right to prosecute the
guilty when all other facts on record point to their culpability.
Petitioner's "arrest" took place six (6) days after the shooting
of Maguan. The "arresting" officers obviously were not
present, within the meaning of Section 5(a), at the time
petitioner had allegedly shot Maguan. Neither could the 3.33 People v. Rodrigueza, 205 SCRA 791 (1992)
"arrest" effected six (6) days after the shooting be reasonably
regarded as effected "when [the shooting had] in fact just been
committed" within the meaning of Section 5(b). Moreover,
none of the "arresting" officers had any "personal knowledge" FACTS: NARCOM agents staged a buy-bust operation, after
of facts indicating that petitioner was the gunman who had gaining information that there was an ongoing illegal traffic of
shot Maguan. The information upon which the police acted prohibited drugs in Tagas, Albay. The participating agents
had been derived from statements made by alleged were given money treated with ultraviolet powder. One of the
eyewitnesses to the shooting — one stated that petitioner was agents went to said location, asked for a certain Don.
the gunman; another was able to take down the alleged Thereafter, the Don, herein accused, met with him and “a
gunman's car's plate number, which turned out to be registered certain object wrapped in a plastic” later identified as
in petitioner's wife's name. That information did not, however, marijuana was given in exchange for P200. The agent went
constitute "personal knowledge." back to headquarters and made a report, based on which, a
team was subsequently organized and a raid was conducted in
the house of the father of the accused. During the raid, the
NARCOM agents were able to confiscate dried marijuana
3.32 People v. Manlulu, 231 SCRA 701 (1994) leaves and a plastic syringe among others. There was no
authorization by any search warrant. The accused was found
positive of ultraviolet powder. The lower court, considering
the evidences obtained and testimonies from the prosecution,
FACTS: A man (Alfaro) was allegedly stabbed and shot in a found him guilty of violating the Dangerous Drugs Act.
drinking spree by Rolando Manlulu and Dante Samson at
around 1 am of May 30, 1986. The assailants fled. Manlulu
was arrested the same day, around 7 pm, by the police officers
recovering from him the pistol used to shoot the victim and a
wristwatch owned by the victim, but the police officers had no

Consti II (section 2) 50
ISSUE: Whether or Not the lower court was correct in its factors that are considered in fixing a reasonable amount of
judgment. bail. However, it is premature for the Court to fix the amount
of bail without an anterior showing that the evidence of the
guilt against accused is not strong. Thus, bail denied.

RULING: The NARCOM agents’ procedure in the


entrapment of the accused failed to meet the qualification that
the suspected drug dealer must be caught red-handed in the act 3.35 People v. Jayson, 282 SCRA 166 (1997)
of selling marijuana to a person posing as a buyer, since the
operation was conducted after the actual exchange. Said raid
also violated accused’ right against unreasonable search and
seizure, as the situation did not fall in the circumstances FACTS: Jayson was a bouncer at a nightclub. He was arrested
wherein a search may be validly made even without a search without a warrant after being pointed by an eyewitness as the
warrant, i.e. when the search is incidental to a lawful arrest; gunman in the killing of Jordan. Recovered from him was a
when it involves prohibited articles in plain view. The gun which was issued by Major Arquillano. He was
NARCOM agents could not have justified their act by subsequently charged with illegal possession of firearm.
invoking the urgency and necessity of the situation because However, accused did not raise any error pertinent to consider
the testimonies of the prosecution witnesses reveal that the the circumstances surrounding accused’s arrest and the seizure
place had already been put under surveillance for quite some from him of the firearm in question considering that both were
time. Had it been their intention to conduct the raid, then they made without any warrant.
should, because they easily could, have first secured a search
warrant during that time.Failure to do so would be fatal to the
cause of the prosecution. Conviction is reversed and set aside
ISSUE: WON the warrantless arrest, search and seizure were
and accused is acquitted.
valid.

RULING: Yes. Rule 113, Sec. 5(b) of the Revised Rules of


3.34 People v. Enrile, 222 SCRA 586 Criminal Procedure provides that a warrantless arrest shall be
lawful when an offense has in fact just been committed, and
he has personal knowledge of facts indicating that the person
to be arrested has committed it. In the case at bar, the arresting
FACTS: Ombudsman charged Enrile and several others with officers acting on the basis of their personal knowledge of the
plunder in the Sandiganbayan on the basis of their purported death of the victim and of facts indicating that Jayson was the
involvement in the diversion and misuse of appropriations assailant. The arrest, search and seizure were incident to a
under the Priority Development Assistance Fund (PDAF). lawful one as allowed under Rule 126, Sec. 12.
Enrile respectively filed his Omnibus Motion and
Supplemental Opposition that he be allowed to post bail
should probable cause be found against him. Sandiganbayan
denied Enrile’s motion on the ground of its prematurity 3.36 People v. Del Rosario, GR 127755, April 14, 1999
considering that he had not yet voluntarily surrendered or been
placed under the custody of the law. Subsequently, warrant for
his arrest was issued, however, Enrile voluntarily surrendered. FACTS: Joselito del Rosario, Ernesto Marquez, Virgilio
Thereafter, Enrile filed his Motion for Detention and to Fix Santos and John Doe were charged with special complex
Bail, heard by the Sandiganbayan but was denied because it is crime of robbery with homicide, for having robbed a 66-year
only after the prosecution shall have presented its evidence old businesswoman, of P200k cash and jewelry and on the
and the Court shall have made a determination that the occasion, shot and killed her.
evidence of guilt is not strong against accused can he demand
bail as a matter of right. Then and only then will the Court be
duty-bound to fix the amount of his bail.
2 man snatched the bag of the old woman. One ran after
another man who tried to help the woman, the other who was
armed with a pistol kicked the woman onto the ground and
ISSUE: WON the contention of the Sandiganbayan in shot her. The armed man brought the bag to another man
denying the accused for fixing bail correct? waiting inside a tricycle and sat behind the tricycle driver
while the other entered the side car. The driver of the tricycle
was Joselito del Rosario.
RULING: For purposes of bail, admittedly, the accused’s age,
physical condition and his being a flight risk are among the
Consti II (section 2) 51
An eye witness (Alonzo) saw the incident and tried to chase have personal knowledge of certain facts indicating that the
the tricycle but the tricycle sped away. He was able to get the person to be taken into custody has committed the crime.
plate number and went to report to the Police station right
away.

The following day, a raid was conducted by the police officers


at the house of Ernesto Marquez. They were arrested,
including del Rosario, without a warrant.

Accused del Rosario countered that he was hired by 3 men and 3.37 People v. Samus, GR 135957, 2002
during the occasion he wanted to leave but he was threatened,
thus, his defense as an exempting circumstance was
“irresistible force” and “uncontrollable fear”.
FACTS: Guillermo Samus was charged with murder for the
The trial court still convicted them including del Rosario. death of a 61-year old woman named Dedicacion Soriano-
Hence, the case for certiorari. Balisi and of 6-year old boy named John Ardee Balisi by
strangling their neck and banging their heads against the
concrete pavement of the floor.

ISSUE: w/n the arrest of del Rosario and the other 3 men falls
under the lawful warrantless arrest provided in Sec 5 Rule 113
of the Criminal Procedures. The alleged crime took place in September 2, 1996, and the
arrest of Guillermo Samus was on September 10, 1996, by
police officers and without a warrant.

RULING: No. The instant case does not fall under the lawful
warrantless arrest.
ISSUE: Is there an unlawful warrantless arrest?
Sec 5(a) requires that the accused be caught in flagrante
delicto or caught immediately after the consummation of the
act. The arrest of del Rosario is obviously outside the purview
RULING: Yes. Under the Rules,peace officers may, without a
of the aforequoted rule since he was arrested on the day
warrant, arrest a person under any of these circumstances: (a)
following the commission of the robbery with homicide.
when, in their presence, the person to be arrested has
Sec 5(b) necessitates that: 1. An offense was just committed; committed, is actually committing, or is attempting to commit,
and, 2. the arresting officer has personal knowledge of facts an offense; (b) when an offense has just been committed, and
indicating that a person to be arrested had committed it. The they have probable cause to believe, based on personal
arrest of del Rosario does not comply with these requirements knowledge of facts or circumstances, that the person to be
since: 1. the arrest came a day after the consummation of the arrested has committed it; and (c) when the person to be
crime and not immediately thereafter, and 2. the arresting arrested is a prisoner who has escaped while being transferred
officers had no personal knowledge of facts indicating that the from one confinement to another, or from a penal
person to be arrested had committed the offense since they establishment where he or she is serving final judgment or is
were not present and were not actual eyewitnesses to the temporarily confined while the case is pending.
crime.

None of these circumstances was present when members of


However, the objection to a valid warrantless arrest was the Criminal Investigation Group (CIG) arrested appellant. He
deemed waived in this case, because the accused submitted to was not a prisoner. The killing of Dedicacion and John Ardee
the arraignment without objection. Balisi was not done in the presence of the arresting
officers. Since it took place on September 2, 1996, it could not
The SC affirmed the conviction of the other 3 men, but have been considered as having just been committed.
acquitted del Rosario from the crime. Evidently, they unlawfully arrested appellant on September
10, 1996.

MAINPOINT: In Sec 5(b) Rule 113 of the criminal procedure,


there must be a large measure of immediacy between the time 3.38 People v. Cubcubin, GR 136267, October 2, 2001
the offense was committed and the time of the arrest, and if
there was an appreciable lapse of time between the arrest and
the commission of the crime, a warrant of arrest must be
secured. Aside from the sense of immediacy, it is also
mandatory that the person making the arrest must
Consti II (section 2) 52
FACTS: A tricycle driver named Henry Piamonte was shot incident took place. There they found a piece of wood with
and killed while he was riding his tricycle around 3 AM, blood stains, a hollow block and two roaches of marijuana.
August 26, 1997. They were informed by Reyes, that she saw the killing and she
pointed toGabriel Gerente as one of the three men who killed
Clarito.
The policemen proceeded to the house of the appellant who
He was last seen in a Café, located a kilometer away from the was then sleeping. They told him to come out of the house and
crime scene, and he was last seen together with Fidel they introduced themselves as policemen. Patrolman Urrutia
Cubcubin Jr. Fidel Cubcubin Jr. was arrested by the police friskedappellant and found a coin purse in his pocket which
officers around 4 am on the same day the alleged crime was contained dried leaves wrapped in cigarette foil. Only
committed, at his house, with no warrant. The police officers the appellant, GabrielGerente, was apprehended by the police.
also seized a shirt, empty shells of a paltik gun. The other suspects, Fredo and Totoy Echigoren, are still at
large.
Two separate informations were filed by Assistant Provincial
Prosecutor Benjamin Caraig against him for Violation of
ISSUE: Does the warrantless arrest of Fidel Cubcubin Jr. fall
Section 8, Article II, of Republic Act No. 6425, and for
under the meaning of Sec 5 (b) Rule 113 of the criminal
Murder. The trial court convicted him of Violation of Section
procedure, making it a lawful one, after consideration that the
8 of R.A. 6425 and of Murder.
arrest was made within the same day the alleged crime was
committed?
ISSUE: Whether or not the court erred in admitting the
marijuana leaves adduced in evidence by the prosecution

RULING: The SC held that the warrantless arrest made by the RULING: YES. The search conducted on Gerente's person
police officers is an illegal one. Sec 5(b) Rulle 113 requires 2 was likewise lawful because it was made as an incident to a
conditions to concur for a warrantless arrest to be valid: 1. The valid arrest. This is in accordance with Section 12, Rule 126 of
offense has just been committed, and 2. The arresting officer the Revised Rules of Court which provides: A person lawfully
has personal knowledge that the person to be arrested has arrested may be searched for dangerous weapons or anything
committed it. IN the case at bar, although the arrest was which may be used as proof of the commission of an offense,
effected shortly after the victim was killed, but the arresting without a search warrant.
officers had no personal knowledge since they were not
present when the victim was killed and since their information The frisk and search of appellant's person upon his arrest was
came entirely from an informant. a permissible precautionary measure of arresting officers to
protect themselves, for the person who is about to be arrested
may be armed and might attack them unless he is first.

3.40 Padilla v. CA, GR 121917, March 12, 1997


3.39 People v. Gorente, 219 SCRA 756

FACTS: One night, Enrique Manarang noticed the accused


FACTS: Edna Edwina Reyes testified that Gabriel Gerente, appellant’s car running fast. After a while, a screech of tires
together with Fredo Echigoren and Totoy Echigoren, started was heard and thus, made the officer run out and investigate.
drinking liquor andsmoking marijuana in the house of Not so long, the car continued to run, so a hot-pursuit took
the appellant. She overheard the three men talking about their place. Manarang then radioed the incident to the Police.
intention to kill Clarito Blace. Fredo, Totoy Echigoren and When the car was put to a stop, the driver rolled down the
Gerente carried out their plan to kill Clarito Blace . Reyes, windows with his hands raised. The officers then noticed that
testified that she witnessed the killing as follows: Fredo it was the famous actor, Robin Padilla. While apprehended,
Echigoren struck the first blow against Clarito Blace, followed because of the hit-and-run incident, the police saw the
by Totoy Echigoren and Gabriel Gerente who hit him twice revolver tucked in the left waist of Robin. So, the police
with a piece of wood in the head and when he fell, Totoy insisted that the gun be shown in the office if it was legal. The
Echigoren dropped a hollow block on the victim's head. crowd had formed and Robin was shaking their hands and
Thereafter, the three men dragged Blace to a place behind the pointing to the police while saying “iyan kinuha ang baril ko”,
house of Gerente.Patrolman Jaime Urrutia of the Valenzuela as if it was in the movies. The gesture then revealed a
Police Station received a report from the Palo Police magazine clip of a rifle which made the police suspect that
Detachment about a mauling incident. He went to the there is a rifle inside the vehicle. Then the rifle was seen. The
Valenzuela District Hospital where the victim was brought. He other firearms were voluntarily surrendered by Robin.
was informed by the hospital officials that the victim died on Now, Robin’s defense was that his arrest was illegal and
arrival. The cause of death was massive fracture of the skull consequently, the firearms and ammunitions taken in the
caused by a hard and heavy object. Right away, Patrolman course thereof are inadmissible in evidence under the
Urrutia, proceeded to Paseo de Blas where the mauling exclusionary rule.

Consti II (section 2) 53
Robin Padilla was arrested, tried, and convicted for illegal Masamlok, they had neither search nor arrest warrant with
possession of firearms. He was in possession of a .357 caliber them is unlawful.
revolver, Smith and Wesson with 6 live ammunitions, One M- The trial court has erred in its conclusion that said
16 baby Armalite Rifle with ammunitions, One .380 Pietro warrantless arrest is under the ambit of aforementioned RoC.
Barreta with 8 live ammunitions, and six live double action At the time of defendant’s arrest, he wasn’t in actual
ammunitions of .38 caliber revolver. possession of any firearm or subversive document, and was
not committing any “subversive” act,he was plowing his
ISSUE: Whether or not the arrest is illegal and the evidences field. It is not enough that there is reasonable ground to
is inadmissible. believe that the person to be arrested has committed a
crime in a warrantless arrest. An essential precondition is
RULING: NO.The policemen's warrantless arrest of that a crime must have been in fact or actually have been
petitioner could likewise be justified as he had in fact just committed first; it isn’t enough to suspect a crime may
committed an offense. There was no supervening event or a have been committed. The test of reasonable ground applies
considerable lapse of time between the hit and run and the only to the identity of the perpetrator.
actual apprehension. Moreover, after having stationed
themselves at the Abacan bridge in response to Manarang's 3.42 People v. Sucro – 195 SCRA 388
report, the policemen saw for themselves the fast approaching
Pajero of petitioner, its dangling plate number (PMA 777 as
reported by Manarang), and the dented hood and railings
thereof. These formed part of the arresting police officer's FACTS: Pat Fulgencio went to Arlie Regalado’s house at C.
personal knowledge of the facts indicating that petitioner's Quimpo to monitor activities of Edison Sucro (Accused).
Pajero was indeed the vehicle involved in the hit and run Sucro was reported to be selling marijuana at a chapel 2
incident. Verily then, the arresting police officers acted upon meters away from Regalado’s house. Accused was monitored
verified personal knowledge and not on unreliable hearsay to have talked and exchanged things three times. These
information. activities are reported through radio to P/Lt. Seraspi. A third
buyer was transacting with appellant and was reported and
3.41 People v. Burgos – 144 SCRA 1 later identified as Ronnie Macabante. From that moment,
Seraspi proceeded to the area. While the police officers were
at the Youth Hostel in Maagama St. Fulgencio told Seraspi to
intercept. Macabante was intercepted at Mabini and Maagama
FACTS: Defendant is charged with illegal possession of crossing in front of Aklan Medical Center. Macabante saw the
firearm in furtherance of subversion (tasks such as recruiting police and threw a tea bag of marijuana on the ground.
members to the NPA and collection of contributions from its Macabante admitted buying the marijuana from accused in
members) and found guilty by the RTC of Digos, Davao del front of the chapel. The police team intercepted and arrested
Sur. accused at the corner of C. Quimpo and Veterans. Recovered
were 19 sticks and 4 teabags of marijuana from a cart inside
From the information filed by the police authorities upon the the chapel and another teabag from Macabante.
information given by Masamlok, allegedly a man defendant
tried to recruit into the NPA, the police authorities arrest
defendant and had his house searched.
RULING: Search and seizures supported by a valid warrant
Subsequently, certain NPA-related documents and a firearm, of arrest is not an absolute rule. Rule 126, Sec. 12 of Rules on
allegedly issued and used by one Alias Cmdr. Pol of the NPA, Criminal Procedure provides that a person lawfully arrested
are confiscated. Defendant denies being involved in any may be searched from dangerous weapons or anything, which
subversive activities and claims that he has been tortured in may be used as proof of the commission of an offense, without
order to accept ownership of subject firearm and that his a search warrant. The failure of the police officers to secure a
alleged extrajudicial statements have been made only under warrant stems from the fact that their knowledge required
fear, threat and intimidation on his person and his family. from the surveillance was insufficient to fulfill requirements
for its issuance. However, warrantless search and seizures are
He avers that his arrest is unlawful as it is done without valid legal as long as Probable Cause existed. The police officers
warrant, that the trial court erred in holding the search warrant have personal knowledge of the actual commission of the
in his house for the firearm lawful, and that the trial court crime from the surveillance of the activities of the accused. As
erred in holding him guilty beyond reasonable doubt for police officers were the ones conducting the surveillance, it is
violation of PD 9 in relation to GOs 6and 7. presumed that they are regularly in performance of their
duties.
ISSUE: If defendant’s arrest, the search of his home, and the
subsequent confiscation of a firearm and several NPA-related
documents are lawful.
3.43 People v. Briones – 202 SCRA 708
RULING:NO. Records disclose that when the police went to
defendant’s house to arrest him upon the information given by

Consti II (section 2) 54
FACTS: The accused-appellants are questioning their
conviction by the RTC of robbery with double homicide. The
contend that the trial court erred in holding that the crime ISSUE: WON the accused was reminded of his constitutional
committed is robbery with double homicide despite safeguards?
insufficiency of evidence, in not holding that the evidence
obtained against them is inadmissible for violation of their
constitutional rights to remain silent, to counsel and against
self-incrimination during custodial investigation, and in not RULING: The Court, however, must express its dismay over
holding that their guilt was not proved beyond reasonable the questionable means employed by the police in
doubt. They likewise argue that their arrest was illegal for investigating the accused. The police officers concerned
having been made without a warrant. mocked the Constitution, which they themselves have sworn
to honor and revere, when they did not remind the accused of
their right to remain silent and to be assisted by counsel. They
must be instructed by their superiors in no uncertain terms to
ISSUE: WON the court erred without considering the validity respect the Constitution at all times in the performance of their
of the arrest. duties. Be that as it may, this unconstitutional act may not
benefit the cause of the accused. After all, nothing in the
evidence for the prosecution was taken from the police
investigation in question. Also, in the course of his testimony,
RULING: The constitutional rights of the appellants to police officer Elpidio Luna was rebuked by the trial court for
remain silent and to counsel, the same cannot be held to have his violation of the constitutional rights of accused.
been violated. It is not disputed that the appellants were
investigated by the police. However, it is important to note
that the confession, admission or evidence obtained from the
appellants was never offered in evidence by the prosecution. 3.45 People v. Nazareno – 260 SCRA 256
on appellants' claim that since their warrantless arrest is void,
all the other proceedings, including their conviction, are also
void, We find such claim undeserving of merit. It is
unequivocally clear that no valid arrest was made on the FACTS: Accused-appellants Narciso Nazareno and Ramil
accused-appellants, the arrest having been made without any Regala was found guilty of murder for the killing of Romulo
warrant at all. Neither can the appellants' arrest qualify as Bunye. Both the accused claimed that they were tortured to
lawful arrest without a warrant under Sec. 5 (b) of Rule 113 of admit to the crime. The trial court ruled the confessions of the
the Rules on Criminal Procedure because the police officer accused to be inadmissible. However, it held Regala and
effected the arrest indubitably had no personal knowledge of Nazareno guilty on the basis of their positive identification by
facts indicating that the person to be arrested has committed witnesses and their testimony in court. Accused-appellant
the crime. It is eyewitnesses Francisco who had such personal Ramil Regala contends that the court erred in not considering
knowledge. In like manner, we cannot accept appellee's the unlawful arrest. Accused-appellants claim that their
allegation that Briones was a fugitive from justice at the time arrests without warrant were illegal and justify the
of the latter's arrest because it is not supported by the evidence nullification of the proceedings of the trial court. It was
on record. In sum, therefore, the warrantless arrest of the alleged that Nazarenos arrest was illegal because it was made
appellants is illegal. Nevertheless, such unavailing technicality without warrant fourteen days after the killing of Romulo
cannot render all the other proceedings, including the Bunye II.
conviction of the appellants, void. It cannot deprive the state
of its right to convict the guilty when all the facts on record
point to their culpability. ISSUE: W/N the court erred in not considering the unlawful
arrest.

3.44 People v. Sequino – 264 SCRA 79


RULING: The Court already held that the warrantless arrest
was in accordance with Rule 113, 5(b) of the Revised Rules of
FACTS: Accused was found guilty by the trial court for the Criminal Procedure, A peace officer or a private person may,
charges of robbery with homicide when he did and there steal without a warrant, arrest a person: (b) When an offense has
and carry away the payroll money of Jose, and on the occasion just been committed, and he has probable cause to believe
thereof, shot Pedro who was back riding, thereby inflicting based on personal knowledge of facts or circumstances that
upon him a gunshot wound on the head, resulting to Pedro’s the person to be arrested has committed it;
death. Accused contends that the prosecution failed to
consider the non-observance of the Constitution in the
investigation with the accused by the police when they did not 4.21 THE PEOPLE OF THE PHILIPPINES,plaintiff-
remind the accused of their right to remain silent and to be appellee, vs. SAMUEL MAHUSAY y FLORES AND
assisted by counsel
Consti II (section 2) 55
CRISTITUTO PASPOS Toto y CATARIG,accused-
appellants.

Facts:Appellants Samuel Mahusay and Cristituto Paspos, 4.22 THE PEOPLE OF THE PHILIPPINES,plaintiff-appellee,
along with Felomino Galo, Alfredo Mendio, Justiniano vs. ARMANDO ALVARIO,accused-appellant.
Velacsi (at large), and Tanciong Egloba (at large), were Facts:ARMANDO ALVARIO guilty of the crime of
charged with the crime of robbery with rape and physical Rape.However,Alvario consistently protested the his arrest
injuries in Criminal Case No. N-1283 before the Regional was made without a valid warrant of arrest.
Trial Court of Naval, Subprovince of Biliran, Leyte, Branch
16, under an information dated July 19, 1988.However,after
the final judgement they contended that the arrest made was a
Issue:Whether the warrantless arrest maid was valid?
warrantless one.

Issue:Whether the arrest made was valid?


Ruling:Suffice it to say that his arrest falls within the purview
RulingSection 5.Arrest without warrant; when lawful. - A of Rule 113, Section 5(b) of the 1985 Rules on Criminal
peace officer or a private person may, without a warrant, arrest Procedure,viz.:
a person: Sec. 5.Arrest without warrant; when lawful. - A peace officer
(a) When, in his presence, the person to be arrested has or a private person may, without a warrant, arrest a person:
committed, is actually committing, or is attempting to commit
an offense; (b)When an offense has in fact just been committed, and he
has personal knowledge of facts indicating that the person to
(b) When an offense has in fact just been committed, and he be arrested has committed it.
has personal knowledge of facts indicating that the person to
be arrested has committed it; The personal knowledge of the arresting officers in the case at
bar was culled from the information supplied by the victim
(c) When the person to be arrested is a prisoner who has herself who pointed to Alvario as the man who raped her at
escaped from a penal establishment or place where he is the time of his arrest
serving final judgment or temporarily confined while his case
is pending, or has escaped while being transferred from one
confinement to another.
4.23 Larranaga Vs CA
Under Section 5(b) of the aforequoted rule, two conditions
Facts:On October 1, 1997, petitioner, represented by his
must concur for a warrantless arrest to be valid:first, the
mother, Margarita G. Larranaga,filed with this Court a petition
person to be arrested must have just committed an offense, and
for certiorari, prohibition and mandamus with writs of
second, the arresting peace officer or private person must have
preliminary prohibitory and mandatory injunction.Petitioner
personal knowledge of facts indicating that the person to be
alleged that he was denied the right to preliminary
arrested is the one who committed the offense.It has been
investigation and sought to annul the informations as well as
ruled that personal knowledge of facts in arrests without a
the warrant of arrest issued in consequence thereof.In the
warrant must be based upon probable cause, which means an
alternative, petitioner prayed that a preliminary investigation
actual belief or reasonable grounds of suspicion.
be conducted and that he be released from detention pending
In the case at bar, appellants were arrested on the sole basis of the investigation.Petitioner filed a supplemental petition for
Bughaos verbal report.The arresting officers were led to habeas corpus or bail on October 6, 1997
suspect that, indeed, appellants had committed a crime.Thus,
the arrest was made in violation of their fundamental right
against an unjustified warrantless arrest.This notwithstanding, Issue:Whether the petitioner is entitled for a preliminary
appellants cannot find comfort solely in this error. investigation?
The Court has ruled on several occasions that any objection Ruling:The rule is that the right to preliminary investigation is
involving a warrant of arrest or the procedure in the waived when the accused fails to invoke it before or at the
acquisition by the court of jurisdiction over the person of an time of entering a plea at arraignment.Petitioner, in this case,
accused must be made before he enters his plea, otherwise the has been actively and consistently demanding a regular
objection is deemed waived.Indeed, it is too late for appellants preliminary investigation even before he was charged in
to raise the question of their arrest.When appellants were court.Also, petitioner refused to enter a plea during the
arrested and a case was filed against them, they pleaded not arraignment because there was a pending case in this Court
guilty upon arraignment, participated in the trial and presented regarding his right to avail of a regular preliminary
their respective evidence.Appellants are thus estopped from investigation.Clearly, the acts of petitioner and his counsel are
questioning the legality of their arrest.At the outset, appellants inconsistent with a waiver.Preliminary investigation is part of
should have moved for the quashal of the information before procedural due process.It cannot be waived unless the waiver
the trial court on this ground.Accordingly, any irregularity in appears to be clear and informed.
their arrest was cured when they voluntarily submitted
themselves to the jurisdiction of the trial court.
Consti II (section 2) 56
Issue:whether the petitioner should be released from detention were not doing nor had just done any criminal act. Neither
pending the investigation were they caught in flagrante delicto or had escaped from
confinement.Probably aware of the illegality of the arrest they
Ruling:The records show that on September 17, 1997, two made, the arresting officers testified that appellants were
informations were filed against petitioner for kidnapping and merely invited to the police precinct. Such invitation,
serious illegal detention.Executive Judge Priscila Agana however, when construed in the light of the circumstances is
issued a warrant of arrest on September 19, 1997. was arrested actually in the nature of an arrest designed for the purpose of
on September 22, 1997 by virtue of said warrant.We held in conducting an interrogation.Mere invitation is covered by the
Sanchez v. Demetriou the filing of charges and the issuance of proscription on a warrantless arrest because it is intended for
the warrant of arrest against a person invalidly detained will no other reason than to conduct an investigation. Thus,
cure the defect of that detention or at least deny him the right pursuant to Section 4(2), Article IV of the 1973 Constitution
to be released because of such defect.The Court ruled: which was in effect at that time, any evidence obtained in
The original warrantless arrest of the petitioner was doubtless violation of their right under Section 3, Article IV (pertaining
illegal.Nevertheless, the Regional Trial Court lawfully to invalid warrantess arrest)shall be inadmissible for any
acquired jurisdiction over the person of the petitioner by virtue purpose in any proceeding.By virtue of said constitutional
of the warrant of arrest it issued on August 26, 1993 against protection, any evidence obtained, including all the things and
him and the other accused in connection with the rape-slay properties alleged to be stolen by appellants which were taken
cases.It was belated, to be sure, but it was nonetheless legal. by the police from the place of the illegal arrest cannot be used
as evidence for their conviction.In the same manner, all the
Even on the assumption that no warrant was issued at all, we products of those illegal arrests cannot be utilized to sustain
find that the trial court still lawfully acquired jurisdiction over any civil liability that they may have incurred by reason of
the person of the petitioner.The rule is that if the accused their acts.This is the clear mandate of the Constitution when it
objects to the jurisdiction of the court over his person, he may provides that those illegally obtained evidence being the fruits
move to quash the information, but only on that ground.If, as of the poisonous tree are inadmissible for any purpose in any
in this case, the accused raises other grounds in the motion to proceeding.The foregoing constitutional protection on the
quash, he is deemed to have waived that objection and to have inadmissibility of evidence (which are the product of an illegal
submitted his person to the jurisdiction of the court. search and arrest) known as the exclusionary rule, applies not
only to criminal cases but even extends to civil, administrative
and any other form of proceedings.No distinction is made by
4.24 People Vs.Olivarez the Constitution; this Court ought not to distinguish.

Facts:Involved in this case is the crime of robbery with


homicide committed during the season of yuletide .Convicting
appellants of the crime charged, sentenced them to suffer the 4.25 Cadua Vs CA
death penalty and to indemnify the victims heirs..However, In
this case, there were no eyewitnesses to the killing and
robbery and; thus, no direct evidence points to appellants Facts:In the evening of January PO3 Joselito Burdeos and
criminal liability.The prosecutions principal evidence against companions, all assigned with the Central Police District in
them is based solely on the testimony of the police officers Quezon City, were aboard mobile unit patrolling the vicinity
who arrested, investigated and subsequently took their of Fairview, Quezon City.Their tour of duty was from 3:00
confession.Such evidence when juxtaposed with appellants p.m. to 11:00 p.m.While deployed, they received a radio
constitutional rights concerning arrests and the taking of dispatch requesting them to proceed to. North Fairview.Said
confessions leads to a conclusion that they cannot he held dispatch was based on a report concerning an alleged holdup
liable for the offense charged despite the inherent weakness of of complainants Lourdes Bulos and her daughter Bernadette,
their defenses of denial and alibi, not because they are not who were in need of police assistance.At said address, police
guilty but because the evidence adduced against them are officers found both complainants who stated that the alleged
inadmissible to sustain a criminal conviction. holduppers had just fled.PO3 Burdeos asked where the
robbery took place.Complainants replied that they were held
up by two (2) men at the corner of Archer and Regalado
Streets, near their house.The police officers also asked in what
Issue: Whether warrantless arrest qualifies in the case at bar
direction the alleged holduppers fled and what they were
Ruling:First, appellants were arrested without a valid a valid wearing.Then, the police officers requested the complainants
warrant of arrest and their arrest cannot even be justified under to board the patrol unit in order to facilitate the search for the
any of the recognized exceptions for a valid warrantless arrest two (2) men.As they were patrolling around the area,
mentioned in Section 6, (now Section 5) Rule 113 of the Rules complainants informed the police officers that one of the
on Criminal Procedure suspects was dressed in jeans and a t-shirt while the other was
dressed in a black top and black pants.The police officers then
None of the foregoing exceptions for a valid warrantless arrest noticed two (2) men walking alongside the street and as the
concurs herein.At the time appellants were apprehended, two officers slowed down the mobile unit to get a closer look, the
days had already lapsed after the discovery of the crime they complainants identified the men as the alleged holduppers, one
Consti II (section 2) 57
of which is the petitioner in this case.The police officers breach of the peace.Probable cause for an arrest without
slowed down to a stop, alighted from the vehicle, and called warrant is such a reasonable ground of suspicion supported by
out to the suspects.As Burdeos was approaching the suspects, circumstances sufficiently strong in themselves as to warrant a
he noticed that petitioner Cadua was about to pull something reasonable man in believing the accused to be guilty.Besides
which was tucked at the right side of his waist.Burdeos reasonable ground of suspicion, action in good faith is another
promptly pointed his firearm at Cadua and warned him not to protective bulwark for the officer.Under such conditions, even
move.He then frisked Cadua and found in his possession a .38 if the suspected person is later found to be innocent, the peace
caliber paltik revolver.PO3 Reynoso Bacnat then apprehended officer is not liable.The cases hold that a peace officer might
Caduas companion, who was later identified as Joselito arrest and detain in prison for examination persons walking in
Aguilar.In Aguilars possession was found a fan the street at night whom there is reasonable ground to suspect
knife.According to petitioner, since his arrest is null and void, of felony, although there is no proof of a felony having been
the search conducted by the police officers as an incident to committed; but the arrest would be illegal if the person so
his arrest is likewise defective.In support of his claim, arrested was innocent and there were no reasonable grounds of
petitioner seeks to invoke his constitutional right to be secure suspicion to mislead the officer.The reason of the rule is
against unreasonable searches and seizures,and the apparent.Good people do not ordinarily lurk about the streets
corresponding prohibition against admitting into evidence and uninhabited premises at midnight.Citizens must be
anything obtained in violation of such right. protected from annoyance and crime.Prevention of crime is
just as commendatory as the capture of criminals.Surely the
Issue:whether or not his right to be protected from any officer must not be forced to await the commission of robbery
unlawful warrantless arrest has been violated. or other felony.The rule is supported by the necessities of life.
Ruling:we find that there was sufficient reason to justify a
warrantless arrest of petitioner for illegal possession of
firearms. Section 5 of Rule 113 of the Rules of Court, provides
that:
4.26 (name of case) People v. Cubcubin
Sec. 5.Arrest without warrant; when lawful.A peace officer or
a private person may, without a warrant, arrest a person: Facts:On August 26, 1997, in the City of Cavite, Republic of
the Philippines and within the jurisdiction of this Honorable
(a)When, in his presence, the person to be arrested has Court, Fidel Abrenica Cubcubin , armed with an unlicensed
committed, is actually committing, or is attempting to commit homemade (paltik) Smith and Wesson caliber .38 revolver,
an offense; with no serial number, with intent to kill, acting with treachery
and evident premeditation and taking advantage of the
(b)When an offense has in fact just been committed, and he darkness of [the] night, did, then and there, willfully,
has personal knowledge of facts indicating that the person to unlawfully, and feloniously, assault, attack and shoot with the
be arrested has committed it; and aforesaid unlicensed firearm a certain HENRY PECHO
(c)When the person to be arrested is a prisoner who has PIAMONTE, hitting and inflicting upon the latter gunshot
escaped from a penal establishment or place where he is wounds in the head which caused the latters instantaneous
serving final judgment or temporarily confined while his case death.October 5, 1998, of the Regional Trial Court, Branch 88,
is pending, or has escaped while being transferred from one Cavite City,finding accused-appellant Fidel Abrenica
confinement to another. Cubcubin, Jr. guilty of murder and sentencing him to suffer
the penalty of death. Accused-appellant contends that his
In cases falling under paragraph (a) and (b) hereof, the person arrest, effected on August 26, 1997 without a warrant, was
arrested without a warrant shall be forthwith delivered to the illegal.
nearest police station or jail, and he shall be proceeded against
in accordance with Rule 112, Section 7.
The findings of the trial court, accepted by the appellate court, Issue:Whether the petitioner’s arrest was valid?
show the pertinence of paragraphs (a) and (b) of Section 5 Ruling:On this point, Rule 113, 5(b) of the 1985 Rules on
abovecited.Through police dispatch to the scene of a crime Criminal Procedure, as amended, provides:
report and in the presence of complainants, it was ascertained
that a robbery had just been committed, and the arresting Sec. 5.without warrant; when lawful.A peace officer or a
officers had personal knowledge that petitioner was directly private person may, without a warrant, arrest a person:
implicated as a suspect.As explained by a respected authority
on criminal procedure: (a) When, in his presence, the person to be arrested has
committed, is actually committing, or is attempting to commit
It has been ruled that personal knowledge of facts, in arrests an offense;
without warrant must be based upon probable cause, which
means an actual belief or reasonable grounds of (b) When an offense has in fact just been committed, and he
suspicion.Peace officers may pursue and arrest without has personal knowledge of facts indicating that the person to
warrant any person found in suspicious places or under be arrested has committed it;
suspicious circumstances reasonably tending to show that such (c) When the person to be arrested is a prisoner who has
person has committed, or is about to commit, any crime or escaped from a penal establishment or place where he is

Consti II (section 2) 58
serving final judgment or temporarily confined while his case inviolable, and no search warrant or warrant of arrest shall
is pending, or has escaped while being transferred from one issue except upon probable cause to be determined personally
confinement to another. by the judge after examination under oath or affirmation of the
complainant and the witnesses he may produce, and
.Under 5(b), two conditions must concur for a warrantless particularly describing the place to be searched and the
arrest to be valid:first, the offender has just committed an persons or things to be seized.
offense and,second, the arresting peace officer or private
person has personal knowledge of facts indicating that the Sec. 3.
person to be arrested has committed it.It has been held that
personal knowledge of facts in arrests without a warrant must (2)Any evidence obtained in violation of this or the preceding
be based upon probable cause, which means an actual belief or section shall be inadmissible for any purpose in any
reasonable grounds of suspicion proceeding.
Said constitutional provisions are safeguards against reckless,
malicious and unreasonable invasion of privacy and
4.27 PEOPLE OF THE PHILIPPINES, plaintiff-appellee, liberty.The Court, in Villanueva v. Querubin,underscored their
vs.ARMANDO COMPACION y SURPOSA,accused- importance:
appellant.
It is deference to ones personality that lies at the core of this
right, but it could be also looked upon as a recognition of a
constitutionally protected area, primarily ones home, but not
Facts:Acting on a confidential tip supplied by a police necessarily thereto confined.What is sought to be guarded is a
informant that accused-appellant was growing and cultivating mans prerogative to choose who is allowed entry to his
marijuana plants, SPO1 Gilbert L. Linda and SPO2 Basilio residence.
Sarong of the 6th Narcotic Regional Field Unit of the Narcotics
Command (NARCOM) of the Bacolod City Detachment A search and seizure, therefore, must be carried out through or
conducted a surveillance of the residence of accused-appellant with a judicial warrant; otherwise, such search and seizure
who was then the barangay captain of barangay Bagonbon, becomes unreasonable within the meaning of the
San Carlos City, Negros Occidental on July 9, 1995.During constitutional provision.Evidence secured thereby, i.e., the
the said surveillance, they saw two (2) tall plants in the fruits of the search and seizure, will be inadmissible in
backyard of the accused-appellant which they suspected to be evidence for any purpose in any proceeding.
marijuana plants.SPO1 Linda and SPO2 Sarong reported the
result of their surveillance to SPO4 Ranulfo T. Villamor, Jr., The requirement that a warrant must be obtained from the
Chief of NARCOM, Bacolod City, who immediately formed a proper judicial authority prior to the conduct of a search and
team composed of the members of the Intelligence Division seizure is, however, not absolute.There are several instances
Provincial Command, the Criminal Investigation Command when the law recognizes exceptions, such as when the owner
and the Special Action Force.Two members of the media, one of the premises consents or voluntarily submits to a
from DYWF Radio and another from DYRL Radio, were also search;when the owner of the premises waives his right
included in the composite team.On July 12, 1995, the team against such incursion;when the search is incidental to a
applied for a search warrant with the office of Executive Judge lawful arrest;when it is made on vessels and aircraft for
Bernardo Ponferrada in Bacolod City.However, Judge violation of customs laws;when it is made on automobiles for
Ponferrada informed them that he did not have territorial the purpose of preventing violations of smuggling or
jurisdiction over the matter.The team then left Bacolod City immigration laws;when it involves prohibited articles in plain
for San Carlos City.They arrived there around six-thirty in the view;when it involves a stop and frisk situation;when the
evening, then went to the house of Executive Judge Roberto S. search is under exigent and emergency circumstances;or in
Javellana to secure a search warrant.They were not able to do cases of inspection of buildings and other premises for the
so because it was nighttime and office hours were obviously enforcement of fire, sanitary and building regulations.In these
over.They were told by the judge to go back in the instances, a search may be validly made even without a
morning.Nonetheless, the team proceeded to barangay warrant.
Bagonbon and arrived at the residence of accused-appellant in In the instant case, the search and seizure conducted by the
the early morning of July 13, 1995.SPO4 Villamor knocked at composite team in the house of accused-appellant was not
the gate and called out for the accused-appellant authorized by a search warrant.It does not appear either that
Issue:whether accused-appelant’s constitutional right against the situation falls under any of the above mentioned
unreasonable searches and seizures had been violated by the cases.Consequently, accused-appellants right against
police authorities. unreasonable search and seizure was clearly violated.

Ruling:The relevant constitutional provisions are found in It is extant from the records that accused-appellant did not
Sections 2 and 3 [2], Article III of the 1987 Constitution consent to the warrantless search and seizure conducted.While
which read as follows: the right to be secure from unreasonable search and seizure
may, like every right, be waived either expressly or
Sec. 2.The right of the people to be secure in their persons, impliedly,such waiver must constitute a valid waiver made
houses, papers, and effects against unreasonable searches and voluntarily, knowingly and intelligently.The act of the
seizures of whatever nature and for any purpose shall be accused-appellant in allowing the members of the military to
Consti II (section 2) 59
enter his premises and his consequent silence during the Ruling:In view of Art. III, 2 of the Constitution, the rule is that
unreasonable search and seizure could not be construed as no arrest may be made except by virtue of a warrant issued by
voluntary submission or an implied acquiescence to a judge after examining the complainant and the witnesses he
warrantless search and seizure especially so when members of may produce and after finding probable cause to believe that
the raiding team were intimidatingly numerous and heavily the person to be arrested has committed the crime.The
armed.His implied acquiescence, if any, could not have been exceptions when an arrest may be made even without a
more than mere passive conformity given under coercive or warrant are provided in Rule 113, 5 of the Rules of Criminal
intimidating circumstances and is, thus, considered no consent Procedure .
at all within the purview of the constitutional
guarantee.Consequently, herein accused-appellants lack of Second,We have already explained what constitutes "personal
objection to the search and seizure is not tantamount to a knowledge" on the part of the arresting officers:"Personal
waiver of his constitutional right or a voluntary submission to knowledge" of facts in arrests without a warrant under Section
the warrantless search and seizure.The case of People v. 5 (b) of Rule 113 must be based upon "probable cause" which
Burgos,is instructive.In Burgos, the Court ruled that the means an "actual belief or reasonable grounds of suspicion."
accused is not to be presumed to have waived the unlawful The grounds of suspicion are reasonable when, in the absence
search simply because he failed to object. There, we held: To of actual belief of the arresting officers, the suspicion that the
constitute a waiver, it must appear first that the right exists; person to be arrested is probably guilty of committing the
secondly, that the person involved had knowledge, actual or offense is based on actual facts,i.e.,supported by
constructive, of the existence of such a right; and lastly, that circumstances sufficiently strong in themselves to create the
said person had an actual intention to relinquish the right probable cause of guilt of the person to be arrested.A
(Pasion Vda. De Garcia v. Locsin, 65) reasonable suspicion therefore must be founded on probable
cause, coupled with good faith on the part of the peace officers
making the arrest.

4.28 ROGER POSADAS, ROSARIO TORRES-YU, and


MARICHU LAMBINO, petitioners, vs. THE HON.
OMBUDSMAN, THE SPECIAL PROSECUTOR, and
ORLANDO V. DIZON, respondents.

4.29 People Vs Acol


Facts:Dennis Venturina, a member of Sigma Rho at the Facts:The People's inculpatory accusations during the joint
University of the Philippines, was killed in a rumble between trial were to the effect that at around 3:45 in the morning of
his fraternity and another fraternity on December 8, 1994.In a September 29, 1990, when Percival Tan was driving his
letter dated December 11, 1994, petitioner Roger Posadas, jeepney, two men boarded the vehicle in Cubao. When they
then Chancellor of U.P. Diliman in Quezon City, asked the crossed Pasay Road, the two wayfarers, together with two
Director of the National Bureau of Investigation for assistance other companions, announced a hold-up. Percival Tan was
in determining the persons responsible for the crime.In instructed to proceed atop the Magallanes interchange where
response to the request, respondent Orlando V. Dizon, Chief the other passengers were divested of their personal
of the Special Operations Group of the NBI, and his men went belongings, including the jacket of passenger Rene Araneta.
to U.P. on December 12 and, on the basis of the supposed Thereafter, the robbers alighted at the Shell Gas Station near
positive identification of two alleged eyewitnesses, Leandro the Magallanes Commercial Center after which Percival Tan
Lachica and Cesar Mangrobang, Jr., attempted to arrest and his passengers went to Fort Bonifacio to report the crime.
Francis Carlo Taparan and Raymundo Narag, A CAPCOM team was forthwith formed to track down the
officers/members of the Scintilla Juris Fraternity, as suspects culprits. Victim Rene Araneta who went with the responding
in the killing of Venturina.It appears that the two suspects had police officers, upon seeing four persons, one of whom was
come that day to the U.P. Police Station for a peace talk wearing his stolen jacket, walking casually towards Fort
between their fraternity and the Sigma Rho Fraternity. The Bonifacio, told the police authorities to accost said persons.
NBI agents in the case at bar tried to arrest Narag and Taparan After the CAPCOM officers introduced themselves, the four
four days after the commission of the crime.They had no men scampered to different directions but three of them,
personal knowledge of any fact which might indicate that the namely, Tirso Acol, Pio Boses, and Albert Blanco, were
two students were probably guilty of the crime.What they had apprehended. Tirso Acol and Pio Boses were each found in
were the supposed positive identification of two alleged possession of an unlicensed .38 caliber revolver with bullets.
eyewitnesses, which is insufficient to justify the arrest without After the arrest, the three men were brought to Fort Bonifacio
a warrant by the NBI. and were identified by Percival Tan and the passengers who
ganged up on the accused.

Issues: Whether the attempted arrest of the student suspects by Issue: Whether the warrantless arrest was valid?
the NBI could be validly made without a warrant Ruling:With respect to the so-called warrantless arrest of
accused--appellant, we are of the view that the search falls

Consti II (section 2) 60
within the purview of Section 5(b) of Rule 113 which serves honorable court erred in rendering two counts of murder and
as an exception to the requisite warrant prior to arrest:When one count of frustrated murder.
an offense has in fact been committed, and the has
personal knowledge of facts indicating that the person to be Issue:Whether the court erred in rendering its decision
arrested has committed it;inasmuch as the police team was
formed and dispatched to look for the persons responsible for
the crime on account of the information related by Percival Ruling:The Information is formally defective as it charged
Tan and Rene Araneta that they had just been robbed (People more the one offense in violation of Rule 110, 13 of the
vs. Gerente, 219 SCRA 756 [1993]; People vs. Tonog, Jr., 205 Revised Rules of Court.However, because of his failure to file
SCRA 772 [1992]). And since accused-appellant's arrest was a motion to quash, accused-appellant is deemed to have
lawful, it follows that the search made incidental thereto was waived objection based on the ground of duplicity.The
valid (People vs. Tanilon,221 SCRA 671 [1993]). Moreover, dispositive portion of the trial courts decision finds accused-
the unlicensed firearms were found when the police team appellant guilty of Double Murder with Frustrated Murder, but
apprehended the accused for the robbery and not for illegal sentences him for two separate counts of murder and one
possession of firearms and ammunition (People vs. Cruz,165 count of frustrated homicide.We hold that accused-appellant
SCRA 135 [1988]). The principle imparted by Justice Padilla was guilty of two counts of murder and one count of attempted
in Cruz was based on the ruling of this Court in Magoncia vs. murder.Under Art. 48 of the Revised Penal Code, a complex
Palacio(90 Phil. 771 [1948]) that:When, in pursuing an illegal crime is committed only when a single act constitutes two or
action or in the commission of a criminal offense, the more grave or less grave felonies. As the victims in this case
offending police officers should happen to discover a criminal were successively shot by accused-appellant with a shotgun,
offense being committed by any person, they are not each shot necessarily constitutes one act.Accused-appellant
precluded from performing their duties as police officers for should thus be held liable for three separate crimes.
the apprehension of the guilty person and the taking of the
The lower court correctly appreciated treachery as having
corpus delicti. qualified the killings of Rogelio Seldera and Rodolfo
Padapat.The essence of treachery is the swift and unexpected
attack on an unarmed victim without the slightest provocation
4.30 People Vs Rabang on the part of the victim.Here, it was clearly established that
Facts:The case before the Court is an appeal taken by accused the victims, when shot, were unarmed and were peacefully
Maximo (Dagit) Rabang, Jr. from the decision of the Regional walking along a trail when accused-appellant suddenly opened
Trial Court, Branch 7, Aparri, Cagayan, convicting him of fire on them.The swiftness of the shooting left them helpless
murder, and sentencing him to reclusion perpetua,and to pay to put up any form of defense.
the heirs of the victim Floramante Talaro the amount of fifty The lower court however erred in convicting accused-
thousand (P50,000.00) pesos as death compensation, without appellant of frustrated homicide for the injuries inflicted on
subsidiary imprisonment in case of insolvency, and to pay the Mario Seldera.Although it correctly appreciated the intent to
costs. Accused-appellant also assails the trial courts finding kill, which can be inferred from the weapon used, the
that treachery qualified the killing to murder.He interposes the proximity of the assailants and the location of the injuries, it
defense of alibi. should have appreciated treachery in the attack.Treachery
Issue:Whether the trial court erred in finding that treachery attended the shooting not only of Rogelio Seldera and Rodolfo
qualified the killing Padapat but also of Mario Seldera.

Ruling:We do not agree.The trial court was in the best position


to evaluate the credibility of the witnesses presented before it 4.32 Velasco Vs CA
for it had the opportunity to observe the witnesses deportment
on the stand and the manner in which they gave their Facts:This is a petition for certiorari and mandamus filed by
testimonies.Thus, in the absence of showing that serious and Lorenzo Velasco and Socorro J. Velasco (hereinafter referred
substantial errors were committed by the lower court in the to as the petitioners) against the resolution of the Court of
appraisal of the evidence before it, the trial judges assessment Appeals dated June 28, 1969 in CA-G.R. 42376, which
of the credibility of the witnesses is accorded great weight and ordered the dismissal of the appeal interposed by the
respect. petitioners from a decision of the Court of First Instance of
Quezon City on the ground that they had failed seasonably to
file their printed record on appeal. According to the petitioner
4.31 People Vs Lopez the CA committed grave abuse of discretion when it dismissed
the appeal of the petitioner.
Issue:Whether the court acted with grave abuse of discretion
Facts:This is an appeal from the decision of the Regional Trial amounting to lack of jurisdiction
Court of Pangasinan (Branch 52), finding accused-appellant
guilty of two counts of murder and one count of frustrated Ruling:This Court held in Bello vs. Fernando that the right to
murder and ordering him to pay a total of P204,300.00 in appeal is nota natural right nor a part of due process; it is
damages. In this appeal, accused-appellant alleges that the merely a statutory privilege, and may be exercised only in the
manner provided by law. In this connection, the Rule of Court
Consti II (section 2) 61
expressly makes it the duty of an appellant to file a printed
record on appeal with the Court of Appeals within sixty (60)
record on appeal approved by the trial court has already been
received by the said court. Thus, section 5 of Rule 46 states:
Sec. 5.Duty of appellant upon receipt of
notice. — It shall be the duty of the appellant
within fifteen (15) days from the date of the
notice referred to in the preceding section, to
pay the clerk of the Court of Appeals the fee
for the docketing of the appeal, and within
sixty (60) days from such notice to submit to
the court forty (40) printed copies of the
record on appeal, together with proof of
service of fifteen (15) printed copies thereof
upon the appelee.
As the petitioners failed to comply with the above-mentioned
duty which the Rules of Court enjoins, and considering that, as
found by the Court of Appeals, there was a deliberate effort on
their part to mislead the said Court in grating them an
extension of time within which to file their printed record on
appeal, it stands to reason that the appellate court cannot be
said to have abused its discretion or to have acted without or
in excess of its jurisdiction in ordering the dismissal of their
appeal
4.33 People Vs Buluran
Facts:On appeal is the decision dated February 4, 1994, of the
Regional Trial Court of Quezon City, Branch 95, convicting
accused-appellants of the crime of murder and sentencing
them to suffer the penalty of reclusion perpetua, to pay
solidarily the heirs of the deceased the amount of P50,000.00
as indemnity, and P8,000.00 as actual damages, and also to
pay proportionately the costs. The petitioner raised the
following the court erred in convicting the appellants who
instead are entitled to acquittal on grounds of violation of their
constitutional rights and procedural rights to due process
which divested the court of jurisdiction
Issue: Whether the court erred in convicting the appellants
who instead are entitled to acquittal on grounds of violation of
their constitutional rights and procedural rights to due process
which divested the court of jurisdiction
Ruling: There is no violation of the constitutional rights of the
accused during custodial investigation since neither one
executed an extrajudicial confession or admission. In fact, the
records show that appellant Cielito Buluran opted to remain
silent during the custodial investigation. Any allegation of
violation of rights during custodial investigation is relevant
and material only to cases in which an extrajudicial admission
or confession extracted from the accused becomes the basis of
their conviction.In this case, the basis of the conviction by the
trial court was the testimonies of the three eyewitnesses,
Artemio Avendao, Jacinto Castillo, and Gloria Castillo. It is
noteworthy that appellants never attempted to impeach their
testimonies during trial. Neither do they assail the credibility
of said witnesses on appeal. The failure to accord appellants
their right to preliminary investigation did not impair the
validity of the information nor affect the jurisdiction of the
trial court.While the right to preliminary investigation is a
substantive right and not a mere formal or technical right of
Consti II (section 2) 62

You might also like